IMMIGRATION LAW REPORTER Fourth Series/Quatri`eme s´erie Recueil de jurisprudence en droit de l’immigration VOLUME 3 (Cited 3 Imm. L.R. (4th))

EDITORS-IN-CHIEF/REDACTEURS´ EN CHEF Cecil L. Rotenberg, Q.C. Mario D. Bellissimo, LL.B. Barrister & Solicitor Ormston, Bellissimo, Rotenberg Don Mills, Ontario Toronto, Ontario Certified Specialist Certified Specialist

ASSOCIATE EDITOR/REDACTEUR´ ADJOINT Randolph Hahn, D.PHIL.(OXON), LL.B. Guberman, Garson Toronto, Ontario Certified Specialist

CARSWELL EDITORIAL STAFF/REDACTION´ DE CARSWELL Cheryl L. McPherson, B.A.(HON.) Director, Primary Content Operations Directrice des activit´es li´ees au contenu principal Graham B. Peddie, LL.B. Product Development Manager Sharon Yale, LL.B., M.A. Jennifer Weinberger, B.A.(HON.), Supervisor, Legal Writing J.D. Supervisor, Legal Writing Peter Bondy, B.A.(HON.), LL.B. Heather Stone, B.A., LL.B. Lead Legal Writer Lead Legal Writer Rachel Bernstein, B.A.(HON.), J.D. Peggy Gibbons, B.A.(HON.), LL.B. Legal Writer Senior Legal Writer Jim Fitch, B.A., LL.B. Stephanie Hanna, B.A., M.A., LL.B. Senior Legal Writer Senior Legal Writer Mark Koskie, B.A.(HON.), M.A., LL.B. Nicole Ross, B.A., LL.B. Legal Writer Legal Writer Amanda Stewart, B.A.(HON.), LL.B. Martin-Fran¸cois Parent, LL.B., Senior Legal Writer LL.M., DEA (PARIS II) Bilingual Legal Writer Erin McIntosh, B.A.(HON.) Content Editor IMMIGRATION LAW REPORTER, a national series of topical law reports, Recueil de jurisprudence en droit de l’immigration, une s´erie nationale de is published twelve times per year. Subscription rate $379 per bound volume recueils de jurisprudence sp´ecialis´ee, est publi´e 12 fois par anne´e. including parts. Indexed: Carswell’s Index to Canadian Legal Literature. L’abonnement est de 379 $ par volume reli´e incluant les fascicules. Indexa- tion : Index a` la documentation juridique au Canada de Carswell.

Editorial Offices are also located at the following address: 430 rue St. Pierre, Le bureau de la r´edaction est situ´e a` Montr´eal — 430, rue St. Pierre, Mon- Montr´eal, Qu´ebec, H2Y 2M5. tr´eal, Qu´ebec, H2Y 2M5.

______© 2012 Thomson Reuters Canada Limited © 2012 Thomson Reuters Canada Limit´ee

NOTICE AND DISCLAIMER: All rights reserved. No part of this publica- MISE EN GARDE ET AVIS D’EXONERATION´ DE RESPON- tion may be reproduced, stored in a retrieval system, or transmitted, in any SABILITE´ : Tous droits r´eserv´es. Il est interdit de reproduire, m´emoriser sur form or by any means, electronic, mechanical, photocopying, recording or un syst`eme d’extraction de donn´ees ou de transmettre, sous quelque forme ou otherwise, without the prior written consent of the publisher (Carswell). par quelque moyen que ce soit, electronique´ ou m´ecanique, photocopie, enre- gistrement ou autre, tout ou partie de la pr´esente publication, a` moins d’en avoir pr´ealablement obtenu l’autorisation ecrite´ de l’´editeur, Carswell. A licence, however, is hereby given by the publisher: Cependant, l’´editeur conc`ede, par le pr´esent document, une licence :

(a) to a lawyer to make a copy of any part of this publication to give to a a) a ` un avocat, pour reproduire quelque partie de cette publication pour judge or other presiding officer or to other parties in making legal submis- remettre a` un juge ou un autre officier-pr´esident ou aux autres parties dans sions in judicial proceedings; une instance judiciaire;

b) a` un juge ou un autre officier-pr´esident, pour produire quelque partie de (b) to a judge or other presiding officer to produce any part of this publication cette publication dans une instance judiciaire; ou in judicial proceedings; or c) a` quiconque, pour reproduire quelque partie de cette publication dans le cadre de d´elib´erations parlementaires. (c) to anyone to reproduce any part of this publication for the purposes of « Instance judiciaire » comprend une instance devant une cour, un tribunal ou parliamentary proceedings. une personne ayant l’autorit´e de d´ecider sur toute chose affectant les droits ou les responsabiliti´es d’une personne. “Judicial proceedings” include proceedings before any court, tribunal or per- Ni Carswell ni aucune des autres personnes ayant particip´e a` la r´ealisation et son having authority to decide any matter affecting a person’s legal rights or a` la distribution de la pr´esente publication ne fournissent quelque garantie liabilities. que ce soit relativement a` l’exactitude ou au caract`ere actuel de celle-ci. Il est entendu que la pr´esente publication est offerte sous la r´eserve expresse que ni Carswell and all persons involved in the preparation and sale of this publica- Carswell, ni le ou les auteurs de cette publication, ni aucune des autres per- tion disclaim any warranty as to accuracy or currency of the publication. This sonnes ayant particip´e a` son elaboration´ n’assument quelque responsabilit´e publication is provided on the understanding and basis that none of Carswell, que ce soit relativement a` l’exactitude ou au caract`ere actuel de son contenu the author/s or other persons involved in the creation of this publication shall ou au r´esultat de toute action prise sur la foi de l’information qu’elle be responsible for the accuracy or currency of the contents, or for the results renferme, ou ne peuvent etreˆ tenus responsables de toute erreur qui pourrait of any action taken on the basis of the information contained in this publica- s’y etreˆ gliss´ee ou de toute omission. tion, or for any errors or omissions contained herein. La participation d’une personne a` la pr´esente publication ne peut en aucun cas etreˆ consid´er´ee comme constituant la formulation, par celle-ci, d’un avis No one involved in this publication is attempting herein to render legal, ac- juridique ou comptable ou de tout autre avis professionnel. Si vous avez counting, or other professional advice. If legal advice or other expert assis- besoin d’un avis juridique ou d’un autre avis professionnel, vous devez tance is required, the services of a competent professional should be sought. retenir les services d’un avocat ou d’un autre professionnel. Les analyses The analysis contained herein should in no way be construed as being either comprises dans les pr´esentes ne doivent etreˆ interpr´et´ees d’aucune fa¸con official or unofficial policy of any governmental body. comme etant´ des politiques officielles ou non officielles de quelque organ- isme gouvernemental que ce soit.

8 The paper used in this publication meets the minimum requirements of 8 Le papier utilis´e dans cette publication satisfait aux exigences minimales American National Standard for Information Sciences — Permanence of Pa- de l’American National Standard for Information Sciences — Permanence of per for Printed Library Materials, ANSI Z39.48-1984. Paper for Printed Library Materials, ANSI Z39.48-1984.

ISSN 0835-3808 ISBN 978-0-7798-4647-4 Printed in Canada by Thomson Reuters

CARSWELL, A DIVISION OF THOMSON REUTERS CANADA LIMITED One Corporate Plaza Customer Relations 2075 Kennedy Road Toronto 1-416-609-3800 Toronto, Ontario Elsewhere in Canada/U.S. 1-800-387-5164 M1T 3V4 Fax 1-416-298-5082 www.carswell.com E-mail www.carswell.com/email Canadian Soc. of Imm. Consultants v. Canada (MCI) 175

[Indexed as: Canadian Society of Immigration Consultants v. Canada (Minister of Citizenship & Immigration)] The Canadian Society of Immigration Consultants, Applicant and The Minister of Citizenship and Immigration, Respondent Docket: IMM-5039-11 2011 FC 1435 J. Heard: October 6, 2011 Judgment: December 8, 2011 Administrative law –––– Review for lack or excess of jurisdiction — Ultra vires –––– C was corporation constituted under Canada Corporations Act to act as independent self-regulating body which operated at arm’s length from Gov- ernment — C has acted as sole regulatory body of immigration consultants — Concerns arose regarding regulatory scheme for immigration consultants — Call for submissions was published setting out selection factors for new regulator — As result, following were enacted: order fixing June 30, 2011 as day on which Chapter 8 of Statutes of Canada comes into Force (GIC order), Regulations Amending Immigration and Refugee Protection Regulations (2011 regulations), and Regulations Designating Body for Purposes of Paragraph 91(2)(c) of Immi- gration and Refugee Protection Act (IRPA) (Ministerial regulations) — C’s des- ignation as regulator was revoked by 2011 regulations, and new regulator (IC) was designated by Ministerial regulations — C submitted that enactments were ultra vires and exceeded regulation-making authority under s. 91 of IRPA on grounds of abuse of discretion, bad faith, and reliance upon irrelevant considera- tions — C brought application for judicial review — Application dismissed — Duty to act fairly and doctrine of legitimate expectations were not applicable; C erroneously assimilated revocation of its designation as if it were decision made by Government adversely affecting rights of individual who makes living — C did not act in any representative capacity but as designated regulator and mem- bers in good standing of C were deemed to be members of IC so long as fees were paid — Any regulatory monopoly granted to C was power exclusively de- rived from Governor in Council or Minister of Citizenship and Immigration; thus any monopoly can always be taken away in same manner — IC was chosen because it demonstrated that it had capacity to meet established organizational competencies that served as selection factors; maintenance of public confidence in immigration system was valid consideration — C argued that Minister failed to follow selection process in call for submissions and thus legitimately ex- pected by C — Selection Committee observed that C did not demonstrate how it 176 IMMIGRATION LAW REPORTER 3 Imm. L.R. (4th)

would address areas of expressed concern — Selection was legitimate policy choice based on delegated legislative authority when Ministerial regulations were enacted in June 2011. Immigration and citizenship –––– Constitutional issues — Legislative pow- ers –––– C was corporation constituted under Canada Corporations Act to act as independent self-regulating body which operated at arm’s length from Govern- ment — C has acted as sole regulatory body of immigration consultants — Con- cerns arose regarding regulatory scheme for immigration consultants — Call for submissions was published setting out selection factors for new regulator — As result, following were enacted: order fixing June 30, 2011 as day on which Chapter 8 of Statutes of Canada comes into Force (GIC order), Regulations Amending Immigration and Refugee Protection Regulations (2011 regulations), and Regulations Designating Body for Purposes of Paragraph 91(2)(c) of Immi- gration and Refugee Protection Act (IRPA) (Ministerial regulations) — C’s des- ignation as regulator was revoked by 2011 regulations, and new regulator (IC) was designated by Ministerial regulations — C submitted that enactments were ultra vires and exceeded regulation-making authority under s. 91 of IRPA on grounds of abuse of discretion, bad faith, and reliance upon irrelevant considera- tions — C brought application for judicial review — Application dismissed — Duty to act fairly and doctrine of legitimate expectations were not applicable; C erroneously assimilated revocation of its designation as if it were decision made by Government adversely affecting rights of individual who makes living — C did not act in any representative capacity but as designated regulator and mem- bers in good standing of C were deemed to be members of IC so long as fees were paid — Any regulatory monopoly granted to C was power exclusively de- rived from Governor in Council or Minister of Citizenship and Immigration; thus any monopoly can always be taken away in same manner — IC was chosen because it demonstrated that it had capacity to meet established organizational competencies that served as selection factors; maintenance of public confidence in immigration system was valid consideration — C argued that Minister failed to follow selection process in call for submissions and thus legitimately ex- pected by C — Selection Committee observed that C did not demonstrate how it would address areas of expressed concern — Selection was legitimate policy choice based on delegated legislative authority when Ministerial regulations were enacted in June 2011. Cases considered by Luc Martineau J.: Apotex Inc. v. Canada (Attorney General) (2000), 188 D.L.R. (4th) 145, 255 N.R. 319, 24 Admin. L.R. (3d) 279, 6 C.P.R. (4th) 165, 2000 CarswellNat 889, [2000] 4 F.C. 264, 180 F.T.R. 278, 2000 CarswellNat 3266, [2000] F.C.J. No. 634 (Fed. C.A.) — referred to Canadian Soc. of Imm. Consultants v. Canada (MCI) 177

Begg v. Canada (Minister of Agriculture) (2005), 341 N.R. 314, 261 D.L.R. (4th) 36, 2005 CAF 362, 2005 CarswellNat 5210, 2005 FCA 362, 2005 Car- swellNat 3560, [2005] F.C.J. No. 1819 (F.C.A.) — referred to Bristol-Myers Squibb Co. v. Canada (Attorney General) (2005), 2005 SCC 26, 2005 CarswellNat 1261, 2005 CarswellNat 1262, 253 D.L.R. (4th) 1, [2005] 1 S.C.R. 533, 39 C.P.R. (4th) 449, 334 N.R. 55, [2005] S.C.J. No. 26, EYB 2005-90541 (S.C.C.) — referred to British Columbia v. Imperial Tobacco Canada Ltd. (2005), 45 B.C.L.R. (4th) 1, [2005] 2 S.C.R. 473, 134 C.R.R. (2d) 46, 2005 SCC 49, 2005 CarswellBC 2207, 2005 CarswellBC 2208, 257 D.L.R. (4th) 193, [2006] 1 W.W.R. 201, 218 B.C.A.C. 1, 359 W.A.C. 1, 339 N.R. 129, 27 C.P.C. (6th) 13, EYB 2005-95296, [2005] S.C.J. No. 50, [2004] S.C.C.A. No. 302 (S.C.C.) — considered Brown v. British Columbia (Minister of Transportation & Highways) (1994), 164 N.R. 161, 112 D.L.R. (4th) 1, 1994 CarswellBC 1236, 1994 CarswellBC 128, [1994] 4 W.W.R. 194, 20 Admin. L.R. (2d) 1, 89 B.C.L.R. (2d) 1, 19 C.C.L.T. (2d) 268, [1994] 1 S.C.R. 420, 42 B.C.A.C. 1, 67 W.A.C. 1, 2 M.V.R. (3d) 43, [1994] S.C.J. No. 20, EYB 1994-67080 (S.C.C.) — re- ferred to C.U.P.E. v. Ontario (Minister of Labour) (2003), 2003 CarswellOnt 1803, 2003 SCC 29, 2003 CarswellOnt 1770, 2003 C.L.L.C. 220-040, [2003] 1 S.C.R. 539, (sub nom. Canadian Union of Public Employees v. Ontario (Minister of Labour)) 173 O.A.C. 38, (sub nom. Canadian Union of Public Employees v. Ontario (Minister of Labour)) 66 O.R. (3d) 735 (note), 226 D.L.R. (4th) 193, (sub nom. Canadian Union of Public Employees v. Ontario (Minister of Labour)) 304 N.R. 76, 50 Admin. L.R. (3d) 1, REJB 2003-41592, [2003] S.C.J. No. 28 (S.C.C.) — referred to Canada (Attorney General) v. Law Society (British Columbia) (1982), [1982] 2 S.C.R. 307, 37 B.C.L.R. 145, [1982] 5 W.W.R. 289, 19 B.L.R. 234, 43 N.R. 451, 137 D.L.R. (3d) 1, 66 C.P.R. (2d) 1, 1982 CarswellBC 133, 1982 Car- swellBC 743, [1982] S.C.J. No. 70 (S.C.C.) — considered Canada (Human Rights Commission) v. Canadian Liberty Net (1998), 157 D.L.R. (4th) 385, 1998 CarswellNat 388, 224 N.R. 241, 31 C.H.R.R. D/433, 22 C.P.C. (4th) 1, 50 C.R.R. (2d) 189, [1998] 1 S.C.R. 626, 147 F.T.R. 305 (note), 1998 CarswellNat 387, 6 Admin. L.R. (3d) 1, [1998] S.C.J. No. 31 (S.C.C.) — followed Canadian Assn. of Regulated Importers v. Canada (Attorney General) (1994), 17 Admin. L.R. (2d) 121, (sub nom. Assoc. canadienne des importateurs reglementes v. Canada) [1994] 2 F.C. 247, 1994 CarswellNat 840, 164 N.R. 342, 73 F.T.R. 80 (note), (sub nom. Assoc. canadienne des importateurs reglementes v. Canada) 1994 CarswellNat 1461 (Fed. C.A.) — referred to Canadian Council for Refugees v. R. (2008), 73 Imm. L.R. (3d) 159, 2008 FCA 229, 2008 CarswellNat 1995, 2008 CAF 229, 74 Admin. L.R. (4th) 79, 2008 178 IMMIGRATION LAW REPORTER 3 Imm. L.R. (4th)

CarswellNat 2197, (sub nom. Canadian Council for Refugees v. Canada) 385 N.R. 1, (sub nom. Canadian Council for Refugees v. Canada) [2009] 3 F.C.R. 136 (F.C.A.) — referred to Canadian Pacific Railway v. Vancouver (City) (2006), 18 M.P.L.R. (4th) 1, 262 D.L.R. (4th) 454, 221 B.C.A.C. 1, 364 W.A.C. 1, 2006 SCC 5, 2006 Car- swellBC 404, 2006 CarswellBC 405, 88 L.C.R. 161, 40 R.P.R. (4th) 159, 345 N.R. 140, [2006] 1 S.C.R. 227, [2006] S.C.J. No. 5 (S.C.C.) — considered Canadian Society of Immigration Consultants v. Canada (Minister of Citizenship & Immigration) (2011), 2011 CarswellNat 2791, 2011 FC 669, 100 Imm. L.R. (3d) 223, 2011 CF 669, 2011 CarswellNat 3926 (F.C.) — considered Canadian Wheat Board v. Canada (Attorney General) (2009), 2009 FCA 214, 2009 CarswellNat 1804, 2009 CAF 214, 392 N.R. 149, 2009 CarswellNat 4014, [2010] 3 F.C.R. 374 (F.C.A.) — referred to Centre hospitalier Mont-Sina¨ı c. Qu´ebec (Ministre de la Sant´e & des Services sociaux) (2001), (sub nom. Mount Sinai Hospital Center v. Quebec (Minister of Health & Social Services)) 200 D.L.R. (4th) 193, (sub nom. Mount Sinai Hospital Centre v. Quebec (Minister of Health & Social Services)) 271 N.R. 104, 2001 SCC 41, 2001 CarswellQue 1272, 2001 CarswellQue 1273, 36 Admin. L.R. (3d) 71, (sub nom. Mount Sinai Hospital Center v. Quebec (Minister of Health & Social Services)) [2001] 2 S.C.R. 281, [2001] S.C.J. No. 43, REJB 2001-24843 (S.C.C.) — considered de Guzman v. Canada (Minister of Citizenship & Immigration) (2005), 42 Ad- min. L.R. (4th) 234, 2005 FCA 436, 2005 CarswellNat 4381, 51 Imm. L.R. (3d) 17, 262 D.L.R. (4th) 13, 137 C.R.R. (2d) 20, [2006] 3 F.C.R. 655, 345 N.R. 73, 2005 CarswellNat 6009, 139 C.R.R. (2d) 376 (note), [2005] F.C.J. No. 2119 (F.C.A.) — considered Homex Realty & Development Co. v. Wyoming (Village) (1980), 1980 Carswell- Ont 508, 13 M.P.L.R. 234, 116 D.L.R. (3d) 1, 33 N.R. 475, [1980] 2 S.C.R. 1011, 1980 CarswellOnt 650, [1980] S.C.J. No. 109 (S.C.C.) — referred to International Assn. of Immigration Practitioners v. R. (2004), 2004 FC 630, 2004 CarswellNat 1249, [2004] F.C.J. No. 770 (F.C.) — followed Inuit Tapirisat of Canada v. Canada (Attorney General) (1980), 1980 Car- swellNat 633, [1980] 2 F.C.R. 735, [1980] 2 S.C.R. 735, 115 D.L.R. (3d) 1, 33 N.R. 304, 1980 CarswellNat 633F, [1980] S.C.J. No. 99 (S.C.C.) — re- ferred to Jafari v. Canada (Minister of Employment & Immigration) (1995), 125 D.L.R. (4th) 141, 30 Imm. L.R. (2d) 139, 1995 CarswellNat 670F, 1995 Car- swellNat 1111, 180 N.R. 330, [1995] 2 F.C. 595, 95 F.T.R. 159 (note) (Fed. C.A.) — referred to Knight v. Imperial Tobacco Canada Ltd. (2011), 2011 CarswellBC 1968, 2011 CarswellBC 1969, 2011 SCC 42, (sub nom. British Columbia v. Imperial Canadian Soc. of Imm. Consultants v. Canada (MCI) 179

Tobacco Canada Ltd.) 419 N.R. 1, 86 C.C.L.T. (3d) 1, (sub nom. British Columbia v. Imperial Tobacco Canada Ltd.) 335 D.L.R. (4th) 513, 21 B.C.L.R. (5th) 215, [2011] 11 W.W.R. 215, 25 Admin. L.R. (5th) 1, (sub nom. British Columbia) 308 B.C.A.C. 1, (sub nom. British Columbia v. Imperial Tobacco Canada Ltd.) 521 W.A.C. 1, [2011] S.C.J. No. 42, [2011] A.C.S. No. 42 (S.C.C.) — referred to Law Society (British Columbia) v. Mangat (2001), 157 B.C.A.C. 161, 256 W.A.C. 161, 16 Imm. L.R. (3d) 1, 205 D.L.R. (4th) 577, 2001 SCC 67, 2001 CarswellBC 2168, 2001 CarswellBC 2169, 276 N.R. 339, [2002] 2 W.W.R. 201, 96 B.C.L.R. (3d) 1, [2001] 3 S.C.R. 113, [2001] S.C.J. No. 66, REJB 2001-26158 (S.C.C.) — referred to Law Society of Upper Canada v. Canada (Minister of Citizenship & Immigra- tion) (2008), 295 D.L.R. (4th) 488, 383 N.R. 200, 2008 CAF 243, 2008 Car- swellNat 5012, [2009] 2 F.C.R. 466, 2008 CarswellNat 2487, 2008 FCA 243, 72 Imm. L.R. (3d) 26 (F.C.A.) — followed Mercier c. Canada (Service correctionnel) (2010), (sub nom. Mercier v. Correctional Service of Canada) 404 N.R. 275, 2010 FCA 167, 2010 Car- swellNat 1960, 2010 CarswellNat 1961, 2010 CAF 167, (sub nom. Mercier v. Canada (Correctional Service)) 320 D.L.R. (4th) 429 (F.C.A.) — re- ferred to Mooney v. Canadian Society of Immigration Consultants (2011), 2011 CF 496, 2011 CarswellNat 3248, 99 Imm. L.R. (3d) 198, 2011 FC 496, 2011 Car- swellNat 1545, 235 C.R.R. (2d) 41 (F.C.) — considered New Brunswick (Board of Management) v. Dunsmuir (2008), 372 N.R. 1, 69 Admin. L.R. (4th) 1, 69 Imm. L.R. (3d) 1, (sub nom. Dunsmuir v. New Brunswick) [2008] 1 S.C.R. 190, 844 A.P.R. 1, (sub nom. Dunsmuir v. New Brunswick) 2008 C.L.L.C. 220-020, D.T.E. 2008T-223, 329 N.B.R. (2d) 1, (sub nom. Dunsmuir v. New Brunswick) 170 L.A.C. (4th) 1, (sub nom. Dunsmuir v. New Brunswick) 291 D.L.R. (4th) 577, 2008 CarswellNB 124, 2008 CarswellNB 125, 2008 SCC 9, 64 C.C.E.L. (3d) 1, (sub nom. Dunsmuir v. New Brunswick) 95 L.C.R. 65, [2008] S.C.J. No. 9, [2008] A.C.S. No. 9 (S.C.C.) — followed Oberlander v. Canada (Attorney General) (2004), 241 D.L.R. (4th) 146, 320 N.R. 366, 2004 CAF 213, 37 Imm. L.R. (3d) 75, 2004 CarswellNat 3084, 2004 CarswellNat 1598, 2004 FCA 213, [2005] 1 F.C.R. 3, [2004] F.C.J. No. 920 (F.C.A.) — referred to Ocean Port Hotel Ltd. v. British Columbia (General Manager, Liquor Control & Licensing Branch) (2001), 2001 SCC 52, 2001 CarswellBC 1877, 2001 CarswellBC 1878, (sub nom. Ocean Port Hotel Ltd. v. Liquor Control & Licensing Branch (B.C.)) 155 B.C.A.C. 193, (sub nom. Ocean Port Hotel Ltd. v. Liquor Control & Licensing Branch (B.C.)) 254 W.A.C. 193, [2001] 10 W.W.R. 1, 34 Admin. L.R. (3d) 1, 204 D.L.R. (4th) 33, 274 N.R. 116, 93 180 IMMIGRATION LAW REPORTER 3 Imm. L.R. (4th)

B.C.L.R. (3d) 1, [2001] 2 S.C.R. 781, [2001] S.C.J. No. 17, REJB 2001- 25683 (S.C.C.) — referred to Old St. Boniface Residents Assn. Inc. v. Winnipeg (City) (1990), 2 M.P.L.R. (2d) 217, [1991] 2 W.W.R. 145, 75 D.L.R. (4th) 385, 116 N.R. 46, 69 Man. R. (2d) 134, [1990] 3 S.C.R. 1170, 1990 CarswellMan 383, 46 Admin. L.R. 161, 1990 CarswellMan 235, EYB 1990-67192, [1990] S.C.J. No. 137 (S.C.C.) — referred to Onuschak v. Canadian Society of Immigration Consultants (2009), 2009 Car- swellNat 5491, 2009 CarswellNat 4214, 2009 FC 1135, 357 F.T.R. 22 (Eng.), 86 Imm. L.R. (3d) 78, 2009 CF 1135, 3 Admin. L.R. (5th) 214 (F.C.) — followed Pharmascience inc. c. Binet (2006), (sub nom. Binet v. Pharmascience Inc.) 353 N.R. 343, 2006 CarswellQue 8953, 2006 CarswellQue 8954, 2006 SCC 48, (sub nom. Binet v. Pharmascience Inc.) 273 D.L.R. (4th) 193, [2006] 2 S.C.R. 513, [2006] S.C.J. No. 48 (S.C.C.) — considered Reference re Canada Assistance Plan (Canada) (1991), (sub nom. Reference re Canada Assistance Plan (British Columbia)) 83 D.L.R. (4th) 297, 1991 Car- swellBC 920, 1991 CarswellBC 168, 58 B.C.L.R. (2d) 1, 1 Admin. L.R. (2d) 1, (sub nom. Reference re Constitutional Question Act (British Columbia)) 1 B.C.A.C. 241, 1 W.A.C. 241, (sub nom. Reference re Constitutional Question Act (British Columbia)) 127 N.R. 161, [1991] 6 W.W.R. 1, (sub nom. Reference re Canada Assistance Plan (British Columbia)) [1991] 2 S.C.R. 525, EYB 1991-67051, [1991] S.C.J. No. 60 (S.C.C.) — referred to Reference re Language Rights Under s. 23 of Manitoba Act, 1870 & s. 133 of Constitution Act, 1867 (1992), 1992 CarswellMan 219, [1992] 2 W.W.R. 385, [1992] 1 S.C.R. 212, (sub nom. Manitoba Language Rights Reference (No. 2)) 76 Man. R. (2d) 124, (sub nom. Manitoba Language Rights Reference (No. 2)) 10 W.A.C. 124, 88 D.L.R. (4th) 385, (sub nom. Manitoba Language Rights Reference (No. 2)) 133 N.R. 88, 1992 CarswellMan 96 (S.C.C.) — followed Reference re Public Service Employee Relations Act (Alberta) (1987), (sub nom. A.U.P.E. v. Alberta (Attorney General)) 28 C.R.R. 305, [1987] D.L.Q. 225, 1987 CarswellAlta 580, 1987 CarswellAlta 705, 87 C.L.L.C. 14,021, [1987] 1 S.C.R. 313, 38 D.L.R. (4th) 161, (sub nom. Reference re Compulsory Ar- bitration) 74 N.R. 99, [1987] 3 W.W.R. 577, 51 Alta. L.R. (2d) 97, 78 A.R. 1, EYB 1987-66907, [1987] S.C.J. No. 10 (S.C.C.) — followed Reference re s. 16 of the Criminal Law Amendment Act, 1968-69 (Canada) (1970), [1970] S.C.R. 777, 10 D.L.R. (3d) 699, [1970] 3 C.C.C. 320, 12 C.R.N.S. 28, 74 W.W.R. 167, 1970 CarswellNS 178, 1970 CarswellNS 52 (S.C.C.) — referred to Reference re Secession of Quebec (1998), 228 N.R. 203, 1998 CarswellNat 1300, 161 D.L.R. (4th) 385, 1998 CarswellNat 1299, 55 C.R.R. (2d) 1, [1998] 2 S.C.R. 217, [1998] S.C.J. No. 61 (S.C.C.) — followed Canadian Soc. of Imm. Consultants v. Canada (MCI) 181

Roncarelli v. Duplessis (1959), 1959 CarswellQue 37, [1959] S.C.R. 121, 16 D.L.R. (2d) 689, [1959] S.C.J. No. 1 (S.C.C.) — referred to Saputo Inc. v. Canada (Attorney General) (2009), 353 F.T.R. 67 (Eng.), 2009 FC 1016, 2009 CarswellNat 3068, 2009 CF 1016, 2009 CarswellNat 5259, [2010] 4 F.C.R. 274 (F.C.) — referred to Saputo Inc. v. Canada (Attorney General) (2011), 414 N.R. 45, 2011 CAF 69, 2011 CarswellNat 2483, 2011 CarswellNat 431, 2011 FCA 69 (F.C.A.) — referred to Syndicat national des employ´es de la commission scolaire r´egionale de l’Outaouais v. U.E.S., local 298 (1988), 95 N.R. 161, 89 C.L.L.C. 14,045, 24 Q.A.C. 244, (sub nom. Union des employ´es de service, local 298 v. Bibeault) [1988] 2 S.C.R. 1048, 1988 CarswellQue 125, 1988 CarswellQue 148, 35 Admin. L.R. 153, EYB 1988-67863, [1988] S.C.J. No. 101 (S.C.C.) — followed Thorne’s Hardware Ltd. v. R. (1983), 1983 CarswellNat 530F, [1983] 1 S.C.R. 106, 143 D.L.R. (3d) 577, 46 N.R. 91, 1983 CarswellNat 530, [1983] S.C.J. No. 10 (S.C.C.) — referred to United Taxi Drivers’ Fellowship of Southern Alberta v. Calgary (City) (2004), 46 M.P.L.R. (3d) 1, 236 D.L.R. (4th) 385, [2004] 7 W.W.R. 603, 346 A.R. 4, 320 W.A.C. 4, 318 N.R. 170, 18 R.P.R. (4th) 1, [2004] 1 S.C.R. 485, 2004 CarswellAlta 355, 2004 CarswellAlta 356, 2004 SCC 19, 26 Alta. L.R. (4th) 1, 12 Admin. L.R. (4th) 1, 50 M.V.R. (4th) 1, [2004] S.C.J. No. 19, REJB 2004-55539 (S.C.C.) — referred to Statutes considered: Bankruptcy and Insolvency Act, R.S.C. 1985, c. B-3 Generally — referred to Canada Corporations Act, R.S.C. 1970, c. C-32 Generally — referred to Pt. II — referred to Canadian Charter of Rights and Freedoms, Part I of the Constitution Act, 1982, being Schedule B to the Canada Act 1982 (U.K.), 1982, c. 11 Generally — referred to Constitution Act, 1867, (U.K.), 30 & 31 Vict., c. 3, reprinted R.S.C. 1985, App. II, No. 5 Generally — referred to s. 95 — referred to s. 96 — referred to Federal Courts Act, R.S.C. 1985, c. F-7 s. 18 — referred to s. 28 — referred to Immigration Act, R.S.C. 1985, c. I-2 s. 114(1)(v) — referred to 182 IMMIGRATION LAW REPORTER 3 Imm. L.R. (4th)

Immigration and Refugee Protection Act, S.C. 2001, c. 27 Generally — referred to s. 4 — considered s. 5 — referred to s. 5(1) — referred to s. 14 — referred to s. 91 — referred to s. 91 [rep. & sub. 2011, c. 8, s. 1] — referred to s. 91(1) [en. 2011, c. 8, s. 1] — considered s. 91(2) [en. 2011, c. 8, s. 1] — considered s. 91(3) [en. 2011, c. 8, s. 1] — considered s. 91(4) [en. 2011, c. 8, s. 1] — referred to s. 91(5) [en. 2011, c. 8, s. 1] — considered s. 91(5.1) [en. 2011, c. 8, s. 1] — considered s. 91(6) [en. 2011, c. 8, s. 1] — considered s. 91(7) [en. 2011, c. 8, s. 1] — considered s. 91(7.1) [en. 2011, c. 8, s. 1] — referred to Immigration and Refugee Protection Act, Act to amend the, S.C. 2011, c. 8 Generally — referred to s. 6 — referred to s. 7 — referred to Immigration au Qu´ebec, Loi sur l’, L.R.Q., c. I-0.2 en g´en´eral — referred to art. 3.3(k)-3.3(q) — referred to Interpretation Act, R.S.C. 1985, c. I-21 s. 7 — considered Statutory Instruments Act, R.S.C. 1985, c. S-22 Generally — referred to s. 2(1) “regulation” — considered s. 2(1) “r`eglement” — considered s. 2(1) “statutory instrument” (a)(i) — considered s. 2(1) “statutory instrument” (a)(ii) — considered s. 2(1) “texte r´eglementaire” (a)(i) — considered s. 2(1) “texte r´eglementaire” (a)(ii) — considered s. 6(b) — considered s. 9 — referred to s. 9(1)(a) — referred to Rules considered: Federal Courts Rules, SOR/98-106 R. 466-472 — referred to Canadian Soc. of Imm. Consultants v. Canada (MCI) Luc Martineau J. 183

Regulations considered: Immigration and Refugee Protection Act, S.C. 2001, c. 27 Immigration and Refugee Protection Regulations, SOR/2002-227 Generally — referred to s. 2 “authorized representative” [en. SOR/2004-59] — referred to s. 10(2)(c.1) [en. SOR/2004-59] — referred to s. 10(2)(c.2) [en. SOR/2004-59] — referred to s. 13.1 [en. SOR/2004-59] — referred to s. 13.1(2) [en. SOR/2004-59] — referred to Immigration and Refugee Protection Regulations, Regulations Amending the, SOR/2011-129 Generally — referred to Regulations Designating a Body for the Purposes of Paragraph 91(2)(c) of the Immigration and Refugee Protection Act, SOR/2011-142 Generally — referred to Immigration au Qu´ebec, Loi sur l’, L.R.Q., c. I-0.2 R`eglement sur les consultants en immigration, R.R.Q., c I-0.2, r.0.1 en g´en´eral — referred to art. 4 — referred to Statutory Instruments Act, R.S.C. 1985, c. S-22 Statutory Instruments Regulations, C.R.C. 1978, c. 1509 s. 11(3)(g) — considered

APPLICATION by former regulatory body of immigration consultants, chal- lenging Minister of Citizenship and Immigration and Governor in Council’s en- actments revoking its designation.

John Callaghan, Benjamin Na, Guy Regimbald, for Applicant Marianne Zoric, Catherine Vasilaros, Neal Samson, for Respondent

Luc Martineau J.:

1 This application invites the Court to address the scope of judicial re- view of regulations dealing with immigration consultants in light of such fundamental principles and Canadian values, as the rule of law and the separation of powers.

I. Introduction 2 The Canadian Society of Immigration Consultants (CSIC), the appli- cant in this judicial review, is a corporation without share capital consti- tuted on October 8, 2003 under Part II of the Canada Corporations Act, 184 IMMIGRATION LAW REPORTER 3 Imm. L.R. (4th)

RSC 1970, c C-32 to fulfill the role of an independent self-regulating body and which operates at arm’s length from the Government. 3 The letters patent of the applicant provide that it shall regulate immi- gration consultants in the public interest and in so doing shall establish a code of conduct, a complaint and disciplinary procedure, an educational program, and a compensation fund with respect to acts and omissions of its members. 4 From April 13, 2004 to June 30, 2011, the applicant has acted as the sole regulatory body of immigration consultants in Canada whose mem- bers are legally authorized to advise, consult with, and represent individ- uals involved in proceedings under the Immigration and Refugee Protec- tion Act, SC 2001, c 27 (the Act) and its regulations: sections 2 and 13.1 of the Immigration and Refugee Protection Regulations, SOR/2002-227 (IRPR), as modified by SOR/2004-59 (the 2004 Regulations). 5 The applicant challenges the legality of the following enactments: (a) The Order Fixing June 30, 2011 as the Day on which Chapter 8 of the Statutes of Canada Comes into Force (SI/2011-731) (GIC Order); (b) The Regulations Amending the Immigration and Refugee Protec- tion Regulations (SOR/2011-129) (2011 Regulations); and (c) The Regulations Designating a Body for the Purposes of Para- graph 91(2)(c) of the Immigration and Refugee Protection Act (SOR/2011-142) (Ministerial Regulations). 6 As of June 30, 2011, concurrently with the coming into force (the GIC Order) of An Act to Amend the Immigration and Refugee Protection Act, SC 2011, c 27, previously known as Bill C-35, the applicant’s desig- nation as the regulator of the immigration consultants is revoked (the 2011 Regulations) and the Immigration Consultants of Canada Regula- tory Council (ICCRC) is designated as the new regulator (the Ministerial Regulations). 7 Although the applicant treats the impugned enactments as a single “decision”, formally speaking, the GIC Order and the 2011 Regulations are made by the Governor in Council (Cabinet), while the Ministerial Regulations are made by the Minister of Citizenship and Immigration (the Minister), the present respondent. More particularly, the impugned enactments are respectively made under the purported authority of sec- tion 7 of Bill C-35 (the GIC Order); subsection 5(1), section 14 and for- mer section 91 of the Act (the 2011 Regulations); and new subsections Canadian Soc. of Imm. Consultants v. Canada (MCI) Luc Martineau J. 185

91(5) and (7) of the Act, as amended by section 1 of Bill C-35 (the Min- isterial Regulations). 8 Both the GIC Order and the 2011 Regulations were published in Part II of the Canada Gazette on July 6, 2011. The Ministerial Regulations along with the Regulatory Impact Analysis Statement (RIAS) were pub- lished in Part II of the Canada Gazette on July 20, 2011 (the July RIAS). There was no prepublication of any of the impugned enactments (SI/2011-731, SOR/2011-129 and SOR/2011-142). That said, on March 19, 2011, regulatory amendments to the IRPR that would have substan- tially the same effect as the 2011 Regulations and the Ministerial Regula- tions were pre-published in Part I of the Canada Gazette along with a Regulatory Impact Analysis Statement (the March RIAS). 9 The applicant submits that the enactments revoking the CSIC’s desig- nation (the 2011 Regulations) and designating the ICCRC as the new regulator (the Ministerial Regulations) are ultra vires and exceed the reg- ulation-making authority under (former or new) section 91 of the Act on the grounds of abuse of discretion, bad faith, and reliance upon irrelevant considerations. The applicant also submits that the making of both the 2011 Regulations and the Ministerial Regulations is contrary to the appli- cant’s legitimate expectations and right to be heard, while the conduct by the Minister and his staff at Citizenship and Immigration Canada (CIC) raises a reasonable apprehension of bias. Finally, the enactment of the GIC Order violates the procedural requirements of section 9 of the Statu- tory Instruments Act, RSC 1985, c S-22 (the SIA), and the Ministerial Regulations are otherwise invalid in law because they were made prior to the coming into force of Bill C-35. 10 To the contrary, the respondent submits that the impugned enact- ments are authorized by Parliament and it is not the function of the Court to examine the reasonableness of regulatory enactments or to criticise policy choices made by Parliament or the Government. In any event, there is no proof of bad faith, improper motive or actual bias. Moreover, rules of procedural fairness do not apply to legislation-making and whatever participatory rights the applicant might have had, they have been amply satisfied. Finally, all procedural requirements found in the SIA were followed and section 7 of the Interpretation Act, RSC 1985, c I-21 permits the making of regulations prior to the coming into force of legislation. Thus, the impugned enactments are valid in law. 11 Having considered the totality of the evidence, the applicable law and relevant case law, the present application must fail. The Court finds that 186 IMMIGRATION LAW REPORTER 3 Imm. L.R. (4th)

the impugned enactments are authorized by statute and validly came into force on June 30, 2011. In principle, regulations or policy decisions are not reviewable, except in cases of excess of jurisdiction or failure to comply with legislative or regulatory requirements. As far as any duty to consult is concerned, it has been satisfied in this case. The process of selecting a new regulatory body, in which the applicant was allowed to participate, was fair and transparent. This is not “an egregious case” where the intervention of the Court is warranted to uphold the rule of law.

II. Factual and Contextual Background 12 The present application is somewhat a continuation of the litigation that commenced in spring 2011 when the Government announced its in- tention to remove the reference to the applicant from the definition of “authorized representative” in section 2 of the IRPR, as modified by the 2004 Regulations, and replace it with the ICCRC. But before examining the spring and summer of 2011 events, it is necessary to go back to the early 2000s when it was decided to federally regulate the occupation of immigration consultant.

The 2004 Regulations 13 Self-governing professions have a long history in Canada — legal and medical professions were already established in the pre-Confedera- tion era — but until the turn of the century, the idea that immigration consultants constituted a group of professionals who should be legally allowed to compete with members of the legal profession and to regulate themselves in the best interests of the public had not yet emerged. 14 The background leading to the creation of a self-regulatory body gov- erning the activities of immigration consultants in Canada and the mak- ing of the 2004 Regulations is largely uncontested and supported by the evidence filed, and by relevant case law: International Assn. of Immigration Practitioners v. R., 2004 FC 630 (F.C.) at paras 3-10; Law Society of Upper Canada v. Canada (Minister of Citizenship & Immigra- tion), 2008 FCA 243 (F.C.A.) at paras 4-35 (Law Society of Upper Can- ada); and Onuschak v. Canadian Society of Immigration Consultants, 2009 FC 1135 (F.C.) at paras 11-19 (Onuschak). 15 In October 2002, the Minister appointed a committee of experts to advise him on the regulation of immigration consultants (the Advisory Committee). Following their recommendations made in May 2003, the Canadian Soc. of Imm. Consultants v. Canada (MCI) Luc Martineau J. 187

Minister accepted that a self-regulatory body be created since this re- quired no legislative changes. 16 Although there were already two existing associations, the Minister (or Cabinet) preferred the creation of a new body as the regulator of the immigration consultants. Apparently, the Association of Immigration Counsel of Canada (AICC) and the Organization of Professional Immi- gration Consultants (OPIC) had not been able to effectively enforce membership or high professional standards. Moreover, they both sup- ported the creation of the CSIC as the new regulator. 17 The Minister rejected the Advisory Committee’s recommendation that the new regulatory body be constituted by the Government and that it be composed of a board of directors composed of CIC representatives, immigration consultants, and members of the public. Instead, the regula- tory body would simply be a non-share capital corporation under the Canada Corporations Act, RSC 1970, c C-32, and the directors would be chosen by the members of the corporation. That said, financial support (by way of contribution agreements) and some external guidance would be provided by CIC in the setting up of the corporation during its early years of operation. 18 Self-government of the new regulatory body had two essential as- pects — the authority to licence and the ability to discipline. However, concerns were expressed that this may not cure the problem of “phan- tom” or “ghost” consultants. Because its jurisdiction only extended to members, this gray area of practice could not be effectively regulated by the newly created self-regulated body. Be that as it may, CIC promised to “closely monitor the situation”, but over time, this proved to be insuffi- cient as will be explained below. 19 The Government was also cognisant that a subsequent board of direc- tors of the newly created regulatory body could modify the code of con- duct and by-laws so as to reduce their professional standards thereby im- pacting consumers’ protection. Indeed, the Government vowed to stakeholders that should the CSIC fail to fulfil its central task of protect- ing consumers and maintaining professional standards, the Government would take action to remove its recognition. 20 In March 2005, one year after the applicant was designated as the regulator of immigration consultants, governmental authorities were sup- portive of the steps taken by the applicant: “Overall, CSIC has been op- erating with success and is meeting the Canadian government’s objective of protecting vulnerable people involved in the immigration process.” 188 IMMIGRATION LAW REPORTER 3 Imm. L.R. (4th)

21 Over time, however, there was a gradual erosion of confidence from part of the applicant’s membership, the public and the Government, and this whether or not partisan views may have also been at work, as sug- gested by the applicant. Apparently, there were external pressures in 2010 to have Mr. John Ryan, appointed years before, removed as Chief Executive Officer of the applicant. There were also pressures for the re- moval of Mr. Imran Qayyum from the Board of directors of the applicant and the Canadian Migration Institute (CMI), a wholly owned subsidiary of CSIC. 22 Be that as it may, as early as 2007, a Toronto Star investigation sug- gested that the regulatory scheme for immigration consultants continued to fail the public, and the Canadian Bar Association (CBA), unaware at that time of any disciplinary hearings against CSIC members, had ex- pressed similar concerns to the Minister. Indeed, the CBA was encourag- ing “the government to conduct a broader assessment of whether CSIC is meeting its mandate for the regulation of consultants, particularly given the persistent allegations of fiscal mismanagement made by past direc- tors of CSIC’s own Board”.

Standing Committee inquiry and recommendations 23 In April 2008, responding to the complaints and discontent from the public and from within the profession regarding unacceptable practices by immigration consultants, the Parliamentary Standing Committee on Citizenship and Immigration (the Standing Committee) undertook to study the issues in the field and to recommend measures to properly reg- ulate the profession. The Standing Committee did not conduct a formal investigation of the complaints made against the CSIC, whose represent- atives were nevertheless offered the opportunity to testify and comment on recommendations made afterwards. This was entirely within the pre- rogative of a Parliamentary Standing Committee. 24 In June 2008, the Standing Committee issued its report, entitled “Regulating Immigration Consultants”. It notably recommended that the Government introduce stand-alone legislation to Parliament to re-estab- lish the CSIC as a non-share capital corporation, to assist in re-establish- ing the new regulator, and to remain involved in its affairs until it is fully functioning. In its report, the Standing Committee noted that a number of immigration consultants were dissatisfied because CSIC’s membership fees were too high, it had failed to develop an industry plan, there was a Canadian Soc. of Imm. Consultants v. Canada (MCI) Luc Martineau J. 189

lack of transparency and accountability, and compensation and spending were extravagant. 25 While the Standing Committee did not make any specific finding of fact (to which the Minister’s representative admitted in this proceeding), it generally identified a number of shortcomings that should nevertheless be addressed by Parliament: These grievances stem from various issues, and no doubt many arise because CSIC is a relatively new organization struggling to strike the right balance to regulate previously unregulated professionals. How- ever, the Committee believes that problems at CSIC are attributable to more than just growing pains. Fundamentally, the Society is not being given the tools it needs to succeed as a regulator. As a feder- ally-incorporated body, CSIC has no power to sanction immigration consultants who are not members of the Society, and it cannot seek judicial enforcement of the disciplinary consequences it imposes on those who are members. Further, because CSIC’s jurisdiction is not governed by statute, there is no possibility for dissatisfied members and others to influence the Society’s internal functioning though [sic] judicial review. In the view of the Committee, these shortcomings should be addressed by new legislation. 26 The Standing Committee’s recommendation that the CSIC be “re-es- tablished” under stand-alone legislation was however not carried out by the Government, who would instead decide two years later to introduce Bill C-35 to Parliament as explained below.

Ministerial Response 27 Before proposing to Parliament legislative amendments to the Act, different options were considered by CIC and the Minister. 28 In 2009, Les Linklater, Director General of CIC Immigration Branch (now Assistant Deputy Minister, Strategic and Program Policy of CIC) retained a consulting group, Sussex Circle to, inter alia, conduct a review and provide “an analysis and assessment of the threshold required to con- clusively determine when the level of governance in a not for profit or- ganization has deteriorated to a point that the mandate of the board of directors could be revoked by the government with minimal legal risk”. 29 Sussex Circle reviewed CSIC’s governance and accountability ar- rangements. They found same to be inadequate in important aspects and proposed a number of options ranging from doing nothing and winding up the CSIC, notably through an amendment to the IRPR that would name another body to replace CSIC (the regulatory option). Other op- 190 IMMIGRATION LAW REPORTER 3 Imm. L.R. (4th)

tions would be to amend the Act or the regulations in order to give the power to appoint “public interest directors” and compel the CSIC (or an- other designated body) to produce information as requested by the Min- ister for consideration and approval, or to set out prescriptive governance and accountability requirements in return for the retaining (or the grant- ing) of monopoly in this area. 30 The regulatory option had some attractive features (notably because it did not require legislative amendments), but it was apparently not the option favoured by Sussex Circle because of its high transitional costs. Sussex Circle thought that a minimally regulatory approach was highly dependent on the cooperation of the CSIC, while a more comprehensive and prescriptive approach could be a fall back position. However, in the long term, it would be far preferable to make just one set of legislative changes to deal with the governance and accountability issues identified in its report. 31 The self-regulatory model chosen in 2004 did not prevent (and still does not prevent in 2011) immigration consultants in Canada and else- where from belonging to other professional associations. At the epoch CSIC became the regulator of immigration consultants, the Canadian As- sociation of Professional Immigration Consultants (CAPIC) was created by the amalgamation of two aforementioned immigration industry orga- nizations, the OPIC and the AICC, who had previously supported the establishment of the CSIC’s self-regulatory body. 32 The CAPIC is a voluntary immigration practitioner association that, among other things, lobbies and advocates on issues concerning immi- gration practitioners. The relationship between the CAPIC and the CSIC have been the cause of much friction between the two organizations and have been particularly strained since 2007 when Mr. Philip Mooney be- came president of the CAPIC as explained in the affidavit of Keith Frank and judicially noted by the Court in Mooney v. Canadian Society of Immigration Consultants, 2011 FC 496 (F.C.) (Mooney). The evidence on record clearly establishes that the CAPIC, notably Mr. Mooney him- self, campaigned against the CSIC and actively advocated for its replacement. 33 Apparently, CAPIC directors, although not registered lobbyists, met in 2008 and 2009 with Mr. Les Linklater — then Director General of CIC Immigration Branch — and other members of the Minister’s staff to lobby for the replacement of the CSIC or its Board of Directors. In this Canadian Soc. of Imm. Consultants v. Canada (MCI) Luc Martineau J. 191

respect, these unnamed CAPIC directors allegedly acted in an “advisory capacity to the Minister in “offering alternatives” to the CSIC. 34 Indeed, some immigration consultants heard in 2008 by the Standing Committee were directors of the CAPIC (or even “ghost consultants” as alleged by the applicant). Be that as it may, in Mooney, above, at para 113, this Court noted in 2011 that “[t]he Standing Committee Report and its principal recommendations are obviously a legitimate and thoughtful attempt to suggest ways in which CSIC could; and should, be reformed so that it might better fulfill its mandate and governing principles”.

Bill C-35 35 On June 8, 2010, Bill C-35, referred to by the Government as the Cracking Down on Crooked Consultants Act, was introduced to the House of Commons by Immigration and Multiculturalism Minister Jason Kenney. 36 In the news release and speaking notes of the Minister, one can read: While most immigration consultants working in Canada are legiti- mate and ethical, it is clear that immigration fraud remains a wide- spread threat to the integrity of Canada’s immigration system, said Minister Kenney. The Cracking Down on Crooked Consultants Act will better protect prospective immigrants from crooked consultants and help safeguard our immigration system against fraud and abuse. [...] The proposed legislation implements unanimous recommendations of the House of Commons Standing Committee on Immigration which were arrived at following extensive consultations.... 37 In passing, the applicant contends that in a television interview on June 12, 2010, the Minister misstated the recommendations of the Parlia- mentary Standing Committee when declaring that “there have been a lot of concerns expressed, including the Parliamentary Standing Committee on immigration, unanimously said the government should set up a new regulatory body”. Also, when questioned about Bill C-35 in a CPAC in- terview, on June 8, 2010, the Minister’s answer implied his objective to have “it done by the end of 2011.” 38 As will be explained below, the Court has found that the allegations of bias against the Minister are not determinative as far as the legality of the impugned enactments is concerned. The Court accepts the respon- dent’s submission that the Minister’s statements or comments have been taken out of context. In the CPAC interview, Minister Kenney was ap- 192 IMMIGRATION LAW REPORTER 3 Imm. L.R. (4th)

parently referring to the coming into effect of Bill C-35 that he wished was done by the end of 2011, and not the designation of a new regulatory body. 39 That said, despite the Minister’s statements, it is apparent that the Government chose not to follow the Standing Committee’s recommenda- tion that “the Government of Canada introduce stand-alone legislation to re-establish the Canadian Society of Immigration Consultants as a non- share capital corporation” and that “[s]uch an “Immigration Consultants Society Act” should provide for the same types of matters covered by founding statutes of provincial law societies, including, but not limited to: functions of the corporation, member licensing and conduct, profes- sional competence, prohibitions and offences, complaints resolution, compensation fund and by-laws”. 40 In effect, once adopted by Parliament and proclaimed in force, Bill C- 35 would significantly amend the manner of regulating third parties in immigration processes. Among other things Bill C-35: • Creates a new offence by extending the prohibition against repre- senting or advising persons for consideration — or offering to do so — to all stages in connection with a proceeding or application under the Act, including before a proceeding has been commenced or an application has been made, and provides for penalties in case of contravention; • Exempts from the prohibition: • Members of a provincial law society or notaries of the Chambre des notaires du Qu´ebec, and students-at-law act- ing under their supervision, • Any other members of a provincial law society or the Chambre des notaires du Qu´ebec, including a paralegal, • Members of a body designated by the Minister, and • Entities, and persons acting on behalf of the entities, acting in accordance with an agreement or arrangement with Her Majesty in right of Canada; • Extends the time for instituting certain proceedings by way of summary conviction from six months to 10 years; • Gives the Minister the power to make transitional regulations in relation to the designation or revocation by the Minister of a body; Canadian Soc. of Imm. Consultants v. Canada (MCI) Luc Martineau J. 193

• Provides for oversight by the Minister of a designated body through regulations made by the Governor in Council requiring the body to provide information to allow the Minister to determine whether it governs its members in the public interest; and, • Facilitates information sharing with regulatory bodies regarding the professional and ethical conduct of their members. 41 On September 23, 2010, Bill C-35 received second reading at the House of Commons and was referred to the Standing Committee. The latter presented its report on November 24, 2010, with a concurrence on December 6, 2010. It received third reading on December 7, 2010. 42 The same day, at the Senate level, Bill C-35 received first reading. It received second reading and was referred to the Standing Senate Com- mittee on Social Affairs, Science and Technology on March 1, 2011. The latter presented its report (with observations) on March 10, 2011. Finally, it received third reading on March 21, 2011. 43 Bill C-35 received Royal Assent on March 23, 2011.

Public selection process 44 On June 8, 2010, concurrently with the tabling at the House of Com- mons of Bill C-35, the Minister announced that it was also taking imme- diate steps to address “a lack of public confidence in the regulation of immigration consultants” and that a Notice of intent would be published announcing CIC’s intention to “launch a transparent public selection pro- cess to identify a governing body for recognition as the regulator of im- migration consultants, under current authority”. 45 Explaining the decision to launch a public selection process, Minister Jason Kenney stated: [...] According to the [House of Commons Standing Committee], complaints were heard from a number of consultants across the coun- try, many of whom have expressed great dissatisfaction with the way that the Canadian Society of Immigration Consultants, or CSIC, is currently governed. That’s why I’m taking immediate steps to ad- dress this problem, a problem that poses a significant threat to the immigration system and has created a lack of public confidence in the regulation of consultants. [...] The Notice of intent will request comments from the public on the proposed selection process. A transparent selection process will then 194 IMMIGRATION LAW REPORTER 3 Imm. L.R. (4th)

identify the body best able to effectively regulate consultants in sup- port of Canada’s public confidence in the immigration system. [...] The regulatory body must regulate effectively and must be held ac- countable for ensuring their membership provides services in a pro- fessional and ethical manner and that real sanctions are taken if their members do otherwise. 46 Effectively, on June 12, 2010, a Notice of intent was published in Part I of the Canada Gazette requesting comments from the public on its proposal to establish a public selection process with the objective of identifying a governing body for recognition as the regulator of immigra- tion consultants. More particularly, such “a competitive public selection will be pursued in order to identify the entity best able to demonstrate capacity to effectively regulate immigration consultants. Selection fac- tors will be established to ensure that the entity identified for recognition as the regulator of immigration consultants has the capacity to effectively regulate.” 47 According to the applicant, prior to the launch of the public selection process, Mr. Linklater allegedly requested Mr. Mooney — who later be- came the President and CEO of the ICCRC — to provide a list of 19-20 individuals who could take over the CSIC in its regulatory functions. In this respect, the Court finds the evidence on record inconclusive and fur- ther notes that there is no credible evidence allowing the Court to con- clude on a balance of probabilities that the public selection process was not fair and transparent. 48 Further to the Notice of intent published on June 12, 2010, after con- sidering comments received by the public, selection factors were devel- oped “to ensure that any entity serving as the regulator of immigration consultants has or will have the capacity to support Canada’s immediate and long term immigration objectives as well as maintain public confi- dence in the immigration system”. As it appears from the Government Notice published in Part I of the Canada Gazette on August 28, 2010 (the Call for Submissions), five selection factors were identified by CIC — competence, integrity, accountability, viability and good govern- ance — however, there could be “other relevant factors” that the Selec- tion Committee or the Minister may want to consider. 49 In the Call for Submissions, the Minister invited interested candidate entities to make submissions which “set out, in detail, how they respond to the selection factors”, but this “does not obligate the Minister, the De- Canadian Soc. of Imm. Consultants v. Canada (MCI) Luc Martineau J. 195

partment of Citizenship and Immigration or the Government of Canada in any way, or to take any action”. That said, the Call for Submissions indicates that “[a]n agreement or arrangement may be entered into be- tween the successful entity and the Government of Canada”. The dead- line for submissions was December 29, 2010. 50 To that effect, a Selection Committee (comprised of four external ex- perts and three senior public servants) was charged with examining the submissions received in response to the Call for Submissions and making recommendations to the Minister after having considered the submis- sions in light of the selection factors and “other relevant factors”. 51 In its final report dated September 24, 2010, Mr. John Scratch, an external consultant whose services were retained in spring 2010 by CIC, reiterated what he had already written in his interim report of July 2010, that the selection process of the regulator chosen by the Minister “must be open, transparent and competitive and must be seen to be so”. In her cross-examination, the Minister’s representative confirmed that the cho- sen selection process would have all those characteristics. Moreover, the report prepared by the external consultant “was a policy tool for the Min- ister to make a decision on who he was going to recommend”. It must be remembered that the selection process undertaken in the summer of 2010 was under the provisions of the Act, as they read at the time, and which conferred the authority to maintain or change the regulator of immigra- tion consultants to the Governor in Council (Cabinet). 52 Apart from the fact that the chosen organization must have, among other things, a code of conduct, a complaint and discipline mechanism, liability insurance, a compensation fund, bilingual services to members and the public, continuing education requirements and programs for members, Mr. Scratch notes that “[m]any of the problems identified with the current regulator are governance issues — democracy, accountability and transparency. Therefore, applicants should be required to demon- strate that they are capable of establishing an organization that will ad- dress these issues and that will provide for effective control of the Board of directors by the membership of the organization”. 53 In its final report, Mr. Scratch also found it difficult to provide spe- cific advice on an implementation plan because it was unclear — the se- lection process still not completed — what the issues would be until a decision had been made on a successful applicant and until there had 196 IMMIGRATION LAW REPORTER 3 Imm. L.R. (4th) been discussions with that applicant. Be that as it may, the following op- tions were mentioned by the external consultant: [...] When the decision is made on the applicant CIC will need to begin negotiations with the applicant on the agreement to determine when the applicant can assume the duties of the regulator. If the current regulator is not selected CIC will also need to have discussions with CSIC to determine if they will act as regulator until the successful applicant is prepared to assume the duties. Ideally CIC should bring CSIC and the successful applicant together to arrange for an orderly transfer of authority. CIC will also have to enter into negotiations with the body chosen as regulator for the agreement between the two parties. CIC should be- ing preparing itself for these negotiations by determining what it wants in this agreement. [...] During any transitional period CIC may have to deal with the follow- ing issues in order to avoid disruption in the operations of the regula- tor: • Will existing authorized immigration consultants continue to be authorized during the transitional period? Bill C-35 gives the Minister authority to provide for this by way of regula- tion. The transitional provision in section 6 of Bill C-35 also deals with this issue. • Will members of CSIC in good standing automatically be- come members of the body chosen as the regulator? The new section 91(7) in Bill C-35 would appear to deal with this issue. • Will there continue to be a Code of Conduct, liability insur- ance, a compensation fund and a complaints and disciplinary system during the transitional period? If there is who will pay for them? This is a particularly difficult issue which could arise if the current regulator is not the successful applicant. The negotiations with CSIC will have to try and resolve these issues. Legal Services will need to be consulted on this point. • If the successful applicant is not the current regulator what will happen to cases in the complaints and disciplinary sys- tem? Again CIC will have to try and resolve this issue with CSIC and the successful applicant. Some sort of interim com- plaints and disciplinary system may need to be established. Canadian Soc. of Imm. Consultants v. Canada (MCI) Luc Martineau J. 197

• There may also be issues relating to the winding up of the current regulator during a transitional period. CIC needs to consult Legal Services on its authority to wind up the current regulator and its ability to preserve the liability insurance and the compensation fund currently in operation. 54 Four submissions were considered in January 2011 by the Selection Committee, including proposals made by the applicant and the Institute of Chartered Canadian Immigration Practitioners (ICCIP). The bid of the ICCIP was actually prepared by the CAPIC (notably Phil Mooney, Lynn Gaudet, and Christopher Daw). The CAPIC had publicly announced that it was not interested in becoming the regulator itself, but would neverthe- less lead a “Consortium of interested parties”. This strategic move — from the CAPIC, who is an activist interest group — is not surprising considering that in its final report of September 2010, the external con- sultant had already noted that “[t]he Regulator should be limited to a reg- ulatory function and should not act as a representative organization for immigration consultants”. 55 In their report delivered to the Minister’s attention on January 27, 2011, the Selection Committee came to the conclusion that the ICCIP and the applicant both met the previously announced selection factors (integrity, competence, good governance, accountability and viability). However, the applicant had missed the opportunity to demonstrate how it would address areas of concern that were expressed by the Standing Committee in their report of June 2008 to the House of Commons. On the other hand, the ICCIP had made a concerted effort to demonstrate how it would fully address these areas of concern. 56 The Minister accepted the recommendation of the Selection Commit- tee that the ICCIP, later incorporated under the name of the ICCPC (on February 18, 2011), be designated as the new regulator of immigration consultants. On March 14, 2011, CIC entered into a Non-Disclosure Agreement with the ICCRC with respect to the possibility of the pro- posed regulations being enacted. On March 16, 2011, a further Contribu- tion Agreement was concluded with the ICCRC. 57 On March 18, 2011, the Minister issued a news release announcing the publication of a Notice proposing to amend the 2004 Regulations so that the applicant would be replaced by the ICCRC who would then be recognized as the regulator of immigration consultants. The following day, on March 19, 2011, the proposed regulatory text amending the defi- nition of “authorized representative” (section 2 of IRPR) was published 198 IMMIGRATION LAW REPORTER 3 Imm. L.R. (4th)

in Part I of the Canada Gazette. Moreover, a transitional provision (sub- section 13.1(2) of the IRPR) would permit persons who are members in good standing of the CSIC to be able to continue to act as authorized representatives for a period of 120 days following the coming into force of the proposed regulations. Same will come into force on the day on which they are registered. 58 In the Regulatory Impact Analysis Statement (the March RIAS), it is explained that the intent of the proposed amendments “is to better protect applicants to immigration processes and enhance public confidence in the immigration system by recognizing a regulator of immigration con- sultants that has demonstrated that it meets the necessary organizational competencies to effectively regulate immigration consultants”. Interested persons were invited to make comments concerning the proposals within 30 days after the date of publication of the Notice in Part I of the Can- ada Gazette. 59 On March 23, 2011, a few days before the dissolution of the Houses, Bill C-35 received Royal Assent, now providing specific authority to the Minister himself to revoke or designate the regulatory body for immigra- tion consultants (new section 91 of the Act), but still, to have force of law, an order of the Governor in Council had to be made. However, no such order was made during spring 2011 (the writs for the 41st Canadian general election to be held on May 2, 2011 were issued by the Governor General on March 26, 2011).

III. Present Litigation 60 On April 4, 2011, the applicant commenced an application for leave and judicial review seeking an order of certiorari to set aside any pur- ported action to revoke the applicant’s designation, together with inter- locutory relief to maintain the status quo until final determination by the Court (Docket IMM-2244-11 [2011 CarswellNat 2791 (F.C.)]).

Stay motion 61 Along with the serving and filing of its application for leave and judi- cial review, the applicant sought an order of the Court to stay the deci- sion of the Minister to revoke the CSIC’s designation as the regulator of immigration consultants. 62 As of April 12, 2011, the applicant counted around 1,910 full mem- bers. Moreover, 137 students had completed the requisite immigration practitioner program and had applied to become full members. In prac- Canadian Soc. of Imm. Consultants v. Canada (MCI) Luc Martineau J. 199

tice, some 38 employees were fulfilling the regulatory tasks delegated to the applicant. The applicant was currently handling over 99 complaints and 155 open investigations from the public regarding immigration con- sultants. There were currently 21 on-going disciplinary proceedings. 63 The stay motion was heard on June 7, 2011 and refused by Madam Justice Snider of this Court (the Motions Judge) on June 9, 2011. Essen- tially, she found that the applicant’s allegation of irreparable harm was “speculative”, noting inter alia that “[t]here is no timeline for the enact- ment [of the proposed regulatory amendments] of which anyone is aware (other than perhaps the Minister and the GIC)” (Canadian Society of Immigration Consultants v. Canada (Minister of Citizenship & Immigra- tion), 2011 FC 669 (F.C.) at para 28). 64 The Motions Judge apparently accepted the following statement made by Mrs. Mary Coulter, the Minister’s representative in her affidavit, dated May 30, 2011: Any decision to enact regulations and to change the regulator of im- migration consultants must be made at the executive level, either by the Minister (pursuant to the coming into force of Bill C-35) or by the Governor-in-Council under the present legislative scheme. It can- not be determined at this point when, or even if, such enactments will be made. [My emphasis] 65 Undisclosed to the Motions Judge and only discovered subsequently in the present proceeding, the process of revoking the applicant’s desig- nation and designating the ICCRC as the new regulator was well underway: (a) By May 19, 2011, the 2011 Regulations had been drafted; (b) By May 25, 2011, the GIC Order had been drafted; and, (c) By May 31, 2011, the Minister had signed the recommendation to the Governor in Council (GIC) to repeal the applicant’s recogni- tion as the regulator and to have Bill C-35 come into force on June 30, 2011. 66 Indeed, days after the dismissal of the stay motion, the Government moved rapidly and the impugned enactments were made and registered so that they could become law on the coming into force of Bill C-35 on June 30, 2011. 67 The Court pauses to mention that during the course of argument on the merit of the present judicial review application, applicant’s counsel 200 IMMIGRATION LAW REPORTER 3 Imm. L.R. (4th)

stressed that the conduct of the Minister or its representatives in the stay motion was evidence of bad faith on the part of the Minister or its repre- sentatives who omitted to disclose key information in the respondent’s evidence (the affidavit of Mrs. Coulter dated May 30, 2011) and at the hearing of stay motion on June 7, 2011. 68 The Court will not make any specific finding of fact in this regard, considering that the allegations made by the applicant are serious and directly pose the question whether the alleged acts or omissions consti- tute an interference with the orderly administration of justice or have im- paired the authority or dignity of the Court. As the case may be, it is preferable in the interest of justice and of all parties that such litigious side issues be raised and examined in a separate proceeding in the man- ner provided by Rules 466 to 472 of the Federal Court Rules, SOR/98- 106, if the applicant (or perhaps the Motions Judge) wishes to pursue the matter further, as the case may be.

Effect of the impugned regulations 69 The 2011 Regulations which have put an end to the regulatory role earlier exercised by the applicant are viewed by the Government as “technical coordinating amendments” that have “low to no impact” on the applicant. That said, the Ministerial Regulations are made under the authority conferred to the Minister by new subsections 91(5) and (7) of the Act: • First, as the designated body, the regulatory role over immigration consultants shall be exercised by the ICCRC (this is subject to any concurrent regulatory regime in the province of Qu´ebec: new sub- section 91(7.1) of the Act and paragraphs 3.3(k) to (q) of An Act respecting immigration to Qu´ebec, RSQ, c I-0.2). • Second, as a transitional measure, members of the applicant are members of the ICCRC and are not required to pay membership fees for a period of 120 days following the coming into force of the Act (June 30, 2011). 70 In practice, this means that members of the CSIC who have regular- ized their membership and paid the fees to the ICCRC by October 29, 2011 (the expiry of 120 day period) are not allowed to act or continue to act as “authorized representatives” in connection with a proceeding or application under the Act. Otherwise, they could be found in contraven- tion of section 91 of the Act, and if found guilty, would be liable to a Canadian Soc. of Imm. Consultants v. Canada (MCI) Luc Martineau J. 201

fine, to imprisonment, or to both. However, the transitional measures do not settle a number of unresolved issues. 71 For instance, what happens to cases currently under investigation and disciplinary proceedings undertaken by the applicant? Is there a transfer of the list of members and files to the ICCRC? Are suspended or ex- pelled members of the applicant entitled to be accepted in the member- ship of the ICCRC? Who controls the ICCRC and who are its first direc- tors and officers? When is the first general assembly of members of the ICCRC? 72 In the case at bar, the Minister and CIC have preferred to enter into direct negotiations with the ICCRC and to conclude an agreement prior to the coming into force of the impugned enactments. Conversely, prior to the coming into force of the impugned enactments, the Minister and CIC have preferred not to enter into discussions with the applicant with respect to ongoing issues which are not resolved by the Ministerial Regu- lations (e.g. winding up, transfer of files, disciplinary matters and finan- cial aspects).

New stay motion and new judicial review applications 73 On June 30, 2011, the applicant served and filed a new stay motion alleging that the impugned enactments would cause its demise in the short term, having in the meantime served and filed three other new no- tices of application (Dockets T-1021-11, T-1068-11 and IMM-4256-11) seeking to set aside decisions of the Minister or Cabinet made prior to the coming into force of the impugned enactments. 74 On July 13, 2011, the stay motion came before the undersigned Judge. I noted at the hearing that no notice of application served and filed since April 2011 directly challenged the legality of the impugned enact- ments and that, perhaps, it may be academic to review past “decisions” of the Minister or Cabinet. Rather than proceeding with the stay motion, counsel agreed that it was preferable to proceed rapidly on the merit once the applicant had discontinued its previous applications and had served and filed a new application seeking to set aside the impugned enactments. 75 Following the discontinuance of the existing applications (Dockets IMM-2244-11, IMM-4256-11, T-1021-11 and T-1068-11) on August 4, 2011, upon consent, leave to make the present judicial review application was granted (Docket IMM-5039-11). On October 6 and 7, 2011, the mat- ter was heard in Toronto before the undersigned Judge. 202 IMMIGRATION LAW REPORTER 3 Imm. L.R. (4th)

Applicant’s challenge on the legality of the impugned enactments 76 The applicant challenges the decisions of the Governor in Council and the Minister, implemented by the above-described regulatory enact- ments, on both substantive and procedural grounds. 77 Substantively, the applicant contends that the Governor in Council and Minister exceeded their jurisdiction and acted ultra vires their regu- lation-making authority under the Act for abuse of statutory discretion because the impugned decisions were not made in good faith and with impartiality, but rather were based on irrelevant grounds and factors other than those outlined in the Call for Submissions. 78 The applicant also submits that the Minister’s decision to revoke CSIC’s designation, as well as the regulatory enactments which imple- mented this decision (including the regulation designating ICCRC as the new regulator), are invalid as they are vitiated for breach of procedural fairness, the Minister having failed to follow the selection process as out- lined in the Call for Submissions and thus legitimately expected by the CSIC. 79 The applicant also asserts that the doctrine of legitimate expectations applies to delegated legislative powers creating participatory rights. The Minister is therefore estopped from not complying with the selection pro- cess previously determined in governmental policy guidelines. In this re- spect, the applicant asserts that the Minister was not entitled to consider factors other than those previously considered by the Selection Commit- tee. The fact that the Selection Committee was not satisfied with the re- sponses provided to the concerns set out in the Parliamentary Standing Committee report in 2008 was not relevant either. 80 Moreover, the applicant says the Minister selected the ICCRC as reg- ulator despite the fact that it had not responded to the Call for Submis- sions under the selection process. According to the applicant, even the ICCIP (which was incorporated only a month before its designation under the name ICCRC) was not really the body making the bid. Instead, the CAPIC filed submissions under the name of ICCIP, which was not a legal entity and had no legal status. 81 The applicant also alleges that the Minister’s actions and comments prior to, during, and following the selection process, as well as those of members of his staff, raise a reasonable apprehension of bias. The appli- cant contends that the current directors, president and CEO of the ICCRC are CAPIC members who lobbied the Minister and CIC before the intro- duction of Bill C-35 to have the CSIC replaced. Canadian Soc. of Imm. Consultants v. Canada (MCI) Luc Martineau J. 203

82 The applicant also stresses that it is Mr. Linklater who retained Sus- sex Circle in 2009 to conduct a review and provide “an analysis and as- sessment of the threshold required to conclusively determine when the level of governance in a not for profit organization had deteriorated to a point that the mandate of the board of directors could be revoked by the government with minimal legal risk”. The applicant contends that the Sussex Circle was instructed by Mr. Linklater to obtain information about the applicant from CIC officials from Immigration Branch (and not from the applicant itself), and that, in any event, its recommendation not to wind down the applicant was disregarded by CIC. 83 As for the second set of procedural issues raised by the applicant, it is submitted that the Governor in Council’s Order fixing June 30, 2011 as the coming into force date of Bill C-35 is of no force and effect because it was not registered within seven days after it was made and thereby fails to comply with section 9 of the SIA. Furthermore, the applicant ar- gues that even if the order is valid, the Ministerial Regulations remain invalid as they were made three days prior to the date on which Bill C-35 granting the Minister statutory authority to make such regulations came into effect. 84 For the reasons expressed hereunder, the applicant’s arguments must be dismissed by the Court.

IV. Limited Scope of Judicial Review 85 In reference to the constitutional role of the superior courts in main- taining the rule of law, speaking for the in Syndicat national des employ´es de la commission scolaire r´egionale de l’Outaouais v. U.E.S., local 298, [1988] 2 S.C.R. 1048 (S.C.C.) at para 127, Justice Beetz eloquently expressed the singular nature of judicial review and its paradox: [...] When an administrative tribunal exceeds its jurisdiction, the ille- gality of its act is as serious as if it had acted in bad faith or ignored the rules of natural justice. The role of the superior courts in main- taining the rule of law is so important that it is given constitutional protection: Crevier v. Attorney General of Quebec, [1981] 2 S.C.R. 220. Yet, the importance of judicial review implies that it should not be exercised unnecessarily, lest this extraordinary remedy lose its meaning. 86 Naturally, in cases involving the exercise of powers granted by Par- liament to the Executive, this judicial review role is performed by the Federal Courts under sections 18 and 28 of the Federal Courts Act, RSC 204 IMMIGRATION LAW REPORTER 3 Imm. L.R. (4th)

1985, c F-7 and this jurisdiction is plenary in principle (Canada (Human Rights Commission) v. Canadian Liberty Net, [1998] 1 S.C.R. 626 (S.C.C.) at paras 35-36). In the case at bar, the applicant submits that the impugned enactments are ultra vires, violate the duty of procedural fair- ness, and were made contrary to the SIA and without statutory authority.

Rule of law 87 Access to the courts is a fundamental tenet of democracy and by ex- tension of the principle of separation of powers. Judicial review is essen- tially concerned with legality, whether from a constitutional, statutory or administrative point of view. At its most basic level, the rule of law vouchsafes to the citizens and residents of the country a stable, predict- able and ordered society in which to conduct their affairs; it provides a shield for individuals from arbitrary state action (Reference re Secession of Quebec, [1998] 2 S.C.R. 217 (S.C.C.) at para 70). 88 As far as the legality of a piece of legislation adopted by Parliament or a Legislature is concerned, the reviewing role of the Court is limited to examining its conformity with the Constitution, including the Canadian Charter of Rights and Freedoms (the Charter) and unwritten constitu- tional principles. A breach of the rule of law cannot lead to the invalidity of a statute, except in cases where a statute has not been enacted in the correct manner and form: British Columbia v. Imperial Tobacco Canada Ltd., 2005 SCC 49 (S.C.C.) at paras 58-60 (Imperial Tobacco).

Divided Constitutional powers over the regulation of immigration consultants 89 In our Canadian system of responsible government, there is no sepa- ration of powers between the two political branches (legislative and ex- ecutive), and subject to the limitation found in section 96 of the Constitu- tional Act 1867 applicable to the Legislatures, there may be laws or regulations conferring legislative, quasi-judicial or administrative and regulatory powers to bodies invested of the functions of regulating an occupation and licensing members of a profession, trade or other activity, subject to the constitutional division of powers between Parliament and the Legislatures. 90 In this respect, Parliament and the Legislatures both possess under section 95 of the Constitutional Act of 1867 a shared jurisdiction in im- migration matters, while the regulation of professions rests in the exclu- sive legislative power of the provinces. Nevertheless, Parliament has Canadian Soc. of Imm. Consultants v. Canada (MCI) Luc Martineau J. 205

constitutional authority to notably allow immigration consultants to give advice or represent people who are subject to a proceeding or application under the Act. 91 Indeed, it has been held that the Governor in Council could legally establish “a licensing system” in the area of persons wishing to act as representatives in an immigration or refugee proceeding, including immi- gration consultants, pursuant to paragraph 114(1)(v) of the former Immi- gration Act, RSC 1985, c I-2. See Law Society (British Columbia) v. Mangat, 2001 SCC 67 (S.C.C.) (Mangat). That said, this is not a case where this Court is asked to revisit aspects of the Mangat decision.

The present attack 92 To a large extent, the applicant has challenged the wisdom and effec- tiveness of the legislative amendments introduced by Bill C-35, notably reproaching the Minister for not having carried out the Standing Com- mittee’s recommendation that the CSIC be “re-established” under stand- alone legislation, while repeatedly and deliberately taking the comments of the Standing Committee out of context. However, the judiciary’s role “is not...to apply only the law of which it approves. Nor is it to decide cases with a view simply to what the judiciary (rather than the law) deems fair or pertinent” (Imperial Tobacco, above, at para 52). 93 Clearly, the Executive made a policy decision, which was ultimately endorsed by Parliament, in choosing not to follow the Standing Commit- tee recommendation that the Government introduce stand-alone legisla- tion to re-establish the applicant as a non-share capital corporation. Whether this was the result of CAPIC’s lobby has no bearing with the legality of Bill C-35, which clearly falls within the purview of Parlia- ment’s legislative powers and is not contrary to the Constitution, includ- ing the Charter and unwritten constitutional principles. 94 That said, the applicant contends that Bill C-35 did not legally come into force June 30, 2011 and is not the law of Canada today. This asser- tion is based on the assumption that the requirements set out in section 9 of the SIA have not been respected in the case of the making and regis- tration of the GIC Order; consequently, new section 91 of the Act and the Ministerial Regulations can have no force and effect. Subsidiarily, the Ministerial Regulations which are purportedly made under the authority of new section 91 of the Act are otherwise invalid because they were made and registered prior to the coming into force of Bill C-35. 206 IMMIGRATION LAW REPORTER 3 Imm. L.R. (4th)

95 Moreover, the applicant submits that the enactments revoking the 2011 Regulations and the Ministerial Regulations are ultra vires and ex- ceed the regulation-making authority under (former or new) section 91 of the Act on the grounds of abuse of discretion, bad faith and reliance upon irrelevant considerations. The applicant also submits that the making of both the 2011 Regulations and the Ministerial Regulations is contrary to the applicant’s legitimate expectations and right to be heard, while the conduct by the Minister and his staff at CIC raises a reasonable appre- hension of bias.

What is jurisdictional? 96 Jurisdictional issues such as the scope of the powers conferred to the Governor in Council and the Minister, issues of procedural fairness (in- cluding allegations of bad faith and bias), and compliance with the proce- dural requirements found in the SIA, are to be reviewed on a standard of correctness. Be that as it may, the pragmatic and functional approach does not apply to legislative acts; such an enquiry is only required where an adjudicative or policy-making function is being exercised: United Taxi Drivers’ Fellowship of Southern Alberta v. Calgary (City), 2004 SCC 19 (S.C.C.) at para 5. 97 In order to be the law of Canada, the amendments introduced by Bill C-35 and the corollary regulatory enactments must be legally in force, which supposes that all procedural requirements found in the SIA must have been respected. However, where it comes to the exercise of statu- tory powers granted to the Governor in Council and the Minister, it is debatable whether the applicant’s vires argument raises a “true question of vires” as described in New Brunswick (Board of Management) v. Dun- smuir, 2008 SCC 9 (S.C.C.) at para 59. 98 What is truly challenged here is the exercise of a discretionary deci- sion-making power by “regulation”, which the applicant submits is re- viewable by the Court at least in case of bad faith or improper purpose. Since the impugned enactments affected its rights, privileges or interests, the applicant further submits that there was a duty to act fairly in the process of revoking its designation and in selecting a new regulator. How should these issues be reviewed by the Court, if they are indeed reviewable? 99 First, judicial review over executive decision-making requires a con- sideration of both the form of the decision and the nature of the decision- maker’s functions in light of the enabling legislation when determining Canadian Soc. of Imm. Consultants v. Canada (MCI) Luc Martineau J. 207

whether a duty of procedural fairness is imposed to the decisionmaker. It is understood that the qualification of an action or decision made by the Government or one of its Ministers as legislative, quasi-judicial or ad- ministrative will naturally have some bearing on the scope of judicial review, but in practice it may be difficult to draw a line. 100 Second, as suggested by Ms. Sara Blake in her book Administrative Law in Canada 4th ed. (Butterworths, 2006) at page 217, “[i]t would be more sensible to draw the line between adjudicative on one side and pol- icy and legislative decisions on the other”. True policy decisions will usually be dictated by financial, economic, social and political factors or constraints. In such decisions, the authority attempts to strike a balance between efficiency and thrift, in the context of planning and predetermin- ing the boundaries of its undertakings and of their actual performance. See Brown v. British Columbia (Minister of Transportation & High- ways), [1994] 1 S.C.R. 420 (S.C.C.) at para 38 and Knight v. Imperial Tobacco Canada Ltd., 2011 SCC 42 (S.C.C.) at paras 72-91. 101 Third, the Governor in Council (Cabinet) and the Minister are known to make policy decisions at the highest level of Government and are ac- countable to Parliament. However, where they are exercising a statutory power (including a legislative one) derived by an Act of Parliament, the legality of their actions is not automatically immune from judicial review (Inuit Tapirisat of Canada v. Canada (Attorney General), [1980] 2 S.C.R. 735 (S.C.C.) at page 748). The Government must always comply with the rule of law which is “a fundamental postulate of our constitu- tional structure” (Roncarelli v. Duplessis, [1959] S.C.R. 121 (S.C.C.) at page 142). Indeed, courts will always be allowed to intervene in “an egregious case or where there is proof of an absence of good faith” (Canadian Wheat Board v. Canada (Attorney General), 2009 FCA 214 (F.C.A.) at para 37; Thorne’s Hardware Ltd. v. R., [1983] 1 S.C.R. 106 (S.C.C.) at page 111). 102 Four, assuming that the rule of law applies to the making of regula- tions — in principle it does not apply to the passing of legislation by Par- liament or a legislature — this could explain why such a regulation-mak- ing power may not be used for a completely irrelevant purpose, so as to make a particular regulation ultra vires of the powers delegated by Par- liament to the Governor in Council or the Minister. Naturally, it is up to the party attacking the regulation to prove bad faith or demonstrate what that illicit purpose might be: Canadian Assn. of Regulated Importers v. Canada (Attorney General), [1994] 2 F.C. 247 (Fed. C.A.) at paras 11- 208 IMMIGRATION LAW REPORTER 3 Imm. L.R. (4th)

24; Jafari v. Canada (Minister of Employment & Immigration), [1995] 2 F.C. 595 (Fed. C.A.) at page 602. 103 Five, regulations or policies of the Governor in Council or the Min- ister are not reviewable, except in cases of excess of jurisdiction, failure to comply with legislative or regulatory requirements. In other words, it is not open to a court to determine the wisdom of the regulation or policy and to assess their validity on the basis of the court’s preferences. See Canadian Council for Refugees v. R., 2008 FCA 229 (F.C.A.) at para 57 and Mercier c. Canada (Service correctionnel), 2010 FCA 167 (F.C.A.) at paras 78 and 80. Such approach is entirely consistent with the treat- ment reserved in case of legislations passed by Parliament or a Legisla- ture (Imperial Tobacco, above, at paras 58-60). 104 Six, regulatory exercise becomes perilous in cases where individual rights may be at stake or an entity has been singled out for adverse treat- ment. Simply stated, one cannot label an act as a “regulation” to abrogate or diminish a citizen’s right to procedural protection. This could be the case of municipal by-laws affecting property rights of land owners on the territory of a municipality, where there may be a right to be “heard” by the municipal Council (Homex Realty & Development Co. v. Wyoming (Village), [1980] 2 S.C.R. 1011 (S.C.C.) at pages 1026, 1030 and 1050). 105 Another example concerns the revocation of citizenship by the Exec- utive. The fact that citizenship is granted to an individual by legislation (an Act of Parliament) and that same can be subsequently revoked by an order in council (delegated legislation) does not prevent the Court from examining the legality of any such order and treating it as a “decision”, considering that it will adversely affect the rights of the individual in question and that the Governor in Council must be satisfied that the citi- zenship was obtained by “false representation or fraud or by knowingly concealing material circumstances” (Oberlander v. Canada (Attorney General), 2004 FCA 213 (F.C.A.)). 106 Closer to the above examples are decisions, policies and regulations which may directly affect the status of immigration consultants acting as “authorized representatives” under the Act. It is useful to begin by recal- ling that licensing is essentially the authority of a regulator to decide who shall be permitted to earn their living by the pursuit of a particular call- ing: Ontario, Royal Commission Inquiry into Civil Rights, (Report No 1, vol 3) Commissioner James Chalmer McRuer (Toronto Queen’s Printer, 1968-1971) 1163 (The McRuer Report). In this regard, the Supreme Court of Canada has stated in Reference re Public Service Employee Canadian Soc. of Imm. Consultants v. Canada (MCI) Luc Martineau J. 209

Relations Act (Alberta), [1987] 1 S.C.R. 313 (S.C.C.) at page 368, that “[w]ork is one of the most fundamental aspects in a person’s life, provid- ing the individual with a means of financial support and, as importantly, a contributory role in Society”. 107 In practice, licensing in connection with a proceeding or application under the Act has been sub-delegated to the body designated by regula- tion. Such sub-delegation has been held to be valid by the (Law Society of Upper Canada, above, at paras 72-80). In turn, the decisions made in membership and discipline matters by the CSIC (or the ICCRC) are judicially reviewable by this Court (Onuschak, above, at paras 33-34 and Mooney, above, at para 83). The decisions of the regulatory body must pass the test of reasonableness and respect rules of fairness. This is not surprising since the power of a self-governing body to discipline its members is clearly a “judicial power” and that “no element of policy should be present in the exercise of this power” (The McRuer Report at 1181).

And the present case... 108 What about policy decisions and regulations revoking the power of a regulatory body to licence individuals and transferring same to another regulatory body selected by the Executive after a Call for Submissions? 109 In this case, the applicant contends that the Government’s decision to revoke its designation as a regulator and the enactments which imple- mented the decision are subject to the duty of fairness, as the applicant is singled out and adversely affected by these. The alleged grounds are le- gitimate expectations and bias. 110 The duty to act fairly and the doctrine of legitimate expectations are not applicable in the circumstances of this case, at least not in the ways suggested by the applicant. The applicant seems to assimilate the revoca- tion of its regulatory designation as if it was some sort of “decision” made by the Government adversely affecting the rights of an individual who makes a living (or a corporation who pursues economic activities), but this is not the case here: • The applicant does not act in any representative capacity (like a professional association or a trade union), but as the designated regulator of immigration consultants; • As of June 30, 2011, members in good standing of the applicant are deemed by the Ministerial Regulations to be members of the ICCRC and are accordingly not deprived of their capacity “to earn 210 IMMIGRATION LAW REPORTER 3 Imm. L.R. (4th)

their living by the pursuit of a particular calling”, so long as they maintain their membership, pay the fees and are not expelled by the ICCRC; • As a corporation without share capital constituted under the Can- ada Corporations Act, the applicant has no regulatory power over any profession; • Any regulatory monopoly granted to the applicant (or the ICCRC) is a power exclusively derived and conferred to the body designed in the regulations of the Governor in Council or the Minister. Thus, any such monopoly can always be taken away by its grantor in the same manner, here by the 2011 Regulations in the case of the applicant; and, • Apart from improper purpose or bad faith (none has been proven in the Court’s opinion), the fact that the Minister or CIC have pre- conceived opinions or expressed a preference is normal in the case of policy oriented decisions. This should not attract the Court’s attention on the ground of reasonable apprehension of bias (Old St. Boniface Residents Assn. Inc. v. Winnipeg (City), [1990] 3 S.C.R. 1170 (S.C.C.)). 111 It must be remembered that Parliament has full plenary power to cre- ate federal boards, commissions, tribunals or other bodies invested with the quasi-judicial or regulatory powers conferred to them by legislation. Such administrative bodies or tribunals are not courts and by contrast, lack this constitutional distinction from the executive. It is properly the role and responsibility of Parliament to determine the composition and structure required to discharge the responsibilities bestowed upon them (Ocean Port Hotel Ltd. v. British Columbia (General Manager, Liquor Control & Licensing Branch), [2001] 2 S.C.R. 781 (S.C.C.) at paras 23- 24). 112 As stated by learned authors, “[a]n essential task of democratic socie- ties is to establish a proper balance between freedom and order”, and thus, from this general principle of democratic governments, “[t]he issue of regulation [of occupations] involves the role of government in recon- ciling the special interests of the members of the occupation with the general concerns of the public” (Alex Bryson and Morris M. Kleiner, “The Regulation of Occupations” (2010) 48 British Journal of Industrial Relations 670-675 at page 670). In the case of immigration consultants, the desirability of allowing by a regulation made by the Governor in Council (the 2004 Regulations) the self-regulating mode over direct li- Canadian Soc. of Imm. Consultants v. Canada (MCI) Luc Martineau J. 211

censing by a board created by statute was clearly a policy choice made by the Government. Whether it would have been preferable to have cre- ated a professional self-regulatory scheme that rested instead on an Act of Parliament was purely a policy question which was not judicially re- viewable (Law Society of Upper Canada, above, at para 62). 113 The Court finds that the decision to terminate the regulatory mandate over immigration consultants given to the CSIC (the 2011 Regulations) by a regulation of the Governor in Council, and to designate in lieu and place the ICCRC by way of a regulation of the Minister (the Ministerial Regulations), is essentially a “legislative” action (whether it results from an Act of Parliament or from a regulation made by the Executive branch). That said, while the duty of fairness and the doctrine of legiti- mate expectations have no application to the exercise of legislative pow- ers, it is debatable whether subordinate legislation can lawfully be made in breach of categorical and specific assurance of prior consultation (Reference re Canada Assistance Plan (Canada), [1991] 2 S.C.R. 525 (S.C.C.) at pages 557-560 (Canada Assistance Plan); Apotex Inc. v. Canada (Attorney General), [2000] 4 F.C. 264 (Fed. C.A.) at paras 22-24 (majority), and 100, 102, 105 and 115 (minority) (Apotex)). 114 As a final note on the limited scope of the judicial review, our accept- ance of the rule of law, whose content may vary from one Society to another, supposes that state action will be consistent with fundamental values of its Society, such as, equality, fairness, transparency, accounta- bility, consistency and predictability. Assuming that the rule of law ap- plies to the making of regulations (which may be debatable), the issue is whether the process which led to the impugned enactments was fair and transparent. For the reasons hereunder, the Court finds that the impugned enactments are authorized by statute, that the conditions for their enact- ment have been respected and that there were no improper purposes or motives in revoking the designation of the applicant as the regulator of immigration consultants and in designating the ICCRC as the new regu- lator. Moreover, this is not “an egregious case” where the intervention of the Court is warranted to uphold the rule of law, and as far as any duty to consult is concerned, it has been satisfied in this case.

V. Impugned Enactments Authorized by Statute and for no Improper Purposes or Motives 115 The “perspective within which a statute is intended to operate” is the starting point of any court analysis of an allegation that a decision-maker 212 IMMIGRATION LAW REPORTER 3 Imm. L.R. (4th)

took into account irrelevant considerations or acted for an improper pur- pose; in other words, the “perspective” is another way of describing the policy and objects of the statute, and as the case may be, of a particular set of regulations (C.U.P.E. v. Ontario (Minister of Labour), 2003 SCC 29 (S.C.C.) at paras 92-95). 116 For the reasons below, the Court finds that the impugned enactments are authorized by statute and that they have been enacted for no improper purposes or motives.

Framework legislation and regulatory scheme 117 The Act is “framework legislation”, that is to say, the Act contains the core principles and policies of the statutory scheme and, in view of the complexity and breadth of the subject matter, is relatively concise. Framework legislation contemplates broad delegations of legislative power. As observed by the Federal Court of Appeal in de Guzman v. Canada (Minister of Citizenship & Immigration), 2005 FCA 436 (F.C.A.) at para 23, “[t]he creation of secondary policies and principles, the implementation of core policy and principles, including exemptions, and the elaboration of crucial operational detail, are left to regulations, which can be amended comparatively quickly in response to new problems and other developments.” 118 Sections 4 and 5 of the Act provide the enabling authority of both the Minister and the Governor in Council. Except as otherwise provided in section 4 of the Act, the Minister is responsible for the administration of the Act. On the other hand, except as otherwise provided, the Governor in Council may make any regulation that is referred to in this Act or that prescribes any matter whose prescription is referred to in the Act. That said, there are multiple ways in which the activities of immigration con- sultants under the Act can be regulated by Parliament, and by extension, the Governor in Council or the Minister. 119 Direct licensing by the federal authority is one option. For example, a trustee appointed in bankruptcy matters under the Bankruptcy and Insol- vency Act, RSC 1985, c B-3, must hold a licence issued by the Super- intendant, whether or not he or she is already a member of a selfregulated body (e.g. certified accountants). Likewise, an immigration consultant who wishes to advise or represent a person in an application made under An Act respecting immigration to Qu´ebec, RSQ, c I-0.2 must make an application for recognition and pay the prescribed fees (Regulation re- specting immigration consultants, RRQ, c I-0.2, r 0.1). Canadian Soc. of Imm. Consultants v. Canada (MCI) Luc Martineau J. 213

120 Another option is to allow members of a particular trade, profession or occupation, such as lawyers, paralegals and immigration consultants, to advise or represent a person if they are members of a designated self- regulated body. This was the option chosen in 2004 by the federal au- thorities. Likewise, the (Qu´ebec) Minister of Immigration and Cultural Communities notably recognizes as an immigration consultant a member in good standing of a body, other than the bar of the province or the Chambre des notaires du Qu´ebec, designated as an “authorized represen- tative” under the federal regulations (section 4 of the Regulation respect- ing the immigration consultants).

Former section 91 of the Act and regulations 121 Former section 91 of the Act specifically provided that “[t]he regula- tions may govern who may or who may not represent, advise or consult with a person who is the subject of a proceeding or application before the Minister, an officer or the Board”. These regulations were made by the Governor in Council and allowed the members of the CSIC to act as “authorized representatives” (section 2 of the IRPR, as amended by the 2004 Regulations). 122 The basic objective of the 2004 Regulations made pursuant to former section 91 of the Act was to prevent unqualified and unethical immigra- tion consultants from representing clients and to enhance public confi- dence in Canada’s immigration and refugee system. Bill C-35 which amends section 91 of the Act, the 2011 Regulations and the Ministerial Regulations, which all have to be read together, have been made for the very same stated purposes. 123 As of June 30, 2011, the 2011 Regulations made by the Governor in Council repealed the former regulatory provisions which defined the per- sons authorized to act in immigration and refugee proceedings and con- ferred monopoly to the applicant with respect to the regulation of immi- gration consultants acting as “authorized representatives” under the Act and its regulations. The 2011 Regulations were made pursuant to the au- thority conferred to the Governor in Council by subsection 5(1), section 14 and former section 91 of the Act. 124 Section 4 of the 2011 Regulations provides: 4. These Regulations come into force on the day on which section 1 of An Act to amend the Immigration and Refugee Protection Act, chapter 8 of the Statutes of Canada, 2011, comes into force, but if 214 IMMIGRATION LAW REPORTER 3 Imm. L.R. (4th)

they are registered after that day, they come into force on the day on which they are registered. 4. Le pr´esent r`eglement entre en vigueur a` la date d’entr´ee en vigueur de l’article 1 de la Loi modifiant la Loi sur l’immigration et la pro- tection des r´efugi´es, chapitre 8 des Lois du Canada (2011), ou, si elle est post´erieure, a` la date de son enregistrement. 125 As explained below, some of the regulatory powers conferred to the Governor in Council by former section 91 of the Act were transferred to the Minister following the enactment of section 1 of Bill C-35. By the effect of the making and registration of the GIC Order, Bill C-35 has purportedly come into force on June 30, 2011.

New section 91 of the Act 126 Most relevant for this application are new subsections 91(1), (2), (5), (5.1) and (7) of the Act which read: 91. (1) Subject to this section, no person shall knowingly, directly or indirectly, represent or advise a person for consideration — or offer to do so — in connection with a proceeding or application under this Act. (2) A person does not contravene subsection (1) if they are: (a) a lawyer who is a member in good standing of a law society of a province or a notary who is a member in good standing of the Chambre des Notaries du Qu´ebec; (b) any other member in good standing of a law society of a province or the Chambre des Notaries du Qu´ebec, including a paralegal; or (c) a member in good standing of a body designated under sub- section (5). [...] (5) The Minister may, by regulation, designate a body whose mem- bers in good standing may represent or advise a person for considera- tion — or offer to do so — in connection with a proceeding or appli- cation under this Act. (5.1) For greater certainty, subsection (5) authorizes the Minister to revoke, by regulation, a designation made under that subsection. [...] (7) The Minister may, by regulation, provide for measures respecting any transitional issues raised by the exercise of his or her power under subsection (5), including measures Canadian Soc. of Imm. Consultants v. Canada (MCI) Luc Martineau J. 215

(a) making any person or member of a class of persons a member for a specified period of a body that is designated under that subsection; and (b) providing that members or classes of members of a body that has ceased to be a designated body under that subsection con- tinue for a specified period to be authorized to represent or advise a person for consideration — or offer to do so — in connection with a proceeding or application under this Act without contravening subsection (1). 91. (1) Sous r´eserve des autres dispositions du pr´esent article, com- met une infraction quiconque sciemment, de fa¸con directe ou in- directe, repr´esente ou conseille une personne, moyennant r´etribution, relativement a` une demande ou a` une instance pr´evue par la pr´esente loi, ou offre de le faire. (2) Sont soustraites a` l’application du paragraphe (1) les personnes suivantes: a) les avocats qui sont membres en r`egle du barreau d’une prov- ince et les notaires qui sont membres en r`egle de la Chambre des notaires du Qu´ebec; b) les autres membres en r`egle du barreau d’une province ou de la Chambre des notaires du Qu´ebec, notamment les parajuristes; c) les membres en r`egle d’un organisme d´esign´e en vertu du paragraphe (5). [...] (5) Le ministre peut, par r`eglement, d´esigner un organisme dont les membres en r`egle peuvent repr´esenter ou conseiller une personne, moyennant r´etribution, relativement a` une demande ou a` une instance pr´evue par la pr´esente loi, ou offrir de le faire. (5.1) Il est entendu que le paragraphe (5) autorise le ministre a` r´evo- quer, par r`eglement, toute d´esignation faite sous son r´egime. [...] (7) Le ministre peut, par r`eglement, pr´evoir des mesuresa ` l’´egard de toute question transitoire soulev´ee par l’exercice du pouvoir que lui conf`ere le paragraphe (5), notamment des mesures: a) donnant a` toute personne — individuellement ou au titre de son appartenance a` une cat´egorie d´etermin´ee — le statut de membre d’un organisme d´esign´e en vertu de ce paragraphe pour la p´eriode pr´evue par r`eglement; 216 IMMIGRATION LAW REPORTER 3 Imm. L.R. (4th)

b) permettant a` tout membre — individuellement ou au titre de son appartenance a` une cat´egorie d´etermin´ee — d’un organ- isme qui a cess´e d’ˆetre un organisme d´esign´e vis´e au mˆeme paragraphe de continuer d’ˆetre soustrait a` l’application du paragraphe (1) pour la p´eriode pr´evue par r`eglement. [My underlinings] 127 Moreover, in addition to the regulatory powers already granted by subsection 5(1) and section 14 of the Act, the Governor in Council is given the power to make “regulations” requiring the body designated by the Minister to provide certain key information regarding its membership and activities under new subsection 91(6) of the Act which reads as fol- lows: (6) The Governor in Council may make regulations requiring the des- ignated body to provide the Minister with any information set out in the regulations, including information relating to its governance and information to assist the Minister to evaluate whether the designated body governs its members in a manner that is in the public interest so that they provide professional and ethical representation and advice. (6) Le gouverneur en conseil peut, par r`eglement, exiger que l’organisme d´esign´e fournisse au ministre les renseignements r´egle- mentaires, notamment des renseignements relatifs a` sa r´egie interne et des renseignements visant a` aider le ministrea ` v´erifier si l’organisme r´egit ses membres dans l’int´erˆet public de mani`ere que ces derniers repr´esentent ou conseillent les personnes en conformit´e avec les r`egles de leur profession et les r`egles d’´ethique. [My underlinings] 128 Regulations pursuant to new subsection 91(6) of the Act have not yet been made by the Governor in Council. Therefore, how can the Minister evaluate if a designated body governs its members in a manner that is in the public interest, or conversely, how can a designated body effectively regulate its members if it is unaware of the rules upon which the Minister may base itself to evaluate its governance? 129 Be that as it may, the question above need not be answered in this proceeding since legally speaking, the Minister was not called to exercise the power to revoke a designation pursuant to new subsections 91(5) and (5.1) of the Act. It was the Governor in Council that effectively revoked, pursuant to former section 91 of the Act, the designation of the applicant as the regulating body of the immigration consultants. There was nothing illegal or objectionable in proceeding in this manner, nor is there any evidence of improper purpose. Canadian Soc. of Imm. Consultants v. Canada (MCI) Luc Martineau J. 217

The 2011 Regulations are technical coordinating amendments 130 The Court accepts that it was necessary to amend provisions of the IRPR in view of the coming into force of new section 91 of the Act. 131 According to the July RIAS, the 2011 Regulations amend the IRPR in order to facilitate application processing and enhance program integrity by providing CIC officers with the applicable membership number and the contact information of a person who is advising or representing an immigration applicant for consideration at any stage, including leading up to the application or proceeding before the Minister or the Immigra- tion and Refugee Board of Canada. 132 The amendments introduced by the 2011 Regulations also ensure that the wording of the IRPR is consistent with the Act. More particularly, technical coordinating amendments have been undertaken: • Repeal the definition of “authorized representative” in section 2 of the IRPR. The entities authorized in that definition are now con- tained in the exception to the general prohibition as set out in sub- section 91(2) of the Act, as amended; • Repeal Part 2, Division 4 of the IRPR regarding the prohibition against “representation for a fee” and its exceptions. Similar pro- visions are now contained in subsections 91(1) and 91(3) of the Act, as amended; • Replace paragraphs 10(2)(c.1) and 10(2)(c.2) of the IRPR regard- ing application requirements for persons using a representative, and replace with the requirements that the application include: • the name, postal address, telephone number, fax number and electronic mail address, if any, of any person or en- tity — or a person acting on its behalf — representing the applicant, whether for consideration or not; • the name of the body and the membership identification number of any person that has provided advice or is repre- senting the applicant for consideration under subsection 91(2) of the Act, including members of a body of the Chambre des notaires du Qu´ebec, members of a body des- ignated by the Minister or members of a provincial law so- ciety, which include members of the bar and paralegals; and, • the name, postal address, telephone number, fax number and electronic mail address, if any, of any entity — or a 218 IMMIGRATION LAW REPORTER 3 Imm. L.R. (4th)

person acting on its behalf — that has provided advice for consideration under subsection 91(4) of the Act. 133 Alternatives prior to the making of the impugned regulations were considered by the Government. Indeed, the March RIAS explains that “[a] legislative approach to reconstitute CSIC as a statutory body, as sug- gested by the House of Commons Standing Committee, was rejected due to concerns about a lengthy and resource intensive implementation pro- cess. While CIC has not initiated such changes as recommended by the Standing Committee, it seeks to move forward with the legislative changes to [the Act] found in Bill C-35, which would strengthen govern- ment oversight of the regulator and should improve discipline of its members through the information sharing provision.”

No improper purpose or motive 134 This now brings us to examine the legality of the Minister’s exercise of his new regulatory power under subsection 91(5) of the Act to desig- nate a body whose members in good standing may represent or advise a person for consideration — or offer to do so — in connection with a pro- ceeding or application under this Act. 135 According to the evidence on file, the GIC Order, the 2011 Regula- tions and the Ministerial Regulations were coordinated together to imple- ment the Minister’s earlier public announcement of March 18, 2011 to replace the applicant with the ICCRC as the regulator. As affirmed at the cross-examination of the Minister’s representative: These regulatory amendments would be considered together in the context of the previously prepublished proposed regulatory amend- ments that dealt substantively with the same issue of changing the regulator of immigration consultants [...] These regulatory changes [...] were meant to coordinate and work together. 136 As a preliminary remark, self-regulation is a privilege granted to the members of a recognized body of professionals, tradesmen or other occu- pational groups. It places important obligations on the regulatory body. Being the designated regulatory body of the immigration consultants, to use the metaphor borrowed in James T. Casey, The Regulation of Profes- sions of Canada (Carswell, Toronto, 1994), at pages 1-3, the applicant had a clear interest in “ridding the profession of the incompetent and the unethical” and in “the proper functioning of their organization”. Canadian Soc. of Imm. Consultants v. Canada (MCI) Luc Martineau J. 219

137 Moreover, as cautioned by the Supreme Court of Canada in Pharmascience inc. c. Binet, 2006 SCC 48 (S.C.C.) at para 36: The privilege of professional self-regulation therefore places the indi- viduals responsible for enforcing professional discipline under an on- erous obligation. The delegation of powers by the state comes with the responsibility for providing adequate protection for the public. Finney confirms the importance of properly discharging this obliga- tion and the seriousness of the consequences of failing to do so. 138 Bill C-35, as mentioned earlier, was tabled at the House of Commons by Minister Kenney on June 8, 2010. Concurrently with its tabling, the Minister announced that it was also taking immediate steps to address “a lack of public confidence in the regulation of immigration consultants”. The resultant was the publication in Part I of the Canada Gazette of the Notice of intent (June 12, 2010) and the Call for Submission (August 28, 2010). This was clearly a policy decision made by the Minister. Despite the allegations made by the applicant, there is no evidence of improper purposes or motives. 139 The use of the RIAS to determine both the purpose and intended ap- plication of a regulation has been frequent in this Court and others, and this across a wide range of interpretive settings: Bristol-Myers Squibb Co. v. Canada (Attorney General), 2005 SCC 26 (S.C.C.) at para 157 and Saputo Inc. v. Canada (Attorney General), 2009 FC 1016 (F.C.) at para 31, confirmed by 2011 FCA 69 (F.C.A.) (leave to appeal to the Su- preme Court of Canada denied). In the case at bar, the Court finds the RIAS a credible and reliable source of information with respect to the intentions of the Minister. 140 Both the March and July RIAS note that the complaints made to the Standing Committee in 2008 appear to indicate that the current govern- ance and accountability framework within which the CSIC operates does not ensure that immigration consultants are being adequately regulated in the public’s interest with respect to the provision of professional and eth- ical counselling, representation and advice. The fact that these com- plaints were unsubstantiated in the applicant’s opinion is irrelevant. What counts here is the perception that the Government had; legislative or reg- ulatory exercise is not conditioned by court rules, as if a person is ac- cused of a crime, but largely by political discourse and debate in Parlia- ment, in the media and other public forums. 141 According to the July RIAS, the Minister’s decision to designate the ICCRC as the new regulator is based on the results of the public selec- 220 IMMIGRATION LAW REPORTER 3 Imm. L.R. (4th)

tion process initiated through the publication of a Notice of intent on June 12, 2010, followed by the publication of a Call for Submissions on August 28, 2010, whereas all stakeholders and the public were allowed to participate and comment. 142 Following the invitation of March 2011 for public comments, of the 207 comments received, 149 were supportive of the Government’s pro- posed amendment to remove the CSIC’s recognition and 39 were op- posed. One of the submissions received also included a petition signed by 479 CSIC members that were supportive of the naming of the ICCRC. Based on the results of the Selection Committee review, the ICCRC has been proposed and retained by the Minister as the regulator to govern immigration consultants. 143 After the completion of the Selection process and pre-publication in March 2011 of its intention to replace the CSIC by the ICCRC, was the Minister ill-advised in putting its confidence in an inexperienced player whose directorship may not be truly independent from the CAPIC and whose membership may accept “ghost consultants” as alleged by the applicant? 144 Questions are also raised by the applicant with respect to the contri- bution agreement concluded with the ICCRC prior to the registration and publication of the impugned enactments. In passing, this clearly falls within the realm of departmental and ministerial discretion. Indeed, a similar type of agreement had been concluded with the CSIC in 2003 prior to the registration and publication of the 2004 Regulations. The fact that CIC’s cost benefit analysis presume without any basis that CAPIC/ICCRC would assume CSIC’s infrastructure, staff and services is completely irrelevant as far as the legality of the Ministerial Regulations is concerned. 145 As decided by the Supreme Court of Canada, “[t]he independence of the Bar from the state in all its pervasive manifestations is one of the hallmarks of a free society. Consequently, regulation of these members of the law profession by the state must, so far as by human ingenuity it can be so designed, be free from state interference, in the political sense, with the delivery of services to the individual citizens in the state, partic- ularly in fields of public and criminal law” (Canada (Attorney General) v. Law Society (British Columbia), [1982] 2 S.C.R. 307 (S.C.C.) at pages 335-336). A corollary issue raised by the applicant is whether the body regulating the conduct of the immigration consultants should enjoy the Canadian Soc. of Imm. Consultants v. Canada (MCI) Luc Martineau J. 221

same independence the Bars of the provinces enjoy from state interference. 146 In this respect, the applicant notes that according to the ICCRC’s by- laws, three public interest directors should be appointed by CIC. This seems to be in direct correlation with one of the supervisory options dis- cussed by Sussex Circle, the consultants hired in 2009, in ensuring that the new regulatory body acts in the public interest and remains accounta- ble to the Minister. Whether the designation of public interest directors is contrary to the warnings expressed both by the Advisory Committee (2003) and the Selection Committee (2010) that the regulator be at arms- length from the Government, is another side issue that the Court should refrain from examining today. Such consideration has no bearing with respect to the selection of the body chosen by the Minister and it is pref- erable that any challenge on the institutional independence of the ICCRC be disputed and decided in a separate judicial proceeding. 147 As far as the reasons for choosing the ICCRC as the new regulator of immigration consultants, the following rationale is provided in the July RIAS: Focusing on membership, competence and compliance, complaints and investigations, and discipline, the ICCRC has demonstrated that is has the capacity to meet established organizational competencies that serve as selection factors for this process. The ICCRC has also demonstrated an understanding of its public protection role and of the vulnerability of its primary constituency, the would-be-users of Can- ada’s immigration programs. 148 The maintenance of public confidence in the immigration system was a valid consideration and suffices to dispose of the allegations of im- proper purposes or motives. Again, it is debatable whether the applicant can challenge before the Court the policy reasons which led to the desig- nation of the ICCRC as the new regulator of the immigration consultants and it is irrelevant whether the Minister was also motivated by public opinion or other considerations (Begg v. Canada (Minister of Agricul- ture), 2005 FCA 362 (F.C.A.) at para 37). In the long term, both the Minister and the Government will be held accountable to Parliament, and ultimately to the Canadian electorate, for the purported benefits and ef- fectiveness of the impugned enactments, or any failure or drawback flowing from their policy choices. 222 IMMIGRATION LAW REPORTER 3 Imm. L.R. (4th)

VI. Fair and Transparent Process of Selection 149 The applicant also asserts that the doctrine of legitimate expectations applies in principle to delegated legislative powers creating participatory rights. The applicant argues that the Minister has failed to follow the se- lection process as outlined in the Call for Submissions and thus legiti- mately expected by the CSIC, and is therefore estopped from not com- plying with the selection process previously determined in governmental policy guidelines. 150 In Centre hospitalier Mont-Sina¨ı c. Qu´ebec (Ministre de la Sant´e & des Services sociaux), [2001] 2 S.C.R. 281 (S.C.C.), the Supreme Court expressly rejected the argument that the doctrine of legitimate expecta- tions can give rise to legally enforceable substantive rights, and it is de- batable whether, as stated above, subordinate legislation can lawfully be made in breach of categorical and specific assurance of prior consultation (see Canada Assistance Plan, above, and Apotex, above). That said, in Canadian Pacific Railway v. Vancouver (City), [2006] 1 S.C.R. 227 (S.C.C.), the Supreme Court decided that a decision-maker might have to treat a legitimate expectation as a factor that had to be taken into account in the exercise of a discretionary power. 151 In any event, the Court finds that there has been no breach of the applicant’s legitimate expectation and that this is not “an egregious case” where the intervention of the Court is warranted to uphold the rule of law. As far as any duty to consult is concerned, it has been satisfied in this case. 152 To that effect, a Selection Committee (comprised of four external ex- perts and three senior public servants) was charged with examining the submissions received in response to the Call for Submissions and making recommendations to the Minister after having considered the submis- sions in light of the selection factors and “other relevant factors”. While the Selection Committee was asked to examine the submissions in light of five selection factors, namely competence, integrity, accountability, viability, and good governance, it was also made clear that “this Call for Submissions does not obligate the Minister, the Department of Citizen- ship and Immigration or the Government of Canada in any way, or to take any action”. 153 Four submissions were considered by the Selection Committee, in- cluding a proposal made by the applicant to continue being the regulator. In a report delivered to the Minister’s attention on January 27, 2011, the Selection Committee came to the conclusion that the ICCIP (later incor- Canadian Soc. of Imm. Consultants v. Canada (MCI) Luc Martineau J. 223

porated under the name ICCRC on February 18, 2011) and the applicant both met the previously announced selection factors. However, the Com- mittee further observed that the applicant “missed the opportunity to demonstrate how the CSIC would address areas of concern that were ex- pressed by the Standing Committee on Citizenship and Immigration in their report of June 2008; and that gave impetus to the Notice in Canada Gazette” while the ICCIP proponents “made a concerted effort to demon- strate how the ICCIP would fully address areas of concern that were ex- pressed by the Standing Committee”. These were certainly valid consid- erations in the Court’s opinion. 154 On February 7, 2011, the Minister was provided a briefing memoran- dum from the Deputy Minister, which recommended proceeding with a proposal to the Governor in Council that the Regulations be amended so as to recognize the ICCRC as the new regulatory body for immigration consultants. Another option (not recommended) was to maintain the sta- tus quo by keeping the CSIC as the regulatory body. The Deputy Min- ister further noted that the Minister, being the final decision-maker, was also entitled to take into consideration relevant and valid factors other than those previously considered and assessed by the Selection Commit- tee or the Parliamentary Standing Committee. The Court is in agreement. 155 With respect to bias, the applicant refers to Ms. Sandra Harder, the Minister’s Acting Director General, stating in the Notice of intent dated June 12, 2010, that the Parliamentary Standing Committee’s “report, sup- ported by a 2009 report titled Migrant Workers and Ghost Consultants, points to the lack of public confidence in the body currently governing immigration consultants. A lack of public confidence poses a significant threat to the immigration system, given the regulator’s role with respect to the integrity of the system as whole.” The applicant takes issue with the fact that notwithstanding a clearly biased opinion against the CSIC, Ms. Harder was later appointed to sit on the Selection Committee. 156 The Court finds that a person who is well informed would not come to the conclusion that a reasonable apprehension of bias on the part of Ms. Harder existed. In the Notice of intent, Ms. Harder simply stated that there was evidence in the Standing Committee report that invoked a lack of public confidence in the regulator and that such lack of public confi- dence would pose a serious threat to the immigration system. Perhaps CIC could have attempted to ascertain whether the complaints about the applicant were valid but for policy reasons it was determined not to enter 224 IMMIGRATION LAW REPORTER 3 Imm. L.R. (4th)

into a direct oversight relationship with the regulatory body and to pro- ceed with its replacement. 157 The applicant raises a number of other irrelevant issues as far as the legality of the impugned enactments is concerned. For example, the ap- plicant refers to a National Post article, published on May 26, 2011 and titled “cleaning the sleaze out of immigration consulting”, arguing that CIC’s posting of this article on its website, and its refusal to remove it despite CSIC’s request, raises a reasonable apprehension of bias. How- ever, according to the evidence, no content from the National Post article was published on the CIC website. Rather, it appeared in a section con- taining links related to Bill C-35, where numerous other articles and sto- ries from different journals and websites appeared as well. 158 In the final analysis, the Court finds that the selection of a single reg- ulator of immigration consultants undertaken according to merit-based or other selection criteria was a legitimate policy choice based on a dele- gated legislative authority when the Ministerial Regulations were enacted in June 2011. Public materials such as the Standing Committee report could also legitimately be consulted during the selection process. In any event, on several occasions, the applicant had the opportunity to put its position forward and to provide input regarding the policy making pro- cess that led to its replacement. The applicant notably appeared before the Standing Committee, participated in the selection process established by CIC, and responded to the pre-publication of the proposed regulatory amendment. This suffices to dismiss the allegations of breach of proce- dural fairness made by the applicant.

VII. Bill C-35 and Impugned Enactments Validly Enacted 159 As stated by the Federal Court of Appeal in Canadian Council for Refugees v. R., 2008 FCA 229 (F.C.A.) at para 56: An attack on the legality of subordinate legislation, on the ground that the conditions precedent prescribed by Parliament were not met at the time of the promulgation, remains what it has always been; an attack on the impugned regulation per se and not on the “decision” to promulgate it. 160 On March 23, 2011, Bill C-35 received Royal Assent. The amend- ments introduced to section 91 of the Act by section 1 of Bill C-35 have purportedly come into force as a result of the enactment of the Order Fixing June 30, 2011 as the Day on which Chapter 8 of the Statutes of Canada Comes into Force (SI/2011-731) (the GIC Order). Canadian Soc. of Imm. Consultants v. Canada (MCI) Luc Martineau J. 225

161 As mentioned by Professor Ruth Sullivan in her book Sullivan on the Construction of Statutes, 5th ed. (Markham, Ontario: LexisNexis, 2008) at page 644: Legislatures may choose to delay the commencement of legislation for one reason or another: to await events, to allow time to prepare administrative machinery, to give fair warning to the public, to achieve a political goal. In such cases, the time chosen for com- mencement is set out or described in the Act or a power is given to the executive branch, usually the Governor General or Lieutenant Governor in Council, to bring the Act into force on a day within its discretion. 162 The GIC Order sets as June 30, 2011 the coming into force of Bill C- 35 (other than section 6, which came into force on Assent). The GIC Order was effectively made on June 23, 2011 and registered on July 6, 2011. The 2011 Regulations were made and registered on June 23, 2011. The Ministerial Regulations were made and registered on June 27, 2011. 163 Like any other power conferred by Parliament, the power of the Gov- ernor in Council to fix the day on which legislation is to come into force is subject to judicial review (Reference re s. 16 of the Criminal Law Amendment Act, 1968-69 (Canada), [1970] S.C.R. 777 (S.C.C.)). The GIC Order was purportedly taken under the authority of section 7 of Bill C-35 which reads as follows: 7. The provisions of this Act, other than section 6, come into force on a day to be fixed by order of the Governor in Council. 7. Les dispositions de la pr´esente loi, a` l’exception de l’article 6, en- trent en vigueur a` la date fix´ee par d´ecret. 164 The applicant submits that the GIC Order is of no force and effect because it was not registered within seven days after it was made, and thereby fails to comply with section 9 of the SIA which reads as follows: 9. (1) No regulation shall come into force on a day earlier than the day on which it is registered unless (a) it expressly states that it comes into force on a day earlier than that day and is registered within seven days after it is made, or (b) it is a regulation of a class that, pursuant to paragraph 20(b), is exempted from the application of subsection 5(1), in which case it shall come into force, except as otherwise authorized or provided by or under the Act pursuant to which it is made, on the day on which it is made or on such later day as may be stated in the regulation. 226 IMMIGRATION LAW REPORTER 3 Imm. L.R. (4th)

9. (1) L’entr´ee en vigueur d’un r`eglement ne peut pr´ec´eder la date de son enregistrement sauf s’il s’agit: a) d’un r`eglement comportant une disposition a` cet effet et enre- gistr´e dans les sept jours suivant sa prise; b) d’un r`eglement appartenant a` la cat´egorie soustraite a` l’application du paragraphe 5(1) aux termes de l’alin´ea 20b). Sauf autorisation ou disposition contraire figurant dans sa loi habilitante ou edict´´ ee sous le r´egime de celle-ci, il entre alors en vigueur a` la date de sa prise ou a` la date ult´erieure qui y est indiqu´ee. [My underlinings] 165 The respondent answers that the requirements in section 9 of the SIA do not apply to an order of the Governor in Council which simply brings legislation into force because it is not a “regulation”. That said, both the applicant and the respondent agree that the GIC Order constitutes a “stat- utory instrument” within the meaning of section 2 of the SIA: “statutory instrument” (a) means any rule, order, regulation, ordinance, direction, form, tariff of costs or fees, letters patent, commission, warrant, proclamation, by-law, resolution or other instrument issued, made or established (i) in the execution of a power conferred by or under an Act of Parliament, by or under which that instrument is expressly authorized to be issued, made or estab- lished otherwise than by the conferring on any person or body of powers or functions in relation to a matter to which that instrument relates, or (ii) by or under the authority of the Governor in Council, otherwise than in the execution of a power conferred by or under an Act of Parliament, but (b) does not include [...] « texte r´eglementaire » a) R`eglement, d´ecret, ordonnance, proclamation, arrˆet´e, r`egle, r`eglement administratif, r´esolution, instruction ou directive, Canadian Soc. of Imm. Consultants v. Canada (MCI) Luc Martineau J. 227

formulaire, tarif de droits, de frais ou d’honoraires, lettres patentes, commission, mandat ou autre texte pris: (i) soit dans l’exercice d’un pouvoir conf´er´e sous le r´e- gime d’une loi f´ed´erale, avec autorisation expresse de prise du texte et non par simple attribution a` quicon- que — personne ou organisme — de pouvoirs ou fonctions li´es a` une question qui fait l’objet du texte, (ii) soit par le gouverneur en conseil ou sous son autorit´e, mais non dans l’exercice d’un pouvoir conf´er´e sous le r´egime d’une loi f´ed´erale; b) la pr´esente d´efinition exclut: [...] [My underlinings] 166 Thus, the issue is whether the GIC Order falls within the definition of “regulation” found in section 2 of the SIA: “regulation” means a statutory instrument (a) made in the exercise of a legislative power conferred by or under an Act of Parliament, or ... and includes a rule, order or regulation governing the practice or procedure in any proceedings before a judicial or quasi- judicial body established by or under an Act of Parliament, and any instrument described as a regulation in any other Act of Parliament; « r`eglement » Texte r´eglementaire: a) soit pris dans l’exercice d’un pouvoir l´egislatif conf´er´e sous le r´egime d’une loi f´ed´erale; ... Sont en outre vis´es par la pr´esente d´efinition les r`eglements, d´ecrets, ordonnances, arrˆet´es ou r`egles r´egissant la pratique ou la proc´edure dans les instances engag´ees devant un organ- isme judiciaire ou quasi judiciaire constitu´e sous le r´egime d’une loi f´ed´erale, de mˆeme que tout autre texte d´esign´e comme r`eglement par une autre loi f´ed´erale. [My underlinings] 167 The interpretation exposed in the two paragraphs below is the correct one in the Court’s opinion. 168 First, the GIC Order does not establish a “rule of conduct”. Thus, the respondent submits that it cannot be “legislative”. Albeit not rendered in 228 IMMIGRATION LAW REPORTER 3 Imm. L.R. (4th)

the context of the SIA, the respondent relies by analogy on the criteria identified in Reference re Language Rights Under s. 23 of Manitoba Act, 1870 & s. 133 of Constitution Act, 1867, [1992] 1 S.C.R. 212 (S.C.C.) at paras 19-20, to determine whether orders in council were “of a legislative nature” (so that the constitutional bilingualism requirement would ap- ply): the instrument embodies a rule of conduct; the instrument has the force of law; and the instrument applies to an undetermined number of persons. 169 Second, pursuant to paragraph 6(b) of the SIA, the Clerk of the Privy Council shall register “every statutory instrument, other than a regula- tion, that is required by or under any Act of Parliament to be published in the Canada Gazette and is so published.” (My underlinings). Paragraph 11(3)(g) of the Statutory Instruments Regulations, CRC, c 1509, requires that “Orders fixing the day or days on which an Act or any provision thereof shall come into force” be published in Part II of the Canada Ga- zette. Therefore, in order to be registered, the GIC Order, as a “statutory instrument, other than a regulation” had to be published first, which was done in this case on July 6, 2011, as submitted by the respondent. 170 Accordingly, the Court finds that contrary to the applicant’s conten- tion, the procedural requirements provided for in the SIA were complied with in the case of the GIC Order made on June 23, 2011, the latter hav- ing been published and accordingly registered on July 6, 2011. 171 Subsidiarily, the applicant argues that even if the GIC Order is valid, the Ministerial Regulations remain invalid as they were made and regis- tered on June 27, 2011; that is three days prior to the date on which Bill C-35, which now grants the Minister statutory authority to make “regula- tions”, came into effect. 172 With respect to the Ministerial Regulations, the respondent submits that section 7 of the Interpretation Act, permits regulation making pow- ers conferred under an Act to be exercised before the enabling provisions of the act come into force, insofar as it is necessary to make “the enact- ment effective on its commencement date”. This is challenged here by the applicant who submits that, as the Minister’s power pursuant to new subsection 91(5) of the Act to designate a new regulator is a discretion- ary one, it is not necessary that the Ministerial Regulations be made prior to the coming into force of the Act to give effect to “the enactment” on its commencement date. Canadian Soc. of Imm. Consultants v. Canada (MCI) Luc Martineau J. 229

173 Section 7 of the Interpretation Act reads as follows: 7. Where an enactment is not in force and it contains provisions con- ferring power to make regulations or do any other thing, that power may, for the purpose of making the enactment effective on its com- mencement, be exercised at any time before its commencement, but a regulation so made or a thing so done has no effect until the com- mencement of the enactment, except in so far as may be necessary to make the enactment effective on its commencement. 7. Le pouvoir d’agir, notamment de prendre un r`eglement, peut s’exercer avant l’entr´ee en vigueur du texte habilitant; dans l’intervalle, il n’est toutefois op´erant que dans la mesure n´ecessaire pour permettre au texte de produire ses effets d`es l’entr´ee en vigueur. [My underlinings] 174 Section 7 of the Interpretation Act obliges the Court to determine whether the power to make regulations was exercised by the Minister prior to the coming into force of section 91 “for the purpose of making the enactment effective on its commencement”. 175 The Court agrees with the respondent that the power given to the Minister by new section 91 of the Act could be exercised prior to the coming into force of the Act for the purpose of making the Ministerial Regulations effective at the commencement date. In fact, a careful read- ing of both the English and French versions of section 7 of the Interpre- tation Act shows that what is intended by the word “enactment” in this section is not necessarily the enabling statute in its entirety, but also the provisions conferring power to make regulations, which includes the pur- ported regulations themselves. 176 While the designation of the ICCRC was certainly a discretionary de- cision of the Minister, it remains that the Ministerial Regulations were made on the purported authority of new subsections 91(5) and (7) of the Act in order to make the designation of the ICCRC and transitional mea- sures applicable to members of the CSIC effective on the coming into force of Bill C-35. Thus, the Court finds that the Ministerial Regulations are authorized by section 7 of the Interpretation Act, and are not other- wise invalid as submitted by the applicant.

VIII. Conclusion 177 For the reasons above, the Court finds that there are no reasons to quash the impugned enactments. Accordingly, the present judicial review application shall be dismissed. The matter of certification of a question 230 IMMIGRATION LAW REPORTER 3 Imm. L.R. (4th) shall be reserved and both parties are invited to submit in writing, within 10 days of the present reasons, any question of general importance they wish to propose to the Court. Any objection or observations with respect of same by the other party can be submitted to the Court in writing within 10 days of filing of same.

Judgment THIS COURT ADJUGES that the present application for judicial re- view is dismissed. THIS COURT FURTHER ADJUGES that the issue of a certified question is reserved pending further submissions from the parties, if any. Both parties are invited to submit in writing, within 10 days of the pre- sent reasons, any question of general importance they wish to propose to the Court. Any objection or observations with respect of same by the other party can be submitted to the Court in writing within 10 days of filing of same. Application dismissed. Mehmi v. Canada (MCI) 231

[Indexed as: Mehmi v. Canada (Minister of Citizenship & Immigration)] Rajwinder Mehmi, Applicant and The Minister of Citizenship and Immigration, Respondent Federal Court Docket: IMM-6206-10 2011 FC 1246 James Russell J. Heard: September 28, 2011 Judgment: November 1, 2011 Immigration and citizenship –––– Admission — Immigrants — Live-in caregivers –––– Applicant was citizen of India — Applicant signed live-in caregiver contract with her employer, who lived in Ontario — Applicant applied to Consulate in Chandigarh for work permit under live-in caregiver program — Consul interviewed applicant to assess her proficiency in English, and informed applicant that interview was to assess her qualifications — Applicant was not able to answer some questions, and sometimes gave answers that were not rele- vant to questions asked — Consul denied applicant’s application — Applicant brought application for judicial review — Application dismissed — There was no evidence that assessment of applicant’s proficiency was either inaccurate or arrived at unfairly — Applicant provided no evidence that standardized test was required to ascertain language proficiency or that it was any better measuring tool than face-to-face oral interview — Onus was on applicant to provide whatever information she thought was necessary to demonstrate that she quali- fied under live-in caregiver program — Applicant might have disagreed with as- sessment, but there was nothing to suggest that it was unreasonable or incorrect. Cases considered by James Russell J.: Al-Kassous v. Canada (Minister of Citizenship & Immigration) (2007), 2007 CarswellNat 1324, 2007 FC 541, 2007 CF 541, [2007] A.C.F. No. 731, [2007] F.C.J. No. 731 (F.C.) — considered Aoanan v. Canada (Minister of Citizenship & Immigration) (2009), 353 F.T.R. 283 (Eng.), 2009 CarswellNat 5783, 2009 CF 734, 2009 CarswellNat 3529, 2009 FC 734, [2009] F.C.J. No. 1395 (F.C.) — referred to Arumugam v. Canada (Minister of Citizenship & Immigration) (2000), 2000 CarswellNat 528, [2000] F.C.J. No. 445 (Fed. T.D.) — referred to De Luna v. Canada (Minister of Citizenship & Immigration) (2010), 2010 Car- swellNat 2763, 90 Imm. L.R. (3d) 67, 2010 CarswellNat 2090, 2010 FC 726, 2010 CF 726, [2010] F.C.J. No. 882 (F.C.) — considered 232 IMMIGRATION LAW REPORTER 3 Imm. L.R. (4th)

Giacca v. Canada (Minister of Citizenship & Immigration) (2001), 200 F.T.R. 107, 2001 CarswellNat 228, 12 Imm. L.R. (3d) 32, [2001] F.C.J. No. 186 (Fed. T.D.) — considered Hajariwala v. Canada (Minister of Employment & Immigration) (1988), [1989] 2 F.C. 79, 34 Admin. L.R. 206, 23 F.T.R. 241, 1988 CarswellNat 74, 1988 CarswellNat 742, 6 Imm. L.R. (2d) 222, [1988] F.C.J. No. 1021 (Fed. T.D.) — referred to Hassani v. Canada (Minister of Citizenship & Immigration) (2006), 2006 CF 1283, 2006 CarswellNat 5123, [2007] 3 F.C.R. 501, 2006 CarswellNat 3387, 2006 FC 1283, 302 F.T.R. 39 (Eng.), [2006] F.C.J. No. 1597 (F.C.) — re- ferred to Jhattu v. Canada (Minister of Citizenship & Immigration) (2005), 2005 FC 853, 2005 CarswellNat 1707, 2005 CF 853, 2005 CarswellNat 5475, [2005] F.C.J. No. 1058 (F.C.) — considered Khosa v. Canada (Minister of Citizenship & Immigration) (2009), 82 Admin. L.R. (4th) 1, 2009 SCC 12, 2009 CarswellNat 434, 2009 CarswellNat 435, 304 D.L.R. (4th) 1, 77 Imm. L.R. (3d) 1, 385 N.R. 206, (sub nom. Canada (Citizenship & Immigration) v. Khosa) [2009] 1 S.C.R. 339, [2009] S.C.J. No. 12 (S.C.C.) — referred to Kumar v. Canada (Minister of Citizenship & Immigration) (2011), 2011 Car- swellNat 2409, 2011 CF 770, 2011 FC 770, 2011 CarswellNat 3463, [2001] F.C.J. No. 970 (F.C.) — considered Ling v. Canada (Minister of Citizenship & Immigration) (1997), 134 F.T.R. 317, 1997 CarswellNat 1068, [1997] F.C.J. No. 1030 (Fed. T.D.) — referred to New Brunswick (Board of Management) v. Dunsmuir (2008), 372 N.R. 1, 69 Admin. L.R. (4th) 1, 69 Imm. L.R. (3d) 1, (sub nom. Dunsmuir v. New Brunswick) [2008] 1 S.C.R. 190, 844 A.P.R. 1, (sub nom. Dunsmuir v. New Brunswick) 2008 C.L.L.C. 220-020, D.T.E. 2008T-223, 329 N.B.R. (2d) 1, (sub nom. Dunsmuir v. New Brunswick) 170 L.A.C. (4th) 1, (sub nom. Dunsmuir v. New Brunswick) 291 D.L.R. (4th) 577, 2008 CarswellNB 124, 2008 CarswellNB 125, 2008 SCC 9, 64 C.C.E.L. (3d) 1, (sub nom. Dunsmuir v. New Brunswick) 95 L.C.R. 65, [2008] S.C.J. No. 9, [2008] A.C.S. No. 9 (S.C.C.) — followed Suresh v. Canada (Minister of Citizenship & Immigration) (2002), 2002 SCC 1, 37 Admin. L.R. (3d) 159, [2002] 1 S.C.R. 3, 2002 CarswellNat 7, 2002 Car- swellNat 8, 18 Imm. L.R. (3d) 1, 208 D.L.R. (4th) 1, 281 N.R. 1, 90 C.R.R. (2d) 1, [2002] S.C.J. No. 3, REJB 2002-27423 (S.C.C.) — referred to Vila v. Canada (Minister of Citizenship & Immigration) (2008), 2008 FC 627, 2008 CarswellNat 1593, 2008 CF 627, 2008 CarswellNat 4531, [2008] F.C.J. No. 823 (F.C.) — considered Statutes considered: Immigration and Refugee Protection Act, S.C. 2001, c. 27 s. 11(1) — considered Mehmi v. Canada (MCI) James Russell J. 233

s. 72(1) — pursuant to Regulations considered: Immigration and Refugee Protection Act, S.C. 2001, c. 27 Immigration and Refugee Protection Regulations, SOR/2002-227 s. 110 — considered s. 111 — considered s. 112 — considered s. 112(d) — considered

APPLICATION for judicial review of decision denying applicant’s application for work permit under live-in caregiver program.

Asiya Hirji, for Applicant Neal Samson, for Respondent

James Russell J.:

1 This is an application pursuant to subsection 72(1) of the Immigration and Refugee Protection Act, S.C. 2001, c. 27 (Act) for judicial review of the decision of the Consul, Immigration (Consul) of the Canadian Consu- late General, Chandigarh, India, dated 22 September 2010 (Decision), which refused the Applicant’s application for a work permit under sec- tion 112 of the Immigration and Refugee Protection Regulations, SOR/2002-227 (Regulations).

Background 2 The Applicant is a citizen of India. On 20 November 2006 she signed a “Live-in Caregiver Contract” with her employer, Rita Mehmi, who lives in Brampton, Ontario. She was to care for a two-year-old child and would be responsible for supervising, bathing, dressing, and feeding the child as well as planning, preparing, and serving meals. In 2006, Service Canada provided a Labour Market Opinion (LMO) on the contract. Hav- ing been provided with the LMO, the Applicant applied to the Consulate in Chandigarh for a work permit under the Live-in Caregiver Program (LCP). The consulate received her application (Application) on 22 Janu- ary 2007. 3 On 22 September 2010, the Consul interviewed the Applicant to as- sess her proficiency in English pursuant to subsection 112(d) of the Reg- ulations. That subsection requires live-in caregivers to have “the ability to speak, read, and listen to English or French at a level sufficient to 234 IMMIGRATION LAW REPORTER 3 Imm. L.R. (4th)

communicate effectively in an unsupervised setting” in order to be granted a work permit. The Consul informed the Applicant that the inter- view was to assess her qualifications. 4 During the interview, the Consul asked several questions, some of which were related to the care of children, some to the Applicant’s per- sonal situation, and some to her education and language training. The Applicant was able to answer some questions but not others and some- times gave answers that were not relevant to the questions asked. The Consul ended the interview when it became apparent to him that the Ap- plicant could not understand the questions put to her. 5 Also on 22 September 2010, the Consul reviewed the Application in light of the Interview and the documents the Applicant had provided, in- cluding a report of her grades from Guru Nanak Dev University. He de- cided that the Applicant did not fulfill the requirements of subsection 112(d) and denied the Application. She was notified of the Decision by a letter dated 22 September 2010.

Decision Under Review 6 The Decision under review in this application consists of the letter provided to the Applicant and the Consul’s CAIPS notes. 7 The Consul was not satisfied that the Applicant was sufficiently pro- ficient in English to carry out the duties of the position. He noted that the Applicant had scored 35/100 in her studies in English at Guru Nanak Dev University and that she said she had studied English for three months and was still taking lessons. He also found that the Applicant’s speaking ability at the Interview was poor. The Consul denied the Appli- cation because he was not satisfied that the Applicant could “care for small children in [an] unsupervised environment and ensure their safety” as required under subsection 112(d) of the Regulations.

Issues 8 The Applicant raises the following issues in this application: a. Whether her right to procedural fairness was breached by the Con- sul’s use of an interview to assess her language skills; b. Whether the Consul’s determination that she was not sufficiently proficient in English was reasonable Mehmi v. Canada (MCI) James Russell J. 235

Statutory Provisions 9 The following provision of the Act is applicable in these proceedings: 11. (1) A foreign national must, before entering Canada, apply to an officer for a visa or for any other document required by the regula- tions. The visa or document may be issued if, following an examina- tion, the officer is satisfied that the foreign national is not inadmissi- ble and meets the requirements of this Act. 11. (1) L’´etranger doit, pr´ealablement a` son entr´ee au Canada, de- mander a` l’agent les visa et autres documents requis par r`eglement. L’agent peut les d´elivrer sur preuve, a` la suite d’un contrˆole, que l’´etranger n’est pas interdit de territoire et se conforme a` la pr´esente loi. 10 The following provisions of the Regulations are also applicable in these proceedings: Live-in caregiver class 110. The live-in caregiver class is prescribed as a class of foreign nationals who may become permanent residents on the basis of the requirements of this Division. Processing 111. A foreign national who seeks to enter Canada as a live-in caregiver must make an application for a work permit in accordance with Part 11 and apply for a temporary resident visa if such a visa is required by Part 9. Work permits — requirements 112. A work permit shall not be issued to a foreign national who seeks to enter Canada as a live-in caregiver unless they (d) have the ability to speak, read and listen to English or French at a level sufficient to communicate effectively in an unsupervised setting; Cat´egorie des aides familiaux 110. La cat´egorie des aides familiaux est une cat´egorie r´eglementaire d’´etrangers qui peuvent devenir r´esidents permanents, sur le fonde- ment des exigences pr´evues a` la pr´esente section. Traitement 111. L’´etranger qui cherche a` entrer au Canada a` titre d’aide familial fait une demande de permis de travail conform´ement a` la partie 11, ainsi qu’une demande de visa de r´esident temporaire si ce visa est requis par la partie 9. 236 IMMIGRATION LAW REPORTER 3 Imm. L.R. (4th)

Permis de travail: Exigences 112. Le permis de travail ne peut etreˆ d´elivr´e a` l’´etranger qui cherche a` entrer au Canada au titre de la cat´egorie des aides familiaux que si l’´etranger se conforme aux exigences suivantes: d) il peut parler, lire et ecouter´ l’anglais ou le fran¸cais suffisamment pour communiquer de fa¸con efficace dans une situation non supervis´ee;

Standard of Review 11 The Supreme Court of Canada in New Brunswick (Board of Management) v. Dunsmuir, 2008 SCC 9, [2008] S.C.J. No. 9 (S.C.C.), held that a standard of review analysis need not be conducted in every instance. Instead, where the standard of review applicable to a particular question before the court is well-settled by past jurisprudence, the re- viewing court may adopt that standard of review. Only where this search proves fruitless must the reviewing court undertake a consideration of the four factors comprising the standard of review analysis. 12 In De Luna v. Canada (Minister of Citizenship & Immigration), 2010 FC 726, [2010] F.C.J. No. 882 (F.C.), Justice Leonard Mandamin consid- ered whether the applicant’s right to procedural fairness was breached by the use of a Speaking Proficiency in English Assessment Knowledge (SPEAK) test rather than an interview to assess her language abilities. He concluded that it was not. In Vila v. Canada (Minister of Citizenship & Immigration), 2008 FC 627, [2008] F.C.J. No. 823 (F.C.), Justice John O’Keefe found that an applicant’s right to procedural fairness was breached when the officer evaluating her application under the LCP failed to consider written documentation on the applicant’s English profi- ciency. Further, in Giacca v. Canada (Minister of Citizenship & Immi- gration), [2001] F.C.J. No. 186, 200 F.T.R. 107 (Fed. T.D.), Justice San- dra Simpson found that there was a breach of natural justice when the speakers in a language testing booth malfunctioned during an interview to assess the applicant’s language abilities. These cases show that the method of evaluating applicants language abilities is an issue of procedu- ral fairness (see also Kumar v. Canada (Minister of Citizenship & Immi- gration), 2011 FC 770, [2001] F.C.J. No. 970 (F.C.) and Hassani v. Canada (Minister of Citizenship & Immigration), 2006 FC 1283, [2006] F.C.J. No. 1597 (F.C.) at paragraphs 28 and 34.) Questions of procedural fairness are evaluated on the standard of correctness (Khosa v. Canada (Minister of Citizenship & Immigration), 2009 SCC 12, [2009] 1 S.C.R. Mehmi v. Canada (MCI) James Russell J. 237

339 (S.C.C.)). The standard of review with respect to the first issue in this case is correctness. 13 In Dunsmuir, above, the Supreme Court of Canada held at paragraph 50: When applying the correctness standard, a reviewing court will not show deference to the decision maker’s reasoning process; it will rather undertake its own analysis of the question. The analysis will bring the court to decide whether it agrees with the determination of the decision maker; if not, the court will substitute its own view and provide the correct answer. From the outset, the court must ask whether the tribunal’s decision was correct. 14 With respect to the second issue, in Kumar, above, Justice David Near found that an officer’s decision on the language skills of an appli- cant for permanent residence as a skilled worker was reasonableness. Us- ing the pre-Dunsmuir pragmatic and functional approach, Justice Max Teitelbaum found in Al-Kassous v. Canada (Minister of Citizenship & Immigration), 2007 FC 541, [2007] F.C.J. No. 731 (F.C.) that the stan- dard of review applicable to an officer’s assessment of an applicant’s language skills was reasonableness simpliciter. Further, in Jhattu v. Canada (Minister of Citizenship & Immigration), 2005 FC 853, [2005] F.C.J. No. 1058 (F.C.), Justice O’Keefe found that the standard of review with respect to an officer’s decision on a work-permit application was reasonableness simpliciter. The Applicant in this case challenges the Consul’s assessment of her language skills, so the standard of review with respect to the second issue is reasonableness. 15 When reviewing a decision on the standard of reasonableness, the analysis will be concerned with “the existence of justification, trans- parency and intelligibility within the decisionmaking process [and also with] whether the decision falls within a range of possible, acceptable outcomes which are defensible in respect of the facts and law.” See Dun- smuir, above, at paragraph 47, and Khosa, above, at paragraph 59. Put another way, the Court should intervene only if the Decision was unrea- sonable in the sense that it falls outside the “range of possible, acceptable outcomes which are defensible in respect of the facts and law.” 238 IMMIGRATION LAW REPORTER 3 Imm. L.R. (4th)

Arguments The Applicant The Officer Breached the Applicant’s Right to Procedural Fairness 16 The Applicant argues that the Consul breached her right to procedural fairness by relying only on an interview to assess her language skills. This breached her right to procedural fairness because oral interviews are subjective, unscientific, and unreliable. Since an oral interview is unrelia- ble, it is a breach of procedural fairness per se. 17 The Applicant notes that CIC’s OP-6 Federal Skilled Workers manual for applications under the Federal Skilled Workers Program directs that officers should use objective measures to test language abilities. Al- though the OP-14 Processing Applicants for the Live-in Caregiver Pro- gram manual directs that officers should use an interview to assess lan- guage skills when they are unsure about an applicant’s ability, the Applicant says that she should have been able to participate in a stan- dardized language test, such as the SPEAK test. The Applicant also notes that office specific guidelines for the Canadian Embassy in Manila direct that applicants under the LCP should take the IELTS, a standardized En- glish test. The Applicant was denied procedural fairness because she was not given the chance to take a standardized test. 18 As noted above, Justice Mandamin held in de Luna that the applicant in that case was not denied procedural fairness because she had been able to take a SPEAK test. The Applicant says this means that her right to procedural fairness was breached because she only had an oral interview and not a standardized test.

The Consul’s Decision was Unreasonable 19 The Applicant also argues that the Consul’s decision that she did not have the required language skills was unreasonable because it was based on irrelevant evidence and a standard that was too high. She says that the standard applied in the language evaluation should be calibrated by the purpose of section 112 of the Regulations. The purpose of that section is to ensure that live-in caregivers have the skills required to care for people in Canada. Since the Applicant will be caring for children, the assess- ment of her language skills should have been carried out with the activi- ties involved in caring for children in mind. When the Consul assessed her language ability on the basis of irrelevant questions, such as her mar- Mehmi v. Canada (MCI) James Russell J. 239

ital status, where she learned to speak English, and where she was living at the time, he committed an error. 20 The Applicant says that her language ability should have been as- sessed as more than sufficient because she correctly answered ten of the eleven questions that were related to childcare. The questions she was unable to answer correctly were unrelated to her ability to care for chil- dren, so they should not have entered the Consul’s assessment. By re- quiring the Applicant to answer unrelated questions correctly, the Consul set a standard that was too high. The Decision should be set aside as unreasonable because the assessment of the Applicant’s language ability was on a standard that was too high in relation to the purpose behind section 112.

The Respondent There was no Breach of the Applicant’s Right to Procedural Fairness 21 The Respondent says that the Applicant’s right to procedural fairness was not breached by the Consul’s reliance on an oral interview. The OP- 6 Federal Skilled Worker manual is not applicable to applications under the LCP. Although a standardized test is an appropriate way to assess language ability for the Federal Skilled Worker Program, this does not mean it is an appropriate way to assess language ability under the LCP. 22 The Respondent notes that the OP-14 Processing Applicants for the Live-in Caregiver Program manual clearly says that there should be an interview to assess language abilities where needed. Where the Minister has chosen the means of assessment — in the case of the LCP, an inter- view when needed — the Court should not interfere with that choice. 23 According to the Respondent, de Luna, above, stands only for the proposition that the SPEAK test is one way of ensuring procedural fair- ness in the assessment of language proficiency. In that case, the issue was whether the applicant had an opportunity to respond to the deci- sionmaker’s concerns about her language ability. Justice Mandamin held that the SPEAK test gave her that opportunity. In this case, the Applicant was present in the interview room with the Consul and had the chance to address any concerns he might have had. She had the opportunity to sug- gest an alternate testing method at that time, but did not take it. Further, the oral interview fairly assessed her English proficiency. The Decision should stand because there was no breach of procedural fairness. 240 IMMIGRATION LAW REPORTER 3 Imm. L.R. (4th)

The Conclusion That the Applicant was Not Sufficiently Proficient was Reasonable 24 The Consul’s conclusion that the Applicant did not meet the require- ment of subsection 112(d) was reasonable, even though she disagrees with it. This conclusion is one of fact and deserves deference. The Appli- cant gave a number of incorrect responses to the Consul’s questions in the Interview and it was reasonable for him to conclude as he did based on those responses. 25 Since the decision was reasonable and there was no breach of proce- dural fairness, the Consul’s Decision should stand.

Analysis 26 There is no evidence before the Court that the Consul’s assessment of the Applicant’s proficiency was either inaccurate or arrived at unfairly. The Applicant’s position is that the Consul’s failure to invite or require the Applicant to take a standardized test was, per se, unfair. She points to the practices adopted in Manila, and the uses made there of the SPEAK test. 27 It seems to me that, in some situations, a standardized test may well be required in order to ensure procedural fairness, but I cannot say that this will always be the case. The Applicant has provided no evidence that a standardized test is required to ascertain language proficiency in all instances and/or that it is any better as a measuring tool than a face-to- face oral interview. It is difficult for the Court to assess whether it would have made any difference in this case because the Applicant has pro- vided no evidence that the Consul’s assessment was inaccurate or that she was not given a fair opportunity to demonstrate her language skills. She merely wants the Court to decide in an abstract way that procedural unfairness occurred because the Consul relied upon her own interview to assess the Applicant’s proficiency in English. 28 The onus was on the Applicant to provide whatever information she thought was necessary to convince the Consul that she qualified under the live-in-caregiver program. See Arumugam v. Canada (Minister of Citizenship & Immigration), [2000] F.C.J. No. 445 (Fed. T.D.) at para- graph 29; Ling v. Canada (Minister of Citizenship & Immigration), [1997] F.C.J. No. 1030 (Fed. T.D.) at paragraph 5; and Hajariwala v. Canada (Minister of Employment & Immigration) (1988), [1989] 2 F.C. 79, [1988] F.C.J. No. 1021 (Fed. T.D.). The Consul felt he could not Mehmi v. Canada (MCI) James Russell J. 241

proceed with the interview “without providing possible answers with in the question itself” because the Applicant’s English was so poor. 29 There is simply nothing before the Court to suggest that, on the facts of this case, the Applicant was not given a fair and full opportunity to demonstrate the level of her proficiency in English or that the Consul got it wrong. Presumably, if the Applicant felt she had not been given a fair opportunity, she would have provided evidence to that effect to the Court. 30 There is no statutory or case-law authority to my knowledge which says that procedural fairness requires, in every case, a standardized lan- guage test. In some instances, it would be pointless. OP-14 directs in these situations that “If an officer has reason to doubt an applicant’s lan- guage ability, then the officer should interview the applicant.” This ap- pears to recognize that a face-to-face interview is an acceptable proce- dure in these circumstances. I do not say that it will suffice in all circumstances. A standardized test may be the only fair way of assessing the ability of some applicants. Much will depend upon the circumstances of each case and whether the interview has provided a particular appli- cant with a fair opportunity to demonstrate their proficiency and the of- ficers ability to make an assessment from what transpires at the inter- view. I have no evidence of procedural unfairness on the facts of this case. The Consul felt that the language problem was so bad that she could not conduct a meaningful interview. The Applicant does not say that the Officer was wrong in this regard. 31 On the Applicant’s second point, it is clear to me that the Consul does conduct a purposive interview and analysis. As his conclusions make clear, he focused on ascertaining whether the Applicant would be able to care for small children in an unsupervised environment and ensure their safety. He asked general questions to test the Applicant’s general ability to communicate in English, as well as more specific questions about car- ing for children. As OP-14 makes clear, applicants must be able to do more than deal with the internal situation. They must also be able to: • Respond to emergency situations by contacting a doctor, ambu- lance, police or fire department; • Read the labels on medication; • Answer the telephone and the door; and • Communicate with others outside the home, such as schools, stores or other institutions. 242 IMMIGRATION LAW REPORTER 3 Imm. L.R. (4th)

32 The Applicant may disagree with the Consul’s assessment, but there is nothing before the Court to suggest it is either unreasonable or incor- rect. The Court cannot re-weigh evidence and must allow officers to ex- ercise the discretion that Parliament has allocated to them. See Suresh v. Canada (Minister of Citizenship & Immigration), 2002 SCC 1, [2002] S.C.J. No. 3 (S.C.C.) at paragraphs 29, 34, and 37 and Aoanan v. Canada (Minister of Citizenship & Immigration), 2009 FC 734 (F.C.) at para- graph 42. 33 Counsel agree that there is no question for certification and the Court concurs.

Judgment THIS COURT’S JUDGMENT is that 1. The application is dismissed. 2. There is no question for certification. Application dismissed. Zingano v. Canada (MCI) 243

[Indexed as: Zingano v. Canada (Minister of Citizenship & Immigration)] Salim Tafadzwa Zingano, Applicant and The Minister of Citizenship and Immigration, Respondent Federal Court Docket: IMM-626-11 2011 FC 1243 James Russell J. Heard: September 28, 2011 Judgment: November 1, 2011 Immigration and citizenship –––– Refugee protection — Practice and proce- dure in refugee claims — Post-determination options — Humanitarian and compassionate review — Family in Canada –––– Applicant was 18-year-old citizen of Zimbabwe — His sponsor and litigation guardian was his father, who was Canadian citizen — Applicant currently lived in Zimbabwe with paternal grandmother — . In 2008, sponsor again applied for permanent resident status on behalf of applicant — When application was denied, again because of opera- tion of s. 117(9)(d) of Immigration and Refugee Protection Regulations, appli- cant requested H&C exemption under s. 25(1) of Immigration and Refugee Pro- tection Act — Immigration officer assessed H&C application and refused application for exemption based on conclusion that H&C considerations were not sufficiently compelling to justify granting applicant exemption — Applicant then applied for judicial review of decision — Application granted — Decision was quashed and matter was returned for reconsideration by different officer — Officer provided no basis for conclusion that situation in Zimbabwe had im- proved and she appeared to be unaware of evidence before her that revealed real situation in Zimbabwe — Incorrect analysis of situation was highly material er- ror that rendered decision unreasonable — Officer would not have come to same conclusion regarding best interests of applicant if she had taken into account what evidence said about declining conditions in Zimbabwe and prospects for applicant if he had to remain there — Matter required reconsideration. Immigration and citizenship –––– Refugee protection — Practice and proce- dure in refugee claims — Judicial review — Leave to apply –––– Applicant was 18-year-old citizen of Zimbabwe — His sponsor and litigation guardian was his father, who was Canadian citizen — Applicant currently lived in Zimbabwe with paternal grandmother — In 2008, sponsor again applied for permanent resi- dent status on behalf of applicant — When application was denied, again be- cause of operation of s. 117(9)(d) of Immigration and Refugee Protection Regu- 244 IMMIGRATION LAW REPORTER 3 Imm. L.R. (4th)

lations, applicant requested H&C exemption under s. 25(1) of Immigration and Refugee Protection Act — Immigration officer assessed H&C application and refused application for exemption based on conclusion that H&C considerations were not sufficiently compelling to justify granting applicant exemption — Ap- plicant then applied for judicial review of decision — Application granted — Decision was quashed and matter was returned for reconsideration by different officer — Officer provided no basis for conclusion that situation in Zimbabwe had improved and she appeared to be unaware of evidence before her that re- vealed real situation in Zimbabwe — Incorrect analysis of situation was highly material error that rendered decision unreasonable — Officer would not have come to same conclusion regarding best interests of applicant if she had taken into account what evidence said about declining conditions in Zimbabwe and prospects for applicant if he had to remain there — Matter required reconsideration. Cases considered by James Russell J.: Adil v. Canada (Minister of Citizenship & Immigration) (2010), 92 Imm. L.R. (3d) 140, 2010 CarswellNat 5174, 2010 CF 987, 377 F.T.R. 113 (Eng.), 2010 FC 987, 2010 CarswellNat 3642, [2010] F.C.J. No. 1228 (F.C.) — re- ferred to Baker v. Canada (Minister of Citizenship & Immigration) (1999), 1 Imm. L.R. (3d) 1, [1999] 2 S.C.R. 817, 14 Admin. L.R. (3d) 173, 174 D.L.R. (4th) 193, 1999 CarswellNat 1124, 1999 CarswellNat 1125, 243 N.R. 22, [1999] S.C.J. No. 39 (S.C.C.) — considered C.U.P.E. v. Ontario (Minister of Labour) (2003), 2003 CarswellOnt 1803, 2003 SCC 29, 2003 CarswellOnt 1770, 2003 C.L.L.C. 220-040, [2003] 1 S.C.R. 539, (sub nom. Canadian Union of Public Employees v. Ontario (Minister of Labour)) 173 O.A.C. 38, (sub nom. Canadian Union of Public Employees v. Ontario (Minister of Labour)) 66 O.R. (3d) 735 (note), 226 D.L.R. (4th) 193, (sub nom. Canadian Union of Public Employees v. Ontario (Minister of Labour)) 304 N.R. 76, 50 Admin. L.R. (3d) 1, REJB 2003-41592, [2003] S.C.J. No. 28 (S.C.C.) — considered David v. Canada (Minister of Citizenship & Immigration) (2007), 2007 CF 546, 2007 CarswellNat 3454, 2007 CarswellNat 1385, 2007 FC 546, [2007] F.C.J. No. 740, [2007] A.C.F. No. 740 (F.C.) — referred to de Guzman v. Canada (Minister of Citizenship & Immigration) (2005), 42 Ad- min. L.R. (4th) 234, 2005 FCA 436, 2005 CarswellNat 4381, 51 Imm. L.R. (3d) 17, 262 D.L.R. (4th) 13, 137 C.R.R. (2d) 20, [2006] 3 F.C.R. 655, 345 N.R. 73, 2005 CarswellNat 6009, 139 C.R.R. (2d) 376 (note), [2005] F.C.J. No. 2119 (F.C.A.) — considered Del Cid v. Canada (Minister of Citizenship & Immigration) (2006), 2006 FC 326, 2006 CarswellNat 601, 2006 CF 326, 2006 CarswellNat 3336, [2006] F.C.J. No. 416 (F.C.) — considered Zingano v. Canada (MCI) 245

Ebonka v. Canada (Minister of Citizenship & Immigration) (2009), 2009 CF 80, 2009 CarswellNat 4526, 2009 FC 80, 2009 CarswellNat 255 (F.C.) — considered Hassani v. Canada (Minister of Citizenship & Immigration) (2006), 2006 CF 1283, 2006 CarswellNat 5123, [2007] 3 F.C.R. 501, 2006 CarswellNat 3387, 2006 FC 1283, 302 F.T.R. 39 (Eng.), [2006] F.C.J. No. 1597 (F.C.) — followed Hurtado v. Canada (Minister of Citizenship & Immigration) (2007), 2007 CF 552, 2007 CarswellNat 1387, 2007 FC 552, 2007 CarswellNat 5644 (F.C.) — referred to Khosa v. Canada (Minister of Citizenship & Immigration) (2009), 82 Admin. L.R. (4th) 1, 2009 SCC 12, 2009 CarswellNat 434, 2009 CarswellNat 435, 304 D.L.R. (4th) 1, 77 Imm. L.R. (3d) 1, 385 N.R. 206, (sub nom. Canada (Citizenship & Immigration) v. Khosa) [2009] 1 S.C.R. 339, [2009] S.C.J. No. 12 (S.C.C.) — considered Kisana v. Canada (Minister of Citizenship & Immigration) (2009), 2009 FCA 189, 2009 CarswellNat 1626, 2009 CAF 189, 392 N.R. 163, 2009 Car- swellNat 4008, [2010] 1 F.C.R. 360, [2009] F.C.J. No. 713 (F.C.A.) — considered Krauchanka v. Canada (Minister of Citizenship & Immigration) (2010), 2010 FC 209, 2010 CarswellNat 428, 2010 CarswellNat 1672, 2010 CF 209, [2010] F.C.J. No. 245 (F.C.) — referred to Lee v. Canada (Minister of Citizenship & Immigration) (2005), 45 Imm. L.R. (3d) 129, 2005 CarswellNat 783, 2005 FC 413, 2005 CarswellNat 2020, 2005 CF 413, [2005] F.C.J. No. 507 (F.C.) — referred to Li v. Canada (Minister of Citizenship & Immigration) (2006), 2006 FC 1292, 2006 CarswellNat 3474, 2006 CF 1292, 2006 CarswellNat 6510, [2006] F.C.J. No. 1613, [2006] A.C.F. No. 1613 (F.C.) — considered Lupsa c. Canada (Ministre de la Citoyennet´e & de l’Immigration) (2009), 2009 CF 1054, 2009 CarswellNat 5128, 2009 FC 1054, 2009 CarswellNat 3180 (F.C.) — considered Maldonado v. Canada (Minister of Employment & Immigration) (1979), [1980] 2 F.C. 302, 1979 CarswellNat 168, 1979 CarswellNat 168F, 31 N.R. 34, [1979] F.C.J. No. 248 (Fed. C.A.) — considered Malveda v. Canada (Minister of Citizenship & Immigration) (2008), 2008 Car- swellNat 849, 2008 FC 447, 2008 CF 447, 2008 CarswellNat 2124, 71 Imm. L.R. (3d) 224, [2008] F.C.J. No. 527 (F.C.) — referred to New Brunswick (Board of Management) v. Dunsmuir (2008), 372 N.R. 1, 69 Admin. L.R. (4th) 1, 69 Imm. L.R. (3d) 1, (sub nom. Dunsmuir v. New Brunswick) [2008] 1 S.C.R. 190, 844 A.P.R. 1, (sub nom. Dunsmuir v. New Brunswick) 2008 C.L.L.C. 220-020, D.T.E. 2008T-223, 329 N.B.R. (2d) 1, (sub nom. Dunsmuir v. New Brunswick) 170 L.A.C. (4th) 1, (sub nom. Dunsmuir v. New Brunswick) 291 D.L.R. (4th) 577, 2008 CarswellNB 124, 246 IMMIGRATION LAW REPORTER 3 Imm. L.R. (4th)

2008 CarswellNB 125, 2008 SCC 9, 64 C.C.E.L. (3d) 1, (sub nom. Dunsmuir v. New Brunswick) 95 L.C.R. 65, [2008] S.C.J. No. 9, [2008] A.C.S. No. 9 (S.C.C.) — followed Owusu v. Canada (Minister of Citizenship & Immigration) (2003), [2003] 3 F.C. 172, 2003 FCT 94, 2003 CFPI 94, 2003 CarswellNat 1577, 228 F.T.R. 19, 2003 CarswellNat 225, 27 Imm. L.R. (3d) 114, [2003] F.C.J. No. 139 (Fed. T.D.) — referred to Owusu v. Canada (Minister of Citizenship & Immigration) (2004), 2004 FCA 38, 2004 CarswellNat 248, 2004 CAF 38, 2004 CarswellNat 1117, 318 N.R. 300, (sub nom. Owusu v. Canada) [2004] 2 F.C.R. 635, [2004] F.C.J. No. 158 (F.C.A.) — considered Ponniah v. Canada (Minister of Citizenship & Immigration) (2003), 2003 FC 1016, 2003 CarswellNat 2575, 2003 CarswellNat 4436, 2003 CF 1016, 35 Imm. L.R. (3d) 88 (F.C.) — considered Rafieyan v. Canada (Minister of Citizenship & Immigration) (2007), 2007 FC 727, 2007 CarswellNat 1921, 2007 CF 727, 2007 CarswellNat 4311, [2007] F.C.J. No. 974 (F.C.) — referred to Sandhu v. Canada (Minister of Citizenship & Immigration) (2007), 2007 Car- swellNat 5778, 2007 CF 156, 2007 FC 156, 2007 CarswellNat 302, 309 F.T.R. 243 (Eng.), [2007] F.C.J. No. 204 (F.C.) — referred to Sketchley v. Canada (Attorney General) (2005), 2006 C.L.L.C. 230-002, 2005 CAF 404, 2005 CarswellNat 5119, [2006] 3 F.C.R. 392, 2005 FCA 404, 2005 CarswellNat 4194, 344 N.R. 257, 44 Admin. L.R. (4th) 4, 56 C.H.R.R. D/490, 263 D.L.R. (4th) 113, [2005] F.C.J. No. 2056 (F.C.A.) — considered Sultana v. Canada (Minister of Citizenship & Immigration) (2009), 2009 CF 533, 2009 CarswellNat 4024, 80 Imm. L.R. (3d) 214, 346 F.T.R. 1 (Eng.), [2010] 1 F.C.R. 175, 2009 FC 533, 2009 CarswellNat 1418, [2009] F.C.J. No. 653 (F.C.) — referred to Vasquez v. Canada (Minister of Citizenship & Immigration) (2005), 268 F.T.R. 122 (Eng.), 2005 CF 91, 2005 CarswellNat 4311, 2005 FC 91, 2005 Car- swellNat 89, [2005] F.C.J. No. 96 (F.C.) — considered Yue v. Canada (Minister of Citizenship & Immigration) (2006), 2006 FC 717, 2006 CarswellNat 1600, 2006 CarswellNat 4607, 2006 CF 717, [2006] F.C.J. No. 914 (F.C.) — considered Statutes considered: Immigration and Refugee Protection Act, S.C. 2001, c. 27 s. 3(1)(d) — considered s. 11(1) — considered s. 25 — considered s. 25(1) — considered s. 72(1) — pursuant to Zingano v. Canada (MCI) James Russell J. 247

Regulations considered: Immigration and Refugee Protection Act, S.C. 2001, c. 27 Immigration and Refugee Protection Regulations, SOR/2002-227 s. 116 — considered s. 117(9)(d) — considered

APPLICATION for judicial review by applicant from decision of Immigration officer who refused application for exemption based on conclusion that H&C considerations were not sufficiently compelling to justify granting applicant exemption.

Daniel Kingwell, for Applicant Sally Thomas, for Respondent

James Russell J.:

1 This is an application pursuant to subsection 72(1) of the Immigration and Refugee Protection Act, S.C. 2001, c. 27 (Act) for judicial review of the decision of the Designated Immigration Officer (Officer) at the High Commission of Canada in Pretoria, South Africa, dated 16 November 2010 (Decision). In the Decision, the Officer refused the Applicant’s ap- plication for a humanitarian and compassionate (H&C) exemption from the operation of paragraph 117(9)(d) of the Immigration and Refugee Protection Regulations SOR/2002-227 (Regulations) under subsection 25(1) of the Act and denied the Applicant permanent resident status.

Background 2 The Applicant is an eighteen-year-old citizen of Zimbabwe. His spon- sor and litigation guardian is his father, Lameck Zingano, a Canadian citizen (Sponsor). The Sponsor’s wife, the Applicant’s step-mother, was the Co-sponsor on his application for permanent residence (Cosponsor). The Applicant currently lives in Zimbabwe with his paternal grandmother. 3 In 1999, the Sponsor left Zimbabwe to study in the Netherlands. He met the Co-sponsor online in 2000 and that year they met in person in the United States. They were married in 2001. After the Co-sponsor had her first child by the Sponsor, the Co-sponsor sponsored the Sponsor as a member of the family class. The Sponsor was granted permanent resident status in Canada in 2002 and became a Canadian citizen in September 2005. 248 IMMIGRATION LAW REPORTER 3 Imm. L.R. (4th)

4 The Sponsor did not list the Applicant as his son on his application for permanent residence though the Applicant was nearly ten years old at that time. This would later ground the denial of a Temporary Resident Visa (TRV) and the Permanent Resident Visa. The Applicant was not examined as part of the Sponsor’s permanent residence application in 2002. Since 2003, the Sponsor has sent money to his family members in Zimbabwe. Since 2005, he has made regular phone calls to Zimbabwe to speak with the Applicant. The Sponsor visited Zimbabwe from Decem- ber 2006 to January 2007. 5 In 2007, the Applicant was interviewed by Citizenship and Immigra- tion Canada (CIC) staff in Harare, Zimbabwe in relation to his applica- tion for a TRV. At this time, he said that he had contact information for his biological mother, who was living in Mozambique. 6 In 2006, the Sponsor made his first application to sponsor the Appli- cant as a member of the family class. This application was denied be- cause the Applicant is permanently excluded from the family class by paragraph 117(9)(d) of the Regulations. No appeal of that decision was taken, nor was an application for judicial review filed. The Sponsor ap- plied for a TRV for the Applicant in 2007, which was also denied. In 2008, the Sponsor again applied for permanent resident status on behalf of the Applicant. When this application was denied, again because of the operation of paragraph 117(9)(d), the Applicant requested an H&C ex- emption under subsection 25(1) of the Act. This application was referred to the High Commission in Pretoria for processing. 7 The Officer assessed the H&C application on 16 November 2010. On that date, she refused the application for an exemption based on her con- clusion that the H&C considerations were not sufficiently compelling to justify granting the Applicant an exemption from paragraph 117(9)(d) of the Regulations. The Applicant was notified by letter dated 16 November 2010.

Decision Under Review 8 The Decision in this case consists of the Officer’s letter of 16 Novem- ber 2010 and the CAIPS notes on the file. 9 The Officer first noted that the Applicant was permanently excluded from the family class under paragraph 117(9)(d) of the Regulations be- cause the Sponsor did not declare him on his 2002 Application. The Ap- plicant was permanently excluded “regardless of the reasons why the Sponsor never declared him.” The Officer found that the Sponsor’s ex- Zingano v. Canada (MCI) James Russell J. 249

planation as to why he had not included the Applicant were not credible, though the reasons why he was not included on the 2002 Application did not change the fact that the Applicant was permanently excluded. 10 The Officer denied the H&C exemption under subsection 25(1) be- cause she did not “find the [humanitarian and compassionate] considera- tions put forward on this case sufficiently compelling to justify granting [the Applicant] an exemption from any applicable criteria or obligation under the Act.” The Sponsor had based his submissions in support of the H&C application on the political instability in Zimbabwe, the lack of ad- equate health care, and the poor educational opportunities available to the Applicant. 11 The Officer found that the Applicant had not demonstrated a suffi- ciently close relationship with the Sponsor to justify an H&C exemption. She noted that the Sponsor had left the Applicant in Zimbabwe in 1999, when the Applicant was only five years old. She also found that the Sponsor had only visited the Applicant once in the ten years since he left Zimbabwe, from December 2006 to January 2007. She was concerned that there were no photos of the Sponsor and the Applicant together dur- ing this visit and she could not be certain that they had actually seen one another at that time. 12 The Officer also found that there was no explanation as to why the Sponsor had waited until 2006 to file the first application for permanent resident status for the Applicant. She noted that the Sponsor had been granted permanent resident status and was thus able to sponsor the Ap- plicant in 2002. She also found that the remittances the Sponsor sent to Zimbabwe beginning in May 2003 were small. 13 The Officer also found that the Sponsor’s family in Canada had not met the Applicant, nor had they made any effort to do so. Although, in the experience of the Officer, many other Zimbabweans had travelled to neighbouring countries to meet family from abroad — being driven to do so by the political situation in Zimbabwe — the Sponsor’s family had not done so. In the mind of the Officer, there was no excuse for the Canadian family not to have met the Applicant in person. 14 The Officer found that, though the Sponsor said that the Applicant’s biological mother was not available to support him, there was no evi- dence to show this. She noted that the Applicant had provided contact information for his biological mother when he was interviewed in rela- tion to his TRV application in 2007. 250 IMMIGRATION LAW REPORTER 3 Imm. L.R. (4th)

15 Finally, the Officer found that the best interests of the Applicant favoured his remaining in Zimbabwe with his paternal grandmother. The Applicant had known his grandmother his whole life, so it was better for him to stay with her, than to be with a family in Canada he had never met. Further, though the situation in Zimbabwe was not ideal, the Officer said that it had improved and was not an impediment to the Applicant remaining there in the care of his grandmother.

Issues 16 The Applicant formally raises the following issues: a. Whether the Officer unreasonably emphasized the Sponsor’s non- disclosure of the Applicant in the 2002 Application; b. Whether the Officer’s conclusion that the Sponsor and Applicant did not have a close relationship was reasonable; c. Whether the Officer’s conclusion that the Applicant’s had a suita- ble living situation in Zimbabwe was unreasonable. 17 The Applicant also raises the following issue in his pleadings: a. Whether the Applicant’s right to procedural fairness was breached.

Statutory Provisions 18 The following provisions of the Act are applicable in his written argu- ment: Objectives — immigration 3. (1) The objectives of this Act with respect to immigration are ... (d) to see that families are reunited in Canada; ... Application before entering Canada 11. (1) A foreign national must, before entering Canada, apply to an officer for a visa or for any other document required by the regula- tions. The visa or document may be issued if, following an examina- tion, the officer is satisfied that the foreign national is not inadmissi- ble and meets the requirements of this Act. ... Zingano v. Canada (MCI) James Russell J. 251

Humanitarian and compassionate Considerations — request of foreign national 25. (1) The Minister must, on request of a foreign national in Canada who is inadmissible or who does not meet the requirements of this Act, and may, on request of a foreign national outside Canada, ex- amine the circumstances concerning the foreign national and may grant the foreign national permanent resident status or an exemption from any applicable criteria or obligations of this Act if the Minister is of the opinion that it is justified by humanitarian and compassion- ate considerations relating to the foreign national, taking into account the best interests of a child directly affected. Objet en mati`ere d’immigration 3. (1) En mati`ere d’immigration, la pr´esente loi a pour objet: ... d) de veiller a` la r´eunification des familles au Canada; ... Visa et documents 11. (1) L’´etranger doit, pr´ealablement a` son entr´ee au Canada, de- mander a` l’agent les visa et autres documents requis par r`eglement. L’agent peut les d´elivrer sur preuve, a` la suite d’un contrˆole, que l’´etranger n’est pas interdit de territoire et se conforme a` la pr´esente loi. S´ejour pour motif d’ordre humanitaire a` la demande de l’´etranger ... S´ejour pour motif d’ordre humanitaire a` la demande de l’´etranger 25. (1) Le ministre doit, sur demande d’un etranger´ se trouvant au Canada qui est interdit de territoire ou qui ne se conforme pas a` la pr´esente loi, et peut, sur demande d’un etranger´ se trouvant hors du Canada, etudier´ le cas de cet etranger;´ il peut lui octroyer le statut de r´esident permanent ou lever tout ou partie des crit`eres et obligations applicables, s’il estime que des consid´erations d’ordre humanitaire relatives a` l’´etranger le justifient, compte tenu de l’int´erˆet sup´erieur de l’enfant directement touch´e. 19 The following provisions of the Regulations are applicable in this proceeding: Family class 116. For the purposes of subsection 12(1) of the Act, the family class is hereby prescribed as a class of persons who may become perma- nent residents on the basis of the requirements of this Division. 252 IMMIGRATION LAW REPORTER 3 Imm. L.R. (4th)

Excluded relationships 117. (9) A foreign national shall not be considered a member of the family class by virtue of their relationship to a sponsor if ... (d) subject to subsection (10), the sponsor previously made an appli- cation for permanent residence and became a permanent resident and, at the time of that application, the foreign national was a nonaccom- panying family member of the sponsor and was not examined. Cat´egorie 116. Pour l’application du paragraphe 12(1) de la Loi, la cat´egorie du regroupement familial est une cat´egorie r´eglementaire de personnes qui peuvent devenir r´esidents permanents sur le fondement des ex- igences pr´evues a` la pr´esente section. Regroupement Familial 117. (9) Ne sont pas consid´er´ees comme appartenant a` la cat´egorie du regroupement familial du fait de leur relation avec le r´epondant les personnes suivantes: ... d) sous r´eserve du paragraphe (10), dans le cas o`u le r´epondant est devenu r´esident permanent a` la suite d’une demande a` cet effet, l’´etranger qui, a` l’´epoque o`u cette demande a et´´ e faite, etait´ un mem- bre de la famille du r´epondant n’accompagnant pas ce dernier et n’a pas fait l’objet d’un contrˆole.

Standard of Review 20 The Supreme Court of Canada in New Brunswick (Board of Management) v. Dunsmuir, 2008 SCC 9, [2008] S.C.J. No. 9 (S.C.C.), held that a standard of review analysis need not be conducted in every instance. Instead, where the standard of review applicable to a particular question before the court is well-settled by past jurisprudence, the re- viewing court may adopt that standard of review. Only where this search proves fruitless must the reviewing court undertake a consideration of the four factors comprising the standard of review analysis. 21 In Baker v. Canada (Minister of Citizenship & Immigration), [1999] 2 S.C.R. 817 (S.C.C.) at paragraphs 61 and 62, the Supreme Court of Canada held that the standard of review with respect to H&C determina- tions was reasonableness simpliciter. This approach was followed by the Federal Court of Appeal in Kisana v. Canada (Minister of Citizenship & Immigration), 2009 FCA 189 (F.C.A.). (See also Lee v. Canada Zingano v. Canada (MCI) James Russell J. 253

(Minister of Citizenship & Immigration), 2005 FC 413 (F.C.)). Specifi- cally with respect to the first issue, the Supreme Court of Canada held in Khosa v. Canada (Minister of Citizenship & Immigration), 2009 SCC 12, [2009] 1 S.C.R. 339 (S.C.C.) at paragraph 61 that it is not the function of the reviewing court to re-weigh the evidence before the decision-maker. This approach was followed by Justice Michel Shore in Lupsa c. Canada (Ministre de la Citoyennet´e & de l’Immigration), 2009 FC 1054 (F.C.) at paragraph 4 where he held that “the Court cannot lightly interfere with the manner in which an immigration officer exercises his or her discre- tion and it is not for the Court to re-weigh the relevant fact-driven factors of the case.” As the first three issues deal with the Officer’s discretion on the H&C application, the standard of review with respect to these issues is reasonableness. 22 When reviewing a decision on the standard of reasonableness, the analysis will be concerned with “the existence of justification, trans- parency and intelligibility within the decisionmaking process [and also with] whether the decision falls within a range of possible, acceptable outcomes which are defensible in respect of the facts and law.” See Dun- smuir, above, at paragraph 47, and Khosa, above, at paragraph 59. Put another way, the Court should intervene only if the Decision was unrea- sonable in the sense that it falls outside the “range of possible, acceptable outcomes which are defensible in respect of the facts and law.” 23 With respect to the fourth issue, the Applicant raises both the oppor- tunity to respond and the adequacy of reasons. Both of these issues raise questions of procedural fairness. (See Malveda v. Canada (Minister of Citizenship & Immigration), 2008 FC 447 (F.C.), Rafieyan v. Canada (Minister of Citizenship & Immigration), 2007 FC 727 (F.C.), and Adil v. Canada (Minister of Citizenship & Immigration), 2010 FC 987 (F.C.)). In C.U.P.E. v. Ontario (Minister of Labour), 2003 SCC 29, [2003] 1 S.C.R. 539 (S.C.C.), the Supreme Court of Canada held that the standard of review with respect to questions of procedural fairness is correctness. Further, the Federal Court of Appeal in Sketchley v. Canada (Attorney General), 2005 FCA 404 (F.C.A.) at paragraph 53 held that the “proce- dural fairness element is reviewed as a question of law. No deference is due. The decision-maker has either complied with the content of the duty of fairness appropriate for the particular circumstances, or has breached this duty.” The standard of review with respect to the fourth issue is correctness. 254 IMMIGRATION LAW REPORTER 3 Imm. L.R. (4th)

Arguments The Applicant The Applicant’s Right to Procedural Fairness was Breached The Applicant was Denied the Opportunity to Respond 24 The Applicant argues that the conclusions the Officer reached were unreasonable because they were based on a breach of his right to proce- dural fairness. The Officer failed to ask him for explanations of the evi- dence that he presented or to fill in the holes in evidence on issues she was concerned about. 25 The Applicant relies on Hassani v. Canada (Minister of Citizenship & Immigration), 2006 FC 1283 (F.C.) for the proposition that an officer has a duty to seek clarification where her concerns do not emanate directly from a requirement of the Act. Where an officer does not seek clarifica- tion, as occurred in this case, the Applicant’s right to procedural fairness will be breached through a denial of the right to respond. 26 The CIC manual OP-4 Processing of Applications under section 25 of IRPA states under the heading “The ‘Case to be Met’” that “it is good practice to clarify possible H&C grounds if these are not articulated.” Further, Baker, above, shows that a high level of participatory rights is called for where the interests of a child are at stake. The Applicant says that Del Cid v. Canada (Minister of Citizenship & Immigration), 2006 FC 326 (F.C.) teaches that an officer should request further evidence where he or she perceives a lack of evidence for a submission with re- spect to the best interest of a child. Taken together, these authorities show that the Officer in this case was under a duty to inquire into the areas where she perceived a lack of evidence. 27 The Applicant says he was denied the opportunity to respond in this manner when the Officer relied on the evidence that the Sponsor waited until 2006 before filing an application for permanent residence on his behalf. He says that this posed a question that the Sponsor had not antici- pated and which did not emanate from a requirement of the Act. The Officer’s duty to inquire was therefore engaged. She breached the Appli- cant’s right to respond when she did not ask for clarification of the rea- sons for the delay. Had he been asked to explain the delay, the Sponsor says he would have explained that he received bad advice from an immi- gration consultant. 28 The Applicant’s right to respond was also breached when the Officer failed to put her concerns about the lack of visits to Zimbabwe to the Zingano v. Canada (MCI) James Russell J. 255

Applicant or the Sponsor. Had she done so, the Sponsor was ready and willing to give further evidence of visits. He was precluded from doing so because he did not know that this was something the Officer was con- cerned about. 29 The Officer also failed to put her concerns about the lack of pictures from the Sponsor’s visit in December 2006 — January 2007 to Zimbabwe to the Applicant or the Sponsor. This denied the Applicant the opportunity to respond to the Officer’s concerns. The Officer had a duty to put this to the Applicant as he could not reasonably foresee that the lack of pictures would be a concern. Counsel had advised the Sponsor that the focus of the inquiry was on demonstrating ongoing support and contact, which he had attempted to do through evidence of visits, phone calls, and remittances to family in Zimbabwe. 30 The Officer also failed to put to the Applicant’s concerns about the sufficiency of the amounts remitted by the Sponsor to family members in Zimbabwe, which again denied the Applicant the opportunity to respond. Had the Officer done so, the Applicant says he would have explained that the amounts were actually quite large, given the rate of inflation Zimbabwe was experiencing at the time. Further, he would have shown that Zimbabwe had imposed restrictions on foreign remittances and, to compensate, the Sponsor had bought groceries for the Applicant on-line. He also would have shown that his expenses were low because he lives with his grandmother. The Applicant says the Officer’s concern here was not well-founded and could have been resolved if she had asked the Ap- plicant for an explanation. 31 The Applicant was also denied the opportunity to respond when the Officer failed to put to him her concerns about the lack of a meeting between him and his Canadian step-family. Had she put this concern to him, he would have adduced evidence that the Sponsor was stateless un- til 2006, the airfare for the family was approximately $10,000, and that the family elected to send what resources they had available to the Appli- cant as remittances, rather than spending money on travel. 32 Finally, the Officer failed to put her concerns about the Applicant’s contact with his biological mother to him. Had she done so, he would have explained the situation to the Officer.

The Reasons Given Were Inadequate 33 The Applicant also argues that his right to procedural fairness was breached when the Officer failed to provide adequate reasons. The rea- 256 IMMIGRATION LAW REPORTER 3 Imm. L.R. (4th)

sons were inadequate because they did not disclose how the Officer con- cluded, in the face of evidence of political instability and deprivation in Zimbabwe, that the Applicant’s living situation was adequate. The Of- ficer failed to engage in a meaningful way with the evidence on the con- ditions in Zimbabwe. By not engaging with the evidence, the reasons provided by the Officer fell below the requirement that she be alert, alive, and sensitive to the best interests of the Applicant.

The Decision was Unreasonable. The Officer’s Finding That the Applicant and the Sponsor Did Not Have a Close Relationship was Unreasonable 34 The Applicant argues that the Officer’s conclusion about the relation- ship between him and the Sponsor was unreasonable on several grounds. 35 First, the Officer failed to account for, or was in error concerning the evidence that was before her. The Applicant says that when she analyzed their relationship, the Officer did not take into account the phone calls and letters which had been exchanged between him and the Sponsor. Also, when she looked at the time between when the Sponsor was granted permanent residence and when he first applied for permanent residence on behalf of the Applicant, the Officer failed to take into ac- count the evidence that the Sponsor would have led, had he been asked. 36 Second, the Officer based her conclusion about the relationship on an erroneous conclusion that the Sponsor and the Applicant had not visited enough. This conclusion was based on the denial of procedural fairness discussed above. Had the Applicant been given an opportunity to re- spond, the Sponsor would have given evidence of passport stamps and visas showing visits to the Applicant in 1999-2000 and 2008-2009. The Officer ignored this evidence, as well as evidence the Sponsor would have introduced which showed he could not have travelled to Zimbabwe. Had the Officer asked for an explanation, these concerns would have been addressed. 37 Third, the Applicant says that the Officer’s conclusion about his rela- tionship with the Sponsor was in error because it ignored evidence of ongoing contact between them. This included evidence that the Sponsor has attempted to gain entry visas to Canada for the Applicant in 2006, 2007, and 2008. Further, the conclusions as to the strength of the rela- tionship placed too much emphasis on the fact that there were no photos from the Sponsor’s visit in December 2006. As the Applicant was denied the opportunity to respond, he did not have the opportunity to adduce Zingano v. Canada (MCI) James Russell J. 257

evidence; to ignore the evidence that would have been adduced makes the Officer’s conclusion unreasonable. 38 Fourth, the conclusion that the relationship was not close enough was unreasonable because it was based on the unreasonable conclusion that the amounts of money the Sponsor remitted to Zimbabwe were relatively small. The Applicant says there was no evidence to support this conclu- sion, though he submitted a list of remittances in support of his applica- tion. This conclusion also ignored evidence in the Sponsor’s letter which noted that the levels of inflation in Zimbabwe were very high. 39 Fifth, the Officer’s conclusion about the Applicant’s relationship with the Sponsor unreasonably emphasized the fact that the Applicant had not met his Canadian step-family and that there would be no hardship from their continued separation. This conclusion ignores the deprivation that both the Applicant and his step-family have suffered from his absence. This conclusion also ignores the affidavit evidence of the Sponsor and Co-Sponsor attesting to the hardship their continued separation would cause. Relying on Maldonado v. Canada (Minister of Employment & Im- migration) (1979), [1980] 2 F.C. 302 (Fed. C.A.) at page 305, the Appli- cant says that these affidavits were entitled to a presumption of truth. It was therefore unreasonable for the Officer to ignore them. For the Of- ficer to conclude that the family would suffer no hardship does not fit with the Sponsor’s repeated attempts to bring the Applicant to Canada. The Applicant says the Officer also ignored evidence of financial limita- tions on the family which prevented them from visiting him in Zimbabwe. This evidence included records of their income, the need to care for the children of the Sponsor and Co-sponsor in Canada, and affi- davit evidence that the Sponsor was unable to work for a time because of health problems. 40 In all the above ways, the Officer fundamentally misapprehended the strength of the ongoing relationship between the Applicant and his Spon- sor in Canada by ignoring the evidence before her and failing to put her unanticipated concerns to the Applicant.

The Officer’s Conclusion That The Applicant Had a Suitable Living Situation in Zimbabwe Was Unreasonable 41 The Applicant also argues that the Officer’s conclusion that he had a suitable living situation in Zimbabwe was unreasonable because it was based on conclusions that the country conditions in Zimbabwe had im- proved, that his biological mother was involved in his life, and that it was 258 IMMIGRATION LAW REPORTER 3 Imm. L.R. (4th)

in his best interests to remain in Zimbabwe, all of which were unreasonable. 42 The conclusion that country conditions in Zimbabwe had improved did not engage in any meaningful way with statements in the Sponsor’s submissions in support of the H&C application about the unemployment rate, inflation, health care situation, and sanitation standards. This con- clusion was also unreasonable because the Officer was not sufficiently alert, alive, or sensitive to the best interests of the Applicant. 43 The only evidence on the role of his biological mother in his life that was before the Officer was the contact information for his mother which the Applicant provided at his interview for the 2007 application for a TRV. The Officer ignored affidavit evidence of the Sponsor that the Ap- plicant’s biological mother was not available to care for him. The Spon- sor’s affidavit is more recent than the contact information so it should have been preferred. Further, the Applicant has been living with his pa- ternal grandmother. All the evidence points to his biological mother abandoning him. Rather than relying on the evidence that the Applicant could contact his biological mother, what should have mattered to the Officer was whether his mother was able and wiling to provide adequate care, which clearly she was not. The Applicant also says that the Officer imposed an impossibly high evidentiary burden — to prove his mother was not involved in his life — so this conclusion was unreasonable. 44 The Applicant also says it was unreasonable for the Officer to con- clude that it was in his best interests to remain in Zimbabwe with his extended family rather than live with his step-family in Canada who he has never met. This conclusion relied on the gross generalization that it its common in Zimbabwean culture for children to live with their grand- parents. The Applicant notes that I said in Ponniah v. Canada (Minister of Citizenship & Immigration), 2003 FC 1016 (F.C.) at paragraph 10 that “[assumptions] based on cultural generalizations, particularly those relat- ing to ancillary issues, are not relevant considerations.” This cultural stereotype was not based on any evidence before the Officer and further, had she put this to him, the Applicant would have explained that neither the Sponsor nor his wife were Zimbabwean, so this generalization does not apply to them anyway. 45 The conclusion that it was in the Applicant’s best interests to remain with his extended family was also unreasonable because there was no evidence before the Officer as to who that extended family was or how they could support him. In Ebonka v. Canada (Minister of Citizenship & Zingano v. Canada (MCI) James Russell J. 259

Immigration), 2009 FC 80 (F.C.) at paragraph 25, Justice Michael Kelen held that it was unreasonable for an officer to rely on a relationship for which there is little evidence as proof that the applicant would not suffer hardship from separation from a relationship which is well established on the evidence. The Applicant says that his is such a case. Further, to hold that it is in the Applicant’s best interests to remain in Zimbabwe does not accord with paragraph 3(1)(a) of the Act, which says that one of the objectives of the Act is the reuniting of families in Canada.

The Officer Unreasonably Emphasized the Sponsor’s Non-Disclosure of the Applicant in his 2002 Application for Permanent Residence 46 As noted above, the Applicant is permanently excluded from the fam- ily class by paragraph 117(9)(d) of the Regulations. He says that subsec- tion 25(1) of the Act can be used to grant an exemption from paragraph 117(9)(d). Further, following de Guzman v. Canada (Minister of Citizenship & Immigration), 2005 FCA 436 (F.C.A.), the Applicant says that, when considering an H&C exemption from that paragraph, the Of- ficer must assess all H&C factors, including the best interests of the child. He also notes that an officer considering such an application must be alert, alive, and sensitive to the best interests of the child. 47 The Applicant says that it is an error for an officer assessing an H&C exemption from paragraph 117(9)(d) to place undue emphasis on the non-disclosure of a child over the H&C considerations or the paragraph 3(1)(d) objective of reuniting families in Canada. For this proposition, he relies on David v. Canada (Minister of Citizenship & Immigration), 2007 FC 546 (F.C.), Hurtado v. Canada (Minister of Citizenship & Immigra- tion), 2007 FC 552 (F.C.), Sultana v. Canada (Minister of Citizenship & Immigration), 2009 FC 533 (F.C.) and Krauchanka v. Canada (Minister of Citizenship & Immigration), 2010 FC 209 (F.C.). 48 The Applicant also says that the refusal of an H&C application is un- reasonable where the non-disclosed child is not otherwise inadmissible to Canada. Where a non-disclosed child is not inadmissible, the non-disclo- sure is immaterial to the non-disclosing parent’s application. In these cases, the policy rationale behind paragraph 117(9)(d) — ensuring that applicants do not later sponsor inadmissible family members — does not hold. An H&C exemption in this type of case is normally warranted and a denial of the application will normally be unreasonable. 260 IMMIGRATION LAW REPORTER 3 Imm. L.R. (4th)

49 In his case, the Applicant was not inadmissible when the Sponsor ap- plied for permanent residence in 2002. As such, an H&C exemption was warranted in his case and the denial of the same was unreasonable. 50 The Officer’s undue emphasis on the non-disclosure by the Sponsor of the Applicant is shown by her statement that “[The Applicant] remains permanently excluded from being sponsored as member of the family class regardless of the reasons why [the Sponsor] never declared him.” The Applicant says the undue emphasis is also shown by the dismissive attitude that the Officer displayed toward the relationship between the Applicant and his father and his circumstances in Zimbabwe, borne out by the unreasonableness of her conclusions on those issues.

The Respondent 51 The Respondent says that the onus is on applicants in H&C applica- tions to provide all relevant facts in support of their applications. In this case, the Officer provided the Applicant with all required procedural en- titlements, considered all the facts that were before her, and drew reason- able conclusions from the evidence.

There Was no Breach of Procedural Fairness 52 The Respondent says that the ultimate question, when examining the issue of procedural fairness, is whether the person subject to a decision had a meaningful opportunity to present his case. The Respondent argues that the Applicant’s right to procedural fairness was not breached, as he had every opportunity to put evidence before the Officer, yet chose not to do so. Here, the onus was clearly on the Applicant to demonstrate that an H&C exemption was warranted in his case. 53 Relying on Kisana, above, the Respondent says that there was no duty on the Officer to highlight the weaknesses in the Applicant’s case. There was no duty to point out the holes in the evidence concerning the relationship between the Applicant and his Canadian step-family; the Applicant had all the evidence in his hands. It was for the Applicant to draw a clear picture of the relationship and there was nothing to prevent the Applicant from submitting additional material to be considered by the Officer. The Respondent says, based on Owusu v. Canada (Minister of Citizenship & Immigration), 2004 FCA 38 (F.C.A.) that, where an appli- cant fails to present his case, as occurred here, he does so at his own peril. Zingano v. Canada (MCI) James Russell J. 261

The Officer Did Not Improperly Emphasize the Sponsor’s Non- Disclosure of the Applicant in 2002 54 The Officer did not unreasonably overemphasize the non-disclosure of the Applicant by the Sponsor in his 2002 application for permanent residence. Non-disclosure of a child is a relevant policy consideration in an H&C application, so it was proper for the Officer to consider it in her analysis. For this proposition, the Respondent relies on Li v. Canada (Minister of Citizenship & Immigration), 2006 FC 1292 (F.C.). 55 The Respondent says that the Officer’s statement in the Decision that “the [Applicant] remains permanently excluded as a member of the fam- ily class regardless of the reasons why [the Sponsor] never declared him” is not a major part of her reasons. This is only one of a number of factors she considered. The Officer simply noted that the Sponsor’s explanation was not convincing; this was not conclusive of the determination. 56 The Respondent also says that the reasonableness of an H&C exemp- tion from paragraph 117(9)(d) is independent of whether the non-dis- closed child is inadmissible. The failure to declare dependants is a rele- vant policy consideration whether or not the non-disclosed dependants are admissible. In this case, the Officer properly considered the non-dis- closure of the Applicant on the Sponsor’s 2002 Application.

There Was no Error in Assessing the Relationship Between the Sponsor and the Applicant 57 The Officer’s conclusion that the relationship between the Applicant and the Sponsor was not sufficiently close to merit an H&C exemption was reasonable, as it was based on all the evidence that was before her. The relationship between the Applicant and the Sponsor was central to the H&C determination in this case. When the Officer looked at the de- lay in applying for status for the Applicant, she was considering relevant evidence. 58 The Applicant has attacked the reasonableness of the Officer’s con- clusion that the remittances sent by the Sponsor to Zimbabwe were rela- tively small, but there was evidence before her that the Sponsor and Co- sponsor had a combined household income of $130,000. She also had before her a list of the remittances sent by the Sponsor to Zimbabwe, none of which was more than $544.00. Further, though the Applicant could have provided further evidence on the remittances to the Officer, he cannot now attempt to do so on judicial review. 262 IMMIGRATION LAW REPORTER 3 Imm. L.R. (4th)

59 The Respondent further says that it was open to the Officer to con- sider the lack of a visit by the Sponsor and the Canadian family to the Applicant in Zimbabwe in examining the relationships of the parties. The Officer addressed the fact that the Co-sponsor and her children had not visited the Applicant due to extenuating circumstances when she noted that the family could have met in a neighbouring country. In addition, the relatively high income of the family does not support the Applicant’s contention that there were financial obstacles preventing the Canadian family from visiting the Applicant in Africa.

The Officer’s Conclusion on the Applicant’s Circumstances in Zimbabwe was Reasonable 60 Finally, the Respondent argues that the Officer’s conclusion with re- spect to the Applicant’s situation in Zimbabwe was reasonable and was based on all the evidence before her. The Officer considered all the evi- dence before her and referred in the Decision to the Sponsor’s submis- sion on the economic situation, political instability, and the availability of education and medical care. The Officer was not required to compare the situation in Zimbabwe with that in Canada and “the fact that [the Applicant] might be better off in Canada in terms of general comfort and future opportunities cannot, [...], be conclusive in an H&C Decision that is intended to assess undue hardship.” (Vasquez v. Canada (Minister of Citizenship & Immigration), 2005 FC 91 (F.C.) at paragraph 43). The Respondent also says that it is not for the Court to examine whether the Officer gave this factor sufficient weight. 61 Overall, the Officer adequately addressed the issue of hardship. She considered the length of time the Sponsor had been absent from the Ap- plicant’s life, the amount of contact between them, the size of the remit- tances from the Sponsor to Zimbabwe, and the Applicant’s residence with his paternal grandmother. Following Yue v. Canada (Minister of Citizenship & Immigration), 2006 FC 717 (F.C.), the Respondent says that these are the kinds of factors which have been found by this Court not to warrant judicial intervention.

Analysis 62 The Applicant complains that, if only the Officer had asked more questions or alerted him to concerns, he could have provided more infor- mation that would have fundamentally changed the picture of the rela- tionship he had with his family in Canada. He says that it was procedur- Zingano v. Canada (MCI) James Russell J. 263

ally unfair for the Officer not to have alerted him to concerns about his application and not to have given him an opportunity to address those concerns. 63 I think that this complaint misconceives the nature of the process. As the Respondent points out, in the H&C context, the onus is on an appli- cant to demonstrate that an exemption is warranted and an officer is under no duty to highlight weaknesses in an application and request fur- ther submissions. See Kisana, above, at paragraph 45. Many of the issues raised by the Applicant in this review application are no more than a request to the Court that the law be changed and the onus placed upon the Officer. This cannot be done. The facts to support the relationship were in the hands of the Applicant and his family. It was up to the Appli- cant to establish the nature of the relationship he had with his Canadian family. This required him to show how that relationship was nurtured and maintained, the nature of the emotional and psychological connec- tion he had with his father, and any barriers they faced in making use of the resources available to them for communication, connection, and support. 64 There were no limits on the information that the Applicant was able to adduce to substantiate the nature of the relationship, and he was at liberty to go on providing additional material at any time prior to the final decision. The Applicant and his family now wish they had provided the Officer with more material and they have attempted to lay before the Court what they could have said and done, blaming the Officer for not allowing them the opportunity to provide that information to him. This aspect of their application has to be dismissed. As the Court of Appeal has said, if an applicant fails to present a fulsome case, he or she does so at their peril. See Owusu, above, at paragraph 8. 65 In Owusu v. Canada (Minister of Citizenship & Immigration), 2003 FCT 94 (Fed. T.D.), Justice Frederick Gibson had the following to say on point at paragraph 11: The onus on an application for humanitarian or compassionate relief lies with the applicant. In Prasad v. Canada (Minister of Citizenship and Immigration), in the context of judicial review of a visa officer decision, Justice Muldoon wrote at paragraph 7: The onus is on the applicant to satisfy the visa officer fully of all the positive ingredients in the applicant’s ap- plication. It is not for the visa officer to wait and to offer the applicant a second, or several opportunities to satisfy 264 IMMIGRATION LAW REPORTER 3 Imm. L.R. (4th)

the visa officer on necessary points which the applicant may have overlooked. In Patel v. Canada (Minister of Citizenship and Immigration), Justice Heald, once again in the context of judicial review of a visa officer’s decision, but dealing with the issue of humanitarian or compassionate grounds, wrote at paragraph 9: The applicant submits that he is entitled to have all rele- vant evidence considered on a humanitarian and compas- sionate application. I agree with that submission. How- ever, the onus in this respect lies with the applicant. It is his responsibility to bring to the visa officer’s attention any evidence relevant to humanitarian and compassionate considerations. 66 I realize there are situations where an officer would have an obliga- tion to make further inquiries and seek clarification. Justice provided guidance on this issue at paragraph 24 of Hassani, above: Having reviewed the factual context of the cases cited above, it is clear that where a concern arises directly from the requirements of the legislation or related regulations, a visa officer will not be under a duty to provide an opportunity for the applicant to address his or her concerns. Where however the issue is not one that arises in this con- text, such a duty may arise. This is often the case where the credibil- ity, accuracy or genuine nature of information submitted by the ap- plicant in support of their application is the basis of the visa officer’s concern, as was the case in Rukmangathan, and in John and Cornea cited by the Court in Rukmangathan, above. 67 On the facts of the present case, I do not believe that any such excep- tion arises. Also, I do not think that any of the points relied upon by the Officer for her conclusions concerning the family relationship could not have been anticipated by the Applicant. The Federal Court of Appeal has established the basic principles applicable to a case such as this in Kisana, above: 33 Many of the factors which an officer is required to consider in determining an H&C application can be found in the guidelines is- sued to immigration officers by the Minister, to which D´ecary J.A. refers in paragraph 7 of his Reasons in Hawthorne, supra, and which can be found at paragraph 30 of Evans J.A.’s concurring Reasons in that case. These factors include hardship arising from the geographi- cal separation of family members. In examining this factor, the of- ficer should consider: the effective links with family members, i.e. in Zingano v. Canada (MCI) James Russell J. 265

terms of ongoing relationship as opposed to the simple biological fact of relationship; has there been any previous period of separation and, if so, for how long and why; the degree of psychological and emo- tional support in relation to other family members; options, if any, for the family to be reunited in another country; financial depen- dence, and; the particular circumstances of the children. ... 45 It is trite law that the content of procedural fairness is variable and contextual (see: Baker, supra, para. 21; and Khan v. Canada (MCI), [2002] 2 F.C. 413). The ultimate question in each case is whether the person affected by a decision “had a meaningful opportunity to pre- sent their case fully and fairly” (see: Baker, supra, para. 30). In the context of H&C applications, it has been consistently held that the onus of establishing that an H&C exemption is warranted lies with an applicant; an officer is under no duty to highlight weaknesses in an application and to request further submissions (see, for example: Thandal v. Canada (MCI), 2008 FC 489 at para. 9). In Owusu, supra, this Court held that an H&C officer was not under a positive obliga- tion to make inquiries concerning the best interests of children in cir- cumstances where the issue was raised only in an “oblique, cursory and obscure way” (at para. 9). The H&C submissions in that case consisted of a 7- page letter in which the only reference to the best interests of the children was contained in the sentence: “Should he be forced to return to Canada, [Mr. Owusu] will not have any way to support his family financially and he will have to live every day of his life in constant fear” (at para. 6). ... 56 There can be no doubt that the officer could have asked more questions in order to obtain additional information with regard to the twins’ situation in India, but, as we shall see, she was under no duty to do so in this case. It may be that the pointed and narrow questions disclosed by the CAIPS notes probably did not constitute the most effective manner of obtaining information from these applicants, par- ticularly in light of the lack of documentary evidence provided by them. However, the vacuum, if any, was created by the appellants’ failure to assume their burden of proof. In these circumstances, the officer’s poor interviewing techniques, if that be the case, are, in my view, insufficient to justify intervention on our part. 68 Paragraph 33 of Kisana provides a checklist of what an H&C officer has to consider. The Officer in this case dealt with the matters referred to in this paragraph. Paragraph 33 also provides the Applicant with a check- list for what should be addressed in his application. The Applicant had 266 IMMIGRATION LAW REPORTER 3 Imm. L.R. (4th)

legal advice in the preparation of his H&C application. The package of information provided to the Officer was not the information that has now been placed before this Court. I do not think the Officer can be faulted for not taking into account facts and explanations that were not placed before her. 69 There are other aspects of the Applicant’s arguments that are just not accurate when the Decision is read in its entirety. For example, there is really no indication, in my view, that the Officer improperly emphasized the Sponsor’s failure to declare the Applicant in the Sponsor’s earlier 2002 application. I think the Respondent is correct on this point. 70 It was open to the Officer to consider the failure of the Sponsor to properly declare the Applicant as it is one public policy factor to be con- sidered in the H&C assessment. See Li, above, at paragraph 33 and Kisana, above, at paragraph 27. 71 A review of the Decision concerning the assessment of H&C factors does not support the Applicant’s allegation that the Officer overempha- sized the Sponsor’s failure to declare his son when the Sponsor landed in Canada. 72 The Applicant’s only example of the Officer placing “particular em- phasis on the fact of the non-disclosure” was the statement that PA remains permanently excluded from being sponsored as a mem- ber of the family class, regardless of the reasons why sponsor never declared him... 73 This statement does not form a major part of the Oficer’ reasons con- cerning the H&C factors. Further, the Officer simply notes that the ex- planation for non-disclosure provided by the Sponsor is not convincing. This comment is one of many which review the evidence presented by the Applicant. It is not a conclusive statement on the strength of the H&C application and does not override the other factors, which determined the final result. 74 Further, the fact of the Applicant’s admissibility does not determine the reasonableness of the Officer’s assessment of H&C factors. The fail- ure to declare raises a public policy concern, regardless of the Appli- cant’s status. See Li, above, Yue, above, and Sandhu v. Canada (Minister of Citizenship & Immigration), 2007 FC 156 (F.C.). 75 That being said, there are some aspects of the Decision that this Court finds troubling, and I think they need to be examined to determine whether they render the Decision unreasonable. I am particularly con- Zingano v. Canada (MCI) James Russell J. 267

cerned by the Officer’s consideration of the best interests of the child (Applicant). It is obvious from the Decision that the Officer took into account the situation in Zimbabwe when dealing with this issue. Her fi- nal conclusions on point read as follows: While the situation in Zimbabwe is not ideal, it has improved. While PA’s representative states that it is in PA’s best interest to be with his father in CDA, I do not agree, as I believe that it is in PA’s best interest to in fact be with his extended family in Zimbabwe who he knows and who has taken care of him most of his life rather than a family in CDA, who he has had limited contact with for most of his life with 3 members whom he had never even met in person. 76 Counsel for the Respondent conceded to the Court that she knows of no evidence before the Officer that would support a conclusion that the situation in Zimbabwe has improved. Reviewing the record myself, it seems to me that the Officer’s conclusions or statements about Zimbabwe are totally inaccurate and perverse. It is not only that the situ- ation in Zimbabwe “is not ideal”; the reality is that it could not be worse. There is no evidence that it has improved or that improvement is likely anytime soon. The picture is one of increasing international concern over ever declining socio-economic conditions, a collapsing education system, and increasing violence. 77 I realize that, in conducting an assessment of the situation of the Ap- plicant, the Officer is not required to make a comparative analysis be- tween the Applicant’s situation in Zimbabwe and his potential situation in Canada. As I pointed out in Vasquez, above, at paragraph 43, The fact that the children might be better off in Canada in terms of general comfort and future opportunities cannot, in my view, be con- clusive in an H&C Decision that is intended to assess undue hardship. 78 I also realize that whether the Officer gave this factor sufficient weight is not for the Court to decide. 79 What concerns me is that the Officer provides no basis for her con- clusion that the situation in Zimbabwe has improved (and Respondent’s counsel cannot point to any), and she appears to be unaware of the evi- dence before her that reveals the real situation in Zimbabwe. 80 The Officer herself makes the situation in Zimbabwe a significant factor in her analysis and, of course, it ought to be when assessing the best interests of the Applicant. I think her incorrect analysis of the situa- tion is a highly material error that renders the Decision unreasonable. I 268 IMMIGRATION LAW REPORTER 3 Imm. L.R. (4th)

cannot say that the Officer would have come to the same conclusion re- garding the best interests of the Applicant if she had taken into account what the evidence does say about declining conditions in Zimbabwe and the prospects for the Applicant if he has to remain there. Consequently, I believe this matter requires reconsideration. 81 I am also concerned by the Officer’s assessment that “it is possible that [the Applicant] does have contact with his [biological] mother.” The evidence from the Sponsor is clear that there is no such contact and there is nothing in the record to suggest this is not true. The fact that the Appli- cant may have had a contact address for his biological mother does not mean that she plays, or will play, any role in his life. If the Officer felt that the Sponsor could not be believed on this issue, then she should have interviewed him to test his credibility. Her failure to do this renders her suggestion that the biological mother could be available to the Applicant unreasonable. Once again, this renders the Decision unsafe regarding the Officer’s analysis of the Applicant’s best interests. The finding was highly material and there is no evidence to support the Officer’s conclusion. 82 Counsel agree there is no question for certification and the Court concurs.

Judgment THIS COURT’S JUDGMENT is that 1. The application is allowed. The Decision is quashed and the mat- ter is returned for reconsideration by a different officer. 2. There is no question for certification. 3. The Style of Cause is amended to show the Applicant as “Salim Tafadzwa Zingano by his litigation guardian Lameck Zingano”. Application granted. Shah v. Canada (MCI) 269

[Indexed as: Shah v. Canada (Minister of Citizenship & Immigration)] Shaffira Shah, Applicant and The Minister of Citizenship and Immigration, Respondent Federal Court Docket: IMM-2596-11 2011 FC 1269 Leonard S. Mandamin J. Heard: November 1, 2011 Judgment: November 7, 2011 Immigration and citizenship –––– Refugee protection — Practice and proce- dure in refugee claims — Post-determination options — Humanitarian and compassionate review — Unusual, undeserved, or disproportionate hard- ship –––– Applicant came to Canada from Trinidad 38 years ago — Applicant became permanent resident but lost that status in 1999 following criminal con- viction — Applicant was granted stay of removal but stay was vacated in 2006 following applicant’s convictions for various shoplifting offences — Applicant was diagnosed with serious mental health problems and was awarded disability benefits, as she was unable to work — Applicant also began counselling for kleptomania — Applicant’s psychiatrist and physician strongly recommended that removal not take place, as it would increase risk of suicide — Applicant made application for permanent residence on humanitarian and compassionate grounds — Immigration officer refused application — Applicant brought appli- cation for judicial review of officer’s decision — Application granted — Where officer is faced with evidence of risk of suicide resulting from removal itself, it is not sufficient for officer to simply examine availability of medical services in destination country — Officer is also required to determine whether putting ap- plicant through removal and its potential for leading to suicide amounted to un- due, undeserved or disproportionate hardship — It was unreasonable for officer to state that applicant would be able to find job and housing in Trinidad — Evi- dence before officer was that application was person with substantial mental im- pairments which made her unfit to work — No evidence existed to support of- ficer’s finding that applicant would not be isolated in Trinidad — Officer ignored unequivocal testimony that applicant had no family or friends left in Trinidad — Officer applied higher standard than was appropriate for humanita- rian and compassionate decisions by requiring applicant to establish personal risk beyond that faced by others in Trinidad — Test of unusual, undeserved or 270 IMMIGRATION LAW REPORTER 3 Imm. L.R. (4th)

disproportionate hardship is not limited to personal risk to applicant’s life or safety. Immigration and citizenship –––– Admission — Application for temporary resident or immigrant visa — Powers and duties of visa or immigration of- ficer — Miscellaneous –––– Requirement to consider request for temporary res- ident permit — Where applicant makes request for temporary resident permit, it must be considered by officer — Failure to consider request for temporary resi- dent permit is reviewable error — Even if there is no basis for issuance of tem- porary resident permit, officer must consider request and indicate that he or she has done so. Cases considered by Leonard S. Mandamin J.: Aboudaia c. Canada (Ministre de la Citoyennet´e & de l’Immigration) (2009), 2009 CF 1169, 2009 CarswellNat 3837, 2009 FC 1169, 2009 CarswellNat 5260 (F.C.) — referred to Baker v. Canada (Minister of Citizenship & Immigration) (1999), 1 Imm. L.R. (3d) 1, [1999] 2 S.C.R. 817, 14 Admin. L.R. (3d) 173, 174 D.L.R. (4th) 193, 1999 CarswellNat 1124, 1999 CarswellNat 1125, 243 N.R. 22, [1999] S.C.J. No. 39 (S.C.C.) — referred to Davis v. Canada (Minister of Citizenship & Immigration) (2011), 2011 FC 97, 96 Imm. L.R. (3d) 267, 2011 CarswellNat 167, 2011 CF 97, 2011 Car- swellNat 746, [2011] F.C.J. No. 114, [2011] A.C.F. No. 114 (F.C.) — followed Dhandal v. Canada (Minister of Citizenship & Immigration) (2009), 82 Imm. L.R. (3d) 214, 2009 CF 865, 2009 CarswellNat 5809, 2009 FC 865, 2009 CarswellNat 2582 (F.C.) — referred to Japson v. Canada (Minister of Citizenship & Immigration) (2004), 2004 FC 520, 2004 CarswellNat 1033, 2004 CF 520, 2004 CarswellNat 2398, [2004] F.C.J. No. 694 (F.C.) — considered Kandhai v. Canada (Minister of Citizenship & Immigration) (2009), 2009 CF 656, 2009 FC 656, 2009 CarswellNat 1919, 2009 CarswellNat 5681, 81 Imm. L.R. (3d) 144 (F.C.) — distinguished Lee v. Canada (Minister of Citizenship & Immigration) (2006), 2006 Car- swellNat 4260, 60 Imm. L.R. (3d) 62, 304 F.T.R. 241 (Eng.), 2006 Car- swellNat 6294, 2006 CF 1461, 2006 FC 1461, [2006] F.C.J. No. 1841 (F.C.) — referred to New Brunswick (Board of Management) v. Dunsmuir (2008), 372 N.R. 1, 69 Admin. L.R. (4th) 1, 69 Imm. L.R. (3d) 1, (sub nom. Dunsmuir v. New Brunswick) [2008] 1 S.C.R. 190, 844 A.P.R. 1, (sub nom. Dunsmuir v. New Brunswick) 2008 C.L.L.C. 220-020, D.T.E. 2008T-223, 329 N.B.R. (2d) 1, (sub nom. Dunsmuir v. New Brunswick) 170 L.A.C. (4th) 1, (sub nom. Dunsmuir v. New Brunswick) 291 D.L.R. (4th) 577, 2008 CarswellNB 124, 2008 CarswellNB 125, 2008 SCC 9, 64 C.C.E.L. (3d) 1, (sub nom. Shah v. Canada (MCI) Leonard S. Mandamin J. 271

Dunsmuir v. New Brunswick) 95 L.C.R. 65, [2008] S.C.J. No. 9, [2008] A.C.S. No. 9 (S.C.C.) — followed Reba¨ı v. Canada (Minister of Citizenship & Immigration) (2008), 2008 Car- swellNat 57, 2008 CF 24, 2008 CarswellNat 556, 67 Imm. L.R. (3d) 191, 2008 FC 24, [2008] F.C.J. No. 43 (F.C.) — followed Sahota v. Canada (Minister of Citizenship & Immigration) (2007), 2007 FC 651, 2007 CarswellNat 1718, [2007] F.C.J. No. 882 (F.C.) — referred to Sha’er v. Canada (Minister of Citizenship & Immigration) (2007), 60 Imm. L.R. (3d) 189, 2007 CF 231, 2007 CarswellNat 446, 2007 FC 231, 2007 Car- swellNat 2328, [2007] F.C.J. No. 297 (F.C.) — referred to Siddiqui v. Canada (Minister of Citizenship & Immigration) (2008), 2008 FC 989, 2008 CarswellNat 3082, 74 Imm. L.R. (3d) 181 (F.C.) — distinguished Statutes considered: Federal Courts Act, R.S.C. 1985, c. F-7 s. 18.1(4)(d) [en. 1990, c. 8, s. 5] — considered Immigration and Refugee Protection Act, S.C. 2001, c. 27 s. 11(1) — considered s. 24(1) — considered s. 25 — pursuant to s. 25(1) — considered s. 96 — referred to s. 97 — referred to s. 122 — considered

APPLICATION for judicial review of decision refusing application for perma- nent residence on humanitarian and compassionate grounds.

Katherine Ramsay, for Applicant Lorne McClenaghan, for Respondent

Leonard S. Mandamin J.:

1 Shaffira Shah (the Applicant) applies for judicial review of a decision made by a Pre-Removal Risk Assessment Officer (the Officer), dated March 3, 2011, refusing the Applicant’s application for permanent resi- dence from within Canada based on humanitarian and compassionate (H&C) considerations under section 25 of the Immigration and Refugee Protection Act, SC 2001, c 27 (IRPA). 2 The Applicant is from Trinidad and Tobago. She came to Canada when she was 18, married, and became a permanent resident. The Appli- cant was abandoned by her spouse in 1979 and left to raise their three 272 IMMIGRATION LAW REPORTER 3 Imm. L.R. (4th)

daughters on her own. The relationship had been very abusive and, not- withstanding separation, she became involved in a fraudulent scheme ini- tiated by her spouse. In 1994, she was convicted for welfare fraud. 3 The Applicant lost her permanent resident status after a 1999 Immi- gration Appeals Division (IAD) decision because of her criminal convic- tion. However she was granted a stay of removal because of her personal circumstances. She was subsequently convicted for numerous shoplifting offences and her stay of removal was dismissed because of continuing criminality after a 2006 IAD hearing. Following the 2006 IAD decision, the Applicant was diagnosed with mental health problems. She began at- tending counselling for shoplifting with the Elizabeth Fry Society. She was also awarded disability benefits because she was found unfit to work due to her mental health problems. 4 In 2006 the Applicant made an H&C application for permanent resi- dence on compassionate grounds. As well, she requested a temporary resident permit (TRP) in hopes of becoming eligible for a pardon for her shoplifting offences. The reviewing Officer refused the H&C application and did not consider the TRP request. 5 For reasons that follow, I am granting the request for judicial review.

Background 6 The Applicant was born in Trinidad and Tobago on May 11, 1955. As a child, she was beaten and abused by her alcoholic father. She escaped to Canada in 1973, at the age of 18, where she met Ahmed Shah and married him in 1975. They had three children together: Sandy, Charlene, and Sabrina, born in 1974, 1977, and 1979 respectively. 7 The Applicant lives with her daughters helping to care for her grandchildren, Aidan who is eleven and Tristan who is two and a half years old. She has no family in Trinidad and has returned to the country only once in 38 years to attend her father’s funeral. Her mother, three brothers, and two sisters live in the United States while another sister lives in Brampton, Ontario. 8 The Applicant was sponsored by her husband and obtained permanent residence in Canada in 1978. Sadly, her husband, like her father, was an alcoholic who beat her severely. At one point, he held a knife to her throat in front of her children. He left the family relationship in 1979 but maintained contact with the Applicant. In 1994 he returned to Trinidad. She never heard from her husband again. She never received financial Shah v. Canada (MCI) Leonard S. Mandamin J. 273

support after his departure or, for that matter, during their separation prior to it. 9 After separating from her husband in 1979, the Applicant began to collect social assistance to support herself and her three children. During the separation, her husband bought two properties and placed her name on the title. He collected rent money from the properties but he did not share it with her. Although she did not receive any money, she was com- plicit and was convicted in 1994 of fraud for receipt of welfare while income was derived from the properties. She received a sentence of eigh- teen months. 10 Because the Applicant received a term of imprisonment of more than six months, a deportation order was issued against her in 1998. On ap- peal to the IAD however, the IAD agreed with a joint recommendation of the Minister and the Applicant for a stay of deportation order. 11 The Applicant has been convicted of theft under and possession of stolen property offences stemming from shoplifting. In July of 2003, de- spite having been convicted of three criminal convictions since having been put on stay in 1999, the stay of removal was extended for another three years. By May of 2006, the Applicant had been convicted of an- other four theft under offences and one trespass to property offence in violation of the conditions of the stay. 12 Due to the continuing criminal behaviour and violations of the stay conditions, the IAD dismissed the stay in May 2006. The IAD found the Applicant to be a habitual shoplifter that had not rehabilitated herself or taken any constructive action to do so during the previous six years of the stay of removal. 13 Following the 2006 IAD decision, the Applicant was diagnosed with severe depression and panic disorder for which she began receiving treat- ment. She also began attending the Shop Lifting and Fraud Program and the Healing from Abuse Group Program at the Elizabeth Fry Society of Canada. In November of 2008, the Ontario Social Benefits Tribunal also determined the Applicant had a substantial mental impairment, namely major depression and anxiety disorder, and concluded she was unable to work because of her mental health problems. The Applicant was awarded and began receiving disability benefits. 14 In February 2009, the Applicant filed an application for permanent residence on H&C grounds. That same month, the Canada Border Ser- vices Agency (CBSA) issued her a direction to report for removal in March, 2009. Following several proceedings over nearly two years, on 274 IMMIGRATION LAW REPORTER 3 Imm. L.R. (4th)

November 9, 2010, the Respondent granted a ministerial stay of removal until the outstanding H&C application could be determined. Further sub- missions to this application were filed on November 17, 2010, and the application was finally considered on March 2, 2011. 15 The Officer refused the Applicant’s application. This decision was communicated to her on April 5, 2011. She filed an application for leave and judicial review on April 19, 2011 and the request for leave was granted by Justice Gauthier on August 3, 2011.

Decision under Review 16 The Officer began by setting out the appropriate test in an H&C ap- plication: the applicant bears the onus of demonstrating that her personal circumstances are such that the hardship of having to obtain a permanent resident visa from outside Canada in the normal manner would be: i) unusual (“a hardship not anticipated by the Act or Regulations”) and undeserved (the result of circumstances beyond the appli- cant’s control) or ii) disproportionate (where the hardship would have a disproportion- ate impact on the applicant due to their personal circumstances). 17 The Officer identified the Applicant’s H&C grounds as being based on the following factors: degree of establishment in Canada; personal re- lationships/ties in Trinidad/Canada; and risk of returning to Trini- dad/Medical concerns. The officer then weighed these factors against the Applicant’s criminal inadmissibility.

Establishment in Canada 18 The Officer noted the Applicant was a housewife from 1974 to 1997, worked from 1997 to 2001, then assisted in raising her grandson until September 2008, before receiving government funded assistance through the Ontario Disability Support Program in November of 2008. The Of- ficer found no evidence of community involvement or upgrading of skills since arriving in 1973 and while there were numerous letters from friends, none demonstrated close interdependent relationships that would suffer hardship if severed.

Personal relationships/ties in Trinidad/Canada 19 After reviewing the letters from the Applicant’s three daughters and the sister in Brampton, the Officer concluded that while the Applicant is largely dependent on her children, she has built strong relationships with Shah v. Canada (MCI) Leonard S. Mandamin J. 275

both her children and grandchildren, and it would be in the best interests of her grandchildren to have their grandmother physically in their lives, these factors did not outweigh the Applicant’s numerous criminal con- victions over a substantial time frame of approximately thirty years. 20 The Officer found that should the Applicant need to re-establish her- self in Trinidad, it would be reasonable to assume that she would have the financial and emotional support and assistance of her family in Can- ada, and be able to apply her skills and work experiences acquired in Canada to assist her in obtaining employment and finding a place to live. 21 The Officer found there were reasonable grounds to believe the Ap- plicant could continue her relationship with her family, albeit long-dis- tance, and that the best interests of her grandsons were not sufficient to outweigh the negative factors associated with her criminal inadmissibil- ity. Thus the Officer concluded that the elements covered in this assess- ment factor would not contribute to a hardship that is unusual and under- served, or disproportionate.

Risk of returning to Trinidad/Medical concerns 22 The Officer noted the Applicant’s counsel’s statement: “Ms. Shah’s issues of hardship, primarily centered around the severe isolation she will face in Trinidad, the effect on her mental health possibly leading to a suicide and the risk posed by both the local criminal elements and her ex- husband, are serious enough to have warranted a stay of removal in the Federal Court until the H&C is decided”. 23 The Officer noted that the Applicant’s ex-husband was deceased and would no longer pose a risk (having passed away after the application for H&C consideration was filed). The Officer noted that there was insuffi- cient evidence the Applicant would be personally targeted by criminal elements upon her return and that the general risk faced by all individuals in the country does not amount to hardship that is unusual and unde- served, or disproportionate. 24 On the issue of isolation, the Officer found it reasonable to assume that at least some of the acquaintances and family the Applicant had in the area she grew up in would continue to reside there. The Officer ac- knowledged the Applicant had no home, immediate family, employment or anything to go back to, but found it reasonable to presume her daugh- ters would assist her in the relocation process, that the Applicant would not be returning to an unfamiliar place, culture or language that would 276 IMMIGRATION LAW REPORTER 3 Imm. L.R. (4th)

render reintegration unfeasible, and that there was insufficient evidence that the Applicant would be unable to secure housing. 25 The Officer acknowledged the Applicant’s diagnosis of anxiety and major depression, the concerns expressed by the Applicant’s family and medical practitioners over the Applicant being removed from Canada and no longer receiving the counselling and medical care she has been receiving here, as well as their concerns she would be unable to obtain the same type of care in Trinidad. The Officer concluded, however, that no objective evidence was provided to substantiate that the same type of counselling and medication would not be available or inaccessible in Trinidad and that her current physicians are not experts as to the medical services available in Trinidad.

Criminal Inadmissibility 26 The Officer began by reiterating the Applicant’s criminal record, in- cluding her conviction for fraud which led to the issuance of her deporta- tion order, and the numerous subsequent convictions for theft under which rendered her criminally inadmissible. The Officer then acknowl- edged the Applicant’s remorse and attempts to change her ways through counselling and by seeking medical attention. 27 The Officer noted that the Applicant committed criminal offences even while under counselling, the latest charges for theft having been laid on May 24 and October 1, 2009, and concluded that despite having a familial support network, counselling, and medication, the Applicant has continued to commit offences of theft. The Officer concluded that the Applicant had submitted insufficient evidence to indicate she would suf- fer unusual and undeserved, or disproportionate hardship to such a de- gree that the hardship would outweigh her criminal inadmissibility.

Conclusion 28 The Officer concluded that the Applicant’s submissions failed to show personal circumstances such that the requirement of having to ob- tain a permanent resident visa from outside Canada constituted unusual and undeserved, or disproportionate hardship and that as a result, there were insufficient H&C grounds to approve the application. Shah v. Canada (MCI) Leonard S. Mandamin J. 277

Relevant Legislation 29 The Immigration and Refugee Protection Act, SC 2001, c 27 pro- vides: 11. (1) A foreign national must, before entering Canada, apply to an officer for a visa or for any other document required by the regula- tions. The visa or document may be issued if, following an examina- tion, the officer is satisfied that the foreign national is not inadmissi- ble and meets the requirements of this Act. ... 24. (1) A foreign national who, in the opinion of an officer, is inad- missible or does not meet the requirements of this Act becomes a temporary resident if an officer is of the opinion that it is justified in the circumstances and issues a temporary resident permit, which may be cancelled at any time. ... 25. (1) The Minister must, on request of a foreign national in Canada who is inadmissible or who does not meet the requirements of this Act, and may, on request of a foreign national outside Canada, ex- amine the circumstances concerning the foreign national and may grant the foreign national permanent resident status or an exemption from any applicable criteria or obligations of this Act if the Minister is of the opinion that it is justified by humanitarian and compassion- ate considerations relating to the foreign national, taking into account the best interests of a child directly affected. 11. (1) L’´etranger doit, pr´ealablement a` son entr´ee au Canada, de- mander a` l’agent les visa et autres documents requis par r`eglement. L’agent peut les d´elivrer sur preuve, a` la suite d’un contrˆole, que l’´etranger n’est pas interdit de territoire et se conforme a` la pr´esente loi. ... 24. (1) Devient r´esident temporaire l’´etranger, dont l’agent estime qu’il est interdit de territoire ou ne se conforme pas a` la pr´esente loi, a` qui il d´elivre, s’il estime que les circonstances le justifient, un permis de s´ejour temporaire — titre r´evocable en tout temps. ... 25. (1) Le ministre doit, sur demande d’un etranger´ se trouvant au Canada qui est interdit de territoire ou qui ne se conforme pas a` la pr´esente loi, et peut, sur demande d’un etranger´ se trouvant hors du Canada, etudier´ le cas de cet etranger;´ il peut lui octroyer le statut de r´esident permanent ou lever tout ou partie des crit`eres et obligations 278 IMMIGRATION LAW REPORTER 3 Imm. L.R. (4th)

applicables, s’il estime que des consid´erations d’ordre humanitaire relatives a` l’´etranger le justifient, compte tenu de l’int´erˆet sup´erieur de l’enfant directement touch´e. 30 The Federal Courts Act, RSC 1985, c F-7: 18.1(4) The Federal Court may grant relief under subsection (3) if it is satisfied that the federal board, commission or other tribunal ... (d) based its decision or order on an erroneous finding of fact that it made in a perverse or capricious manner or without regard for the material before it; 18.1(4) Les mesures pr´evues au paragraphe (3) sont prises si la Cour f´ed´erale est convaincue que l’office f´ed´eral, selon le cas: ... d) a rendu une d´ecision ou une ordonnance fond´ee sur une conclu- sion de fait erron´ee, tir´ee de fa¸con abusive ou arbitraire ou sans tenir compte des el´´ ements dont il dispose;

Issues 31 In my view, the issues are: 1. Did the Officer fail to provide adequate reasons as to why the H&C factors did not 2. In light of the evidence submitted by the Applicant, was the Of- ficer’s decision to refuse 3. Did the Officer err by applying the criteria for PRRA applications, as set out in sections 4. Did the Officer err in failing to consider the Applicant’s request for a TRP?

Standard of Review 32 The Supreme Court of Canada has held in New Brunswick (Board of Management) v. Dunsmuir, 2008 SCC 9, [2008] 1 S.C.R. 190 (S.C.C.), that there are only two standards of review: correctness for questions of law and reasonableness involving questions of mixed fact and law and fact. The Supreme Court has also held that where the standard of review has been previously determined, a standard of review analysis need not be repeated. 33 The appropriate standard of review of an officer’s decision to refuse an applicant’s H&C application is reasonableness: Baker v. Canada Shah v. Canada (MCI) Leonard S. Mandamin J. 279

(Minister of Citizenship & Immigration), [1999] 2 S.C.R. 817, 174 D.L.R. (4th) 193 (S.C.C.) at paras 57-62. 34 The adequacy of reasons is a question of procedural fairness, which is reviewable on the standard of correctness: Siddiqui v. Canada (Minister of Citizenship & Immigration), 2008 FC 989, 74 Imm. L.R. (3d) 181 (F.C.) at para 17 [Siddiqui]. 35 Applying the incorrect analysis to an H&C application is an error of law and warrants judicial review: Sha’er v. Canada (Minister of Citizenship & Immigration), 2007 FC 231, 60 Imm. L.R. (3d) 189 (F.C.) at para 15, [Sha’er]. 36 Finally, the failure to consider the Applicant’s request for a TRP has been treated in previous case law as an error in law or an error in due process, both reviewable on a standard of correctness: Dhandal v. Canada (Minister of Citizenship & Immigration), 2009 FC 865, 82 Imm. L.R. (3d) 214 (F.C.) paras at 11-17 [Dhandal]; Lee v. Canada (Minister of Citizenship & Immigration), 2006 FC 1461, 60 Imm. L.R. (3d) 62 (F.C.) at para 18 [Lee ].

Analysis 1. Did the Officer fail to provide adequate reasons as to why the H&C factors did not outweigh the Applicant’s criminal inadmissibility? 37 The Applicant argues the Officer simply concluded the H&C factors did not outweigh the negative factors associated with her criminal inad- missibility without providing any reasons or analysis to support this con- clusion. She argues that she is left in the position of not knowing why her application was refused and that this amounts to a violation of her right to procedural fairness. 38 For its part, the Respondent submits that the reasons show the Of- ficer’s conclusion is due to the serious and continuing nature of the Ap- plicant’s criminality, with no basis for any belief the Applicant will stop. 39 After reviewing Siddiqui and Kandhai v. Canada (Minister of Citizenship & Immigration), 2009 FC 656, 81 Imm. L.R. (3d) 144 (F.C.) [Kandhai], cited by the Applicant in support of her position, I find these decisions to be distinguishable from the case at hand. 40 In Siddiqui, the officer’s reasons only considered positive H&C fac- tors before concluding they did not amount to undue and undeserved, or disproportionate hardship. Similarly in Kandhai, the officer recited the applicants’ submissions regarding militating factors and then simply con- 280 IMMIGRATION LAW REPORTER 3 Imm. L.R. (4th)

cluded that these factors were not sufficient to justify the granting of an exemption, without any explanation as to why that was. Here, the officer examined the militating factors and, for the most part, raised concerns and reasons why they were not sufficient to cause the necessary hardship required to justify an exemption. 41 I say for the most part because there is one aspect of the Applicant’s criminal conduct which the Officer did not consider. The Elizabeth Fry Society is an agency that provides counselling in the Shoplifting and Fraud Group Program and in the Healing from Abuse Group Program. It is a credible and well regarded agency as evidenced by the nature of the services it provides dealing with curtailing criminal conduct and helping heal victims of trauma arising from abuse. 42 The Applicant has been attending counselling and the Elizabeth Fry Society provided three letters on August 1, 2007, December 4, 2008 and December 7, 2009, which are relevant to the Officer’s assessment of the Applicant’s mental health and criminal conduct. 43 In an overly brief summary, the letters explain that the Applicant’s shoplifting behaviour is consistent with what is referred to as ‘kleptoma- nia’ which is defined as a mental illness resulting from physical and emotional violence and trauma. 44 Shoplifting, in the writer’s view, must be understood as an illness which needs be addressed though trauma counselling which provides coping strategies for shoplifting behaviour. The writer, the Applicant’s counsellor, indicates that shoplifting behaviour tends to increase during stressful periods and that the Applicant’s involvement with shoplifting decreases with counselling. The writer indicates the Applicant will need counselling for trauma over several years. 45 Finally, the writer inquired of the Trinidadian Consulate in Canada as well as the Social Services Ministry in Trinidad. Neither agency was able to provide evidence of the availability of such counselling in Trinidad. 46 I do not consider this aspect to be so much as relating to the assess- ment of H&C factors in the context of criminal behaviour since the Ap- plicant must be responsible for law-abiding conduct, but I do consider it a necessary consideration when addressing the question of the Appli- cant’s mental health issues. 47 The Officer does, in regards to her personal relationships and ties to Trinidad and Canada, find that it would be reasonable to assume the Ap- plicant would have the financial and emotional support, and assistance of Shah v. Canada (MCI) Leonard S. Mandamin J. 281

her family in Canada, to assist her in obtaining employment and finding a place to live. The officer also concluded there were reasonable grounds to believe the Applicant could continue her relationship with her children and grandchildren, albeit a long-distance one. Also, in considering the risk of returning to Trinidad and the associated medical concerns, the Officer examined the specific issues raised by the applicant: the danger posed by her ex-husband and local criminal elements; the Applicant’s possible isolation; and the Applicant’s medical condition. 48 The Officer addressed each issue and provided reasons why these constituted insufficient hardship. Finally, the Officer weighed the posi- tive militating factors against the Applicant’s criminal record and inabil- ity to stop her behaviour, despite seeking treatment and therapy. 49 In Siddiqui, Justice Mactavish cautioned that “[t]his Court must be careful not to read a decision such as this microscopically, or to take a word or sentence out of context, in an effort to identify an error on the part of the officer”: Siddiqui, supra, at para 12. 50 I conclude the Officer’s decision informs the Applicant why her ap- plication was refused.

2. In light of the evidence submitted by the Applicant, was the Officer’s decision to refuse the Applicant’s H&C application reasonable? 51 The Applicant submits that the Officer ignored medical evidence in- dicating that a removal from Canada increased the likelihood that the Applicant would commit suicide. The Applicant refers here to three re- ports from her psychiatrist and a letter from her treating physician, in- cluded in her H&C application, which all reported on her condition and the impact of a removal. 52 The Respondent maintains that the medical evidence was considered appropriately, that the issue of whether a risk of suicide existed indepen- dently of any issues of access to medical treatment in Trinidad is not reflected in the record, and accuses the Applicant of attempting to change the focus of the submissions in the face of the Officer’s reasons. 53 The Respondent submits that the Officer was entitled to find that, with no evidence showing the Applicant could not access medical treat- ment, there was no basis for finding that there was undue hardship. The Respondent further contends that the Applicant has not shown any seri- ous issue that would put in doubt the reasonability of the Officer’s deci- sion and that evidence in the form of letters from family members do not require the Officer to approve the application. 282 IMMIGRATION LAW REPORTER 3 Imm. L.R. (4th)

54 The Applicant’s psychiatrist strongly recommended that removal not take place “as this would have a quite detrimental effect on [the Appli- cant] and would only lead to further deterioration and suffering”, that “the treatment that [the Applicant] has been receiving here would be in- terrupted drastically and therefore [the] patient is at much higher risk to act out or resort to attempting suicide”, and “that return to Trinidad where she will not have as much support by family as she does here and after so many years living in Canada, would be detrimental to her mental health being and be causing deterioration of her symptoms and increas- ing the risk of suicidal behaviour”. The Applicant’s treating physician also indicated “that if she were to be forced to return to Trinidad, her psychiatric condition would worsen, due to the stress and trauma precipi- tated by the deportation”. 55 The Officer’s reasons show that the risk of suicide was raised. The Officer noted Applicant’s counsel’s submission that issues of hardship, including “the effect on [the Applicant’s] mental health possibly leading to a suicide,” were serious enough to have warranted a stay of removal in the Federal Court until the H&C was decided. When addressing this par- ticular issue of hardship, the Officer acknowledged the Applicant’s diag- nosis for anxiety and major depression and mentioned the concerns ex- pressed by the Applicant, her family, and medical practitioners as to her removal, given that it would prevent her from continuing the counselling and medical care she had been receiving here. 56 The Officer’s very brief subsequent analysis, however, is that there is no evidence to substantiate the same counselling and medication would not be available to her in Trinidad or that she could not access it. The Officer concluded that her medical practitioners are not experts “in terms of the nature/quality and type of medical services available in Trinidad”. Nowhere does the Officer consider the risk of suicide from the removal itself and from the Applicant’s physical separation from her family. 57 In similar circumstances in Davis v. Canada (Minister of Citizenship & Immigration), 2011 FC 97, [2011] F.C.J. No. 114 (F.C.) [P.M.D.], the Court found it was not sufficient for the officer to simply look at the availability of mental health care in the target country. The officer was still required to determine whether putting the applicant through removal and its impact amounted to undue, undeserved or disproportionate hard- ship. Failing to do so rendered the decision unreasonable. 58 I find that in these circumstances where the Officer was faced with evidence of a risk of suicide resulting from the removal itself, it was not Shah v. Canada (MCI) Leonard S. Mandamin J. 283

sufficient for the Officer to simply examine the availability of medical services in the destination country. The Officer was also required to de- termine whether putting the Applicant through removal and its potential for possibly leading to suicide amounted to undue, undeserved or dispro- portionate hardship. I adopt the Court’s reasoning in P.M.D. and find that the Officer’s failure to do so renders the decision unreasonable. 59 In generally considering the Applicant’s mental health issues, the Of- ficer said she was not provided with any evidence that the same type of counselling and medication would not be available or was inaccessible in Trinidad and that her current physicians are not experts as to the medical services available in Trinidad. 60 Contrary to the Officer’s assertion, she had before her the letters of the Elizabeth Fry Society advising that the Society had inquired and neither the Trinidadian Consulate nor the Social Services Ministry could advise that the same counselling would not be available for the Applicant in Trinidad. 61 The Applicant also argues against the Officer’s finding that the Ap- plicant would be able to apply her skills and work experiences acquired in Canada to obtain employment in Trinidad. The Applicant asserts that in light of the Ontario Social Benefits Tribunal’s finding that the Appli- cant was a person with substantial mental impairment and disability, the Officer’s finding is unreasonable. 62 I agree that it was unreasonable for the Officer to state that the Appli- cant would “be able to apply her skills and work experiences acquired in Canada to obtain employment and find housing in Trinidad”. In terms of skills and work experiences, the Officer is presumably referring to the one year the Applicant spent working at Sunshine Carpet and the four years she spent as a painter for a manufacturer and distributor of indus- trial safety products. Meanwhile, the Officer ignored clear evidence dem- onstrating that the Applicant’s disability severely limited her ability to work, including the following findings from the Ontario Social Benefits Tribunal: [The Applicant’s] ability to function in the workplace is substantially restricted by [her] substantial impairments: She is afraid to walk outside for fear of falling, she is constantly feeling tired, and help- less. She has tried working, but her employer let her go as she was told she works too slowly. She has no computer knowledge and has not been able to learn to use a computer because of her cognitive impairments. She has difficulty reading, and cannot sit for more than 284 IMMIGRATION LAW REPORTER 3 Imm. L.R. (4th)

half an hour before she has to move around. She is subject to fre- quent crying spells. The Ontario Social Benefits Tribunal found the Applicant was a person with substantial impairment such that she was unfit to work and awarded her disability benefits. 63 The Applicant’s treating physician also noted that “due to her medical conditions, I don’t believe she would be able to find work in Trinidad”. Yet somehow this evidence was also completely ignored by the Officer. 64 Finally, the Applicant contests the finding that “it would be reasona- ble to assume that at least some of the acquaintances that [the Applicant] had and some of the families in the area she grew up in would continue to reside in that area of Trinidad”. The Applicant refers to two letters from her daughters, evidence before the Officer, which clearly indicated the Applicant had no family or friends left in Trinidad. 65 There is no evidence for the Officer’s finding when addressing the Applicant’s potential isolation in Trinidad. It must be remembered that she left Trinidad at the age of 18, some 38 years ago. What neighbourhood or area did the Applicant grow up in? Is the neighbourhood still there? Do the same families or friends live in that area? If so, would they be in a position or willing to help? There simply is no evidence. In coming to this dubious conclusion, the Officer ignored unequivocal testimony from the two daughters that the Applicant had no family or friends left in Trinidad. 66 Considering the above, I find the Officer’s decision with respect to availability of counselling and isolation to be unreasonable.

3. Did the Officer err by applying the criteria for PRRA applications, as set out in sections 96 and 97 of IRPA, to the applicant’s H&C application? 67 The Applicant submits the Officer applied the wrong legal criteria, that of sections 97 and 122 of IRPA, which it states has been deemed a reviewable error by this Court. 68 The Respondent is of the view the Officer’s analysis was appropriate and did not employ the precise language of the PRRA process, but was simply characterizing the nature of the hardship the Applicant claimed she would face. The Respondent submits the Officer correctly considered evidence of risk so as to determine whether the Applicant would suffer hardship in Trinidad and merely noted that the Applicant had not claimed Shah v. Canada (MCI) Leonard S. Mandamin J. 285

she was at risk of any specific criminal elements and only expressed con- cerns about general country conditions. 69 The law surrounding this issue is set out by Justice Pinard in Reba¨ı v. Canada (Minister of Citizenship & Immigration), 2008 FC 24, 67 Imm. L.R. (3d) 191 (F.C.) at paragraph 7, [Reba¨ı]: When performing a PRRA analysis, the question to be answered is whether the applicant would personally be subjected to a danger of torture or to a risk to life or to cruel and unusual treatment or punish- ment (Sahota v. Canada (Minister of Citizenship & Immigration), 2007 FC 651, [2007] F.C.J. No. 882 (F.C.)). On an H&C application, the underlying question is whether the requirement that the applicant apply for permanent residence from outside of Canada would cause the applicant unusual and undeserved or disproportionate hardship (Sha’er v Canada (Minister of Citizenship & Immigration) (2007), 60 Imm. L.R. (3d) 189, [2007] F.C.J. No. 297 (F.C.)). The risk to the applicant must be assessed as one factor in that determination (Sahota, supra). While the officer can adopt the factual findings from the PRRA analysis, the officer must consider these factors in light of the lower threshold of risk applicable to H&C decisions, of “whether the risk factors amount to unusual, undeserved or disproportionate hardship” (Gallardo v. Canada (Minister of Citizenship & Immigra- tion), 2007 FC 554, [2007] F.C.J. No. 749 (F.C.)). 70 The question at hand is whether the Officer applied the correct analy- sis to the Applicant’s concerns over crime rates in Trinidad, specifically in Port of Spain. On this point, the Officer undertook the following anal- ysis: The applicant has submitted concerns over the rising crime rate in Trinidad; however she has provided insufficient objective evidence that she would be personally targeted by the criminal elements upon her return to Trinidad. I acknowledge that there are problems of crime in Trinidad, however, in this regard I find the situation and hardship the applicant fears is faced generally by other individuals in the country. ... The applicant has not resided in Trinidad for thirty-seven years and has provided insufficient evidence that she would be personally threatened or targeted by criminal elements upon her return to Trini- dad. Although the applicant’s removal to Trinidad would subject her personally to some hardship and while I acknowledge country condi- tions are not always favourable, I do not find that this would amount to a hardship that is unusual and undeserved, or disproportionate. I 286 IMMIGRATION LAW REPORTER 3 Imm. L.R. (4th)

find that the applicant has provided insufficient evidence to indicate that returning to Trinidad would subject her to a risk that would amount to unusual and undeserved, or disproportionate hardship [emphasis added]. 71 It is clear the Officer’s final conclusion applied the appropriate test. However, the Officer’s analysis leading up to this conclusion is problem- atic and focused on the personal risk faced by the Applicant. The analy- sis begins by immediately noting the Applicant provided insufficient ob- jective evidence that she would be personally targeted. It then recognized problems with crime in Trinidad, but found that this hardship is faced generally by other individuals in the country. After citing a country re- port, the Officer again stated the Applicant had provided insufficient evi- dence that she would be personally threatened or targeted. This is the only analysis conducted before the Officer stated her final conclusion. 72 The Officer set aside all of the country conditions and dismissed rele- vant facts indicative of hardship by incorrectly applying a standard which required the Applicant to show that she would be personally targeted or threatened. This Court has determined such an approach to be incorrect and reviewable: see Sahota v. Canada (Minister of Citizenship & Immi- gration), 2007 FC 651, [2007] F.C.J. No. 882 (F.C.) [Sahota ]; Sha’er, supra. 73 I find the Officer applied a higher standard than appropriate for H&C decisions by incorrectly requiring the Applicant to establish a personal risk beyond that faced by other individuals in Trinidad. The test of risk causing unusual, undeserved or disproportionate hardship is not limited to personal risks to an Applicant’s life or safety, and the Officer failed to properly consider whether the overall problem of criminality constituted unusual and undeserved, or disproportionate hardship in the circum- stances. This constitutes a reviewable error: Aboudaia c. Canada (Ministre de la Citoyennet´e & de l’Immigration), 2009 FC 1169 (F.C.) at para 17, Reba¨ı, supra; Sahota, supra; Sha’er, supra.

4. Did the Officer err in failing to consider the Applicant’s request for a TRP? 74 The Applicant submits that a TRP may be appropriate where an H&C application has been refused, but an officer feels the applicant should be allowed to remain in Canada temporarily, for example, so as to apply for a pardon for a criminal conviction as noted in CIC Inland Processing Manual #5 at section 5.22. The Applicant notes she requested a TRP and Shah v. Canada (MCI) Leonard S. Mandamin J. 287

would have been eligible for a pardon within the maximum three year time period of a TRP. 75 The Respondent contends that there was no basis for the issuance of a TRP as the Applicant did not receive first stage approval of her H&C application. 76 However, the CIC Inland Processing Manual #5 indicates that the is- suance of a TRP may still be appropriate in situations where an H&C application has been refused, for example in cases where an officer “does not believe that there are sufficient grounds to grant an exemption under H&C but feels the applicant should be allowed to remain in Canada tem- porarily, perhaps to apply for a pardon for a criminal conviction”. 77 This Court has established that where an applicant makes a TRP re- quest, it must be considered and that a failure to do so is a reviewable error. In Japson v. Canada (Minister of Citizenship & Immigration), 2004 FC 520 (F.C.) [Japson], the Court dealt with an officer’s failure to render a TRP decision following a rejection of an H&C application. Even if there is no basis for the issuance of a TRP, the officer should indicate the request was given consideration: Japson at para 25; Lee, supra, at paras 16 and 18; Dhandal, supra, at para 17. 78 The Applicant requested a TRP in her initial H&C application, in a letter to CIC dated December 23, 2009, and in her updated H&C submis- sions dated November 17, 2010. The Officer did not consider the request. 79 The Officer’s failure to consider the request for a TRP is a reviewable error.

Conclusion 80 For the above reasons, the matter must be remitted to a different im- migration officer for reconsideration, who must, in addition to consider- ing the H&C application, assess the Applicant’s TRP application. 81 The Application for judicial review is granted. 82 No question of general importance is certified.

Judgment THIS COURT’S JUDGMENT is that: 1. The Application for judicial review is granted. 2. The matter is remitted to a different immigration officer for recon- sideration, who must, in addition to considering the H&C applica- tion, assess the Applicant’s temporary resident permit application. 288 IMMIGRATION LAW REPORTER 3 Imm. L.R. (4th)

3. No question of general importance is certified. Application granted. Hamam v. Canada (MCI) 289

[Indexed as: Hamam v. Canada (Minister of Citizenship & Immigration)] Nazih Charif Hamam, Ghada Yehya Issa, Mohammad Ali Hamam, Sara Nazih Hamam, Applicants and The Minister of Citizenship and Immigration, Respondent Federal Court Docket: IMM-1443-11 2011 FC 1296 Leonard S. Mandamin J. Heard: October 31, 2011 Judgment: November 10, 2011 Immigration and citizenship –––– Admission — Application for temporary resident or immigrant visa — Inland applications — Undue hardship –––– Applicants were parents and four children — Mother and father were both citi- zens of Lebanon — Two oldest children were citizens of United States, and two youngest children were Canadian citizens — Applicants came to Canada in Jan- uary 2005, after failed claim for asylum in United States — After applicants’ claim for refugee status in Canada was refused, applicants submitted inland ap- plications for permanent residence on humanitarian and compassionate (H&C) grounds — On January 18, 2011, officer refused applicants’ applications — Ap- plicants brought application for judicial review — Application granted — Of- ficer made reviewable error in assessing risk — Officer confused duty given to her, which was to assess applicants’ allegation of risk under context of H&C application, not pre-removal risk assessment — Officer required applicants to demonstrate personal risk and found that they were unable to do so — This was wrong legal test for assessing risk in H&C application. Immigration and citizenship –––– Admission — Application for temporary resident or immigrant visa — Inland applications — Application of human- itarian and compassionate considerations — Establishment in Canada –––– Applicants were parents and four children — Mother and father were both citi- zens of Lebanon — Two oldest children were citizens of United States, and two youngest children were Canadian citizens — Applicants came to Canada in Jan- uary 2005, after failed claim for asylum in United States — After applicants’ claim for refugee status in Canada was refused, applicants submitted inland ap- plications for permanent residence on humanitarian and compassionate grounds — On January 18, 2011, officer refused applicants’ applications — Ap- plicants brought application for judicial review — Application granted — Of- ficer committed reviewable error by simply providing her conclusion without 290 IMMIGRATION LAW REPORTER 3 Imm. L.R. (4th)

providing reasons as to why she made conclusion she did — Officer found that applicants were established in Canada — Officer listed applicants’ positive es- tablishment evidence, like that applicants had contributed substantially to Mus- lim community and had close ties with numerous people in London — How- ever, officer failed to conduct any analysis and simply concluded that hardship applicants would face would not be unusual, undeserved or disproportionate. Immigration and citizenship –––– Admission — Application for temporary resident or immigrant visa — Best interests of child –––– Applicants were parents and four children — Mother and father were both citizens of Lebanon — Two oldest children were citizens of United States, and two youngest children were Canadian citizens — Applicants came to Canada in January 2005, after failed claim for asylum in United States — After applicants’ claim for refugee status in Canada was refused, applicants submitted inland applications for per- manent residence on humanitarian and compassionate grounds — On January 18, 2011, officer refused applicants’ applications — Applicants brought applica- tion for judicial review — Application granted — Officer was alert, alive and sensitive to interests of children — Officer took into account factors put forward by applicants, such as children’s age, their level of dependency on father and mother, and their establishment in Canada — Only issue that required closer look was whether officer erred by incorporating unusual and undeserved hard- ship threshold standards into assessment of best interest of children — It was unclear whether officer made statement about hardship as her conclusion in analysis of best interests of children or as her overall conclusion. Cases considered by Leonard S. Mandamin J.: Adu v. Canada (Minister of Citizenship & Immigration) (2005), 2005 FC 565, 2005 CarswellNat 1047, 2005 CF 565, 2005 CarswellNat 3483, [2005] F.C.J. No. 693 (F.C.) — considered Arulraj v. Canada (Minister of Citizenship & Immigration) (2006), 2006 Car- swellNat 1106, 2006 FC 529, 2006 CarswellNat 3740, 2006 CF 529, [2006] F.C.J. No. 672 (F.C.) — considered Dharamraj v. Canada (Minister of Citizenship & Immigration) (2006), 2006 FC 674, 2006 CarswellNat 1498, 294 F.T.R. 156, 2006 CarswellNat 3399, 2006 CF 674, [2006] F.C.J. No. 853 (F.C.) — followed Irimie v. Canada (Minister of Citizenship & Immigration) (2000), 2000 Car- swellNat 2810, 10 Imm. L.R. (3d) 206, [2000] F.C.J. No. 1906 (Fed. T.D.) — considered Kolosovs v. Canada (Minister of Citizenship & Immigration) (2008), 2008 CF 165, 2008 CarswellNat 745, 2008 FC 165, 2008 CarswellNat 281, 323 F.T.R. 181 (Eng.), [2008] F.C.J. No. 211 (F.C.) — followed Mikhno v. Canada (Minister of Citizenship & Immigration) (2010), 2010 CF 386, 2010 CarswellNat 2901, 2010 FC 386, 2010 CarswellNat 1304, [2010] F.C.J. No. 583 (F.C.) — referred to Hamam v. Canada (MCI) Leonard S. Mandamin J. 291

New Brunswick (Board of Management) v. Dunsmuir (2008), 372 N.R. 1, 69 Admin. L.R. (4th) 1, 69 Imm. L.R. (3d) 1, (sub nom. Dunsmuir v. New Brunswick) [2008] 1 S.C.R. 190, 844 A.P.R. 1, (sub nom. Dunsmuir v. New Brunswick) 2008 C.L.L.C. 220-020, D.T.E. 2008T-223, 329 N.B.R. (2d) 1, (sub nom. Dunsmuir v. New Brunswick) 170 L.A.C. (4th) 1, (sub nom. Dunsmuir v. New Brunswick) 291 D.L.R. (4th) 577, 2008 CarswellNB 124, 2008 CarswellNB 125, 2008 SCC 9, 64 C.C.E.L. (3d) 1, (sub nom. Dunsmuir v. New Brunswick) 95 L.C.R. 65, [2008] S.C.J. No. 9, [2008] A.C.S. No. 9 (S.C.C.) — referred to Ramirez v. Canada (Minister of Citizenship & Immigration) (2006), 60 Imm. L.R. (3d) 27, 2006 CF 1404, 2006 CarswellNat 6135, 304 F.T.R. 136 (Eng.), 304 F.T.R. 136, 2006 FC 1404, 2006 CarswellNat 3950, [2006] F.C.J. No. 1763 (F.C.) — considered Raudales v. Canada (Minister of Citizenship & Immigration) (2003), 2003 FCT 385, 2003 CarswellNat 809, 2003 CFPI 385, 2003 CarswellNat 3401, [2003] F.C.J. No. 532 (Fed. T.D.) — considered Tariq v. Canada (Minister of Citizenship & Immigration) (2005), 2005 FC 404, 2005 CarswellNat 758, 2005 CarswellNat 1869, 2005 CF 404, 44 Imm. L.R. (3d) 256, [2005] F.C.J. No. 502 (F.C.) — distinguished Uddin v. Canada (Minister of Citizenship & Immigration) (2002), 2002 CFPI 937, 2002 CarswellNat 3405, 2002 FCT 937, 2002 CarswellNat 2274, [2002] F.C.J. No. 1222 (Fed. T.D.) — distinguished Statutes considered: Immigration and Refugee Protection Act, S.C. 2001, c. 27 s. 11(1) — considered s. 25 — referred to s. 25(1) — considered

APPLICATION for judicial review of officer’s decision refusing applicants’ in- land applications for permanent residence on humanitarian and compassionate grounds.

Naseem Mithoowani, for Applicants Manuel Mendelzon, for Respondent

Leonard S. Mandamin J.:

1 Nazih Charif Hamam and his family apply for judicial review of the Immigration Officer’s decision dated January 18, 2011, refusing their ap- plications for permanent residence processed from within Canada on hu- manitarian and compassionate (H&C) grounds. 292 IMMIGRATION LAW REPORTER 3 Imm. L.R. (4th)

2 The Applicants are a family with four children. The father Nazih Charif Hamam (the “Father”) and mother Ghada Yehya Issa (the “Mother”) are both citizens of Lebanon. The two oldest children are citi- zens of the United States and the two youngest children are Canadian citizens. They currently live in London, Ontario. 3 The Officer addressed the three H&C grounds claimed by the Applicants. 1. Risk of returning to Lebanon 2. Establishment in Canada, and 3. The best interests of the children. 4 The Officer concluded in respect to each of these grounds that the Applicants had not satisfied the burden of proving they would face unu- sual, undeserved or disproportionate hardship if their H&C application was not accepted. 5 The Applicants submit the Officer erred by assessing the Applicants’ personal risk rather than the degree of hardship arising on return to Leba- non, failed to give adequate reasons for concluding the Applicants have not demonstrated sufficient establishment in Canada such that their re- moval would constitute unusual, undeserved or disproportionate hardship and applying the wrong test for the best interests of the children involved. 6 I have concluded the application for judicial review should be granted for the reasons that follow.

Background 7 The Applicants came to Canada in January 2005 after a failed claim for asylum in the United States. Their claim for refugee status in Canada was also unsuccessful. 8 In 2009, the Applicants submitted an inland application for perma- nent residence on the following H&C grounds: 1. risk of returning to Lebanon, 2. establishment in Canada, and 3. the best interests of the children. 9 The application was forwarded to the Montreal Pre-Removal Risk Assessment (PRRA) Office in October, 2010. An update to the file was provided by the Applicants in November, 2010. The Officer reviewed the Hamam v. Canada (MCI) Leonard S. Mandamin J. 293

Applicants’ file. She denied their H&C application giving reasons for the decision in a letter dated January 18, 2011.

Decision under Review 10 The reasons for decision were written in French. However, a trans- lated copy of the decision (by the Translation Bureau) is included within the Certified Tribunal Record (CTR). For the purposes of this decision, I will refer to the English translation. The Officer’s reasons address the three H&C grounds claimed by the Applicants.

Risk on returning to Lebanon 11 The Officer starts by briefly summarizing the allegations made by the Father in his refugee protection claim before the Refugee Protection Di- vision (RPD). The Officer notes the RPD determined that the Applicants had not established the alleged risks and were neither convention refu- gees nor persons in need of protection. The Officer also noted the Fed- eral Court dismissed their application for leave for judicial review of the RPD’s decision. 12 The Officer specifically states that her role was not to overturn the RPD’s determinations as the Officer was neither sitting in appeal of nor reviewing the RPD’s decisions. The Officer observes that in the current application, the Applicants repeated the allegations they made before the RPD and filed the evidence submitted to the RPD, the RPD’s reasons, the Father’s PIF, and other documentation. The Officer notes that the additional documentation did not confirm that the Applicants would be exposed to personal risks, but that it was consistent with the documentary sources the Officer had consulted. The Officer found that these sources indicated that although the security situation in Lebanon remained a cause for concern, the government and the armed forces continued to ful- fill their roles. 13 The Officer concludes by stating that the onus was on the Applicants to demonstrate a connection between the latent crisis situation in Leba- non and their personal risk and that the Applicants had failed to do so.

Establishment in Canada 14 The Officer begins by noting that the Applicants arrived in Canada on January 5, 2005. The Officer then goes on to discuss the Applicants’ En- glish ability. The Officer notes that the Father stated he speaks English and the Mother speaks, reads and writes English; however, the Officer 294 IMMIGRATION LAW REPORTER 3 Imm. L.R. (4th)

found no independent assessment documents were submitted to confirm any effort made to learn or know the language. The Officer was of the opinion the Father’s functioning in English is very likely limited. 15 The Officer notes the Father runs his own business as a carpet layer and that notices of assessment indicate a taxable income of about $14,000 over the previous three years. The Officer notes the same amount is shown on the Mother’s notices for 2008 and 2009. The Officer acknowledges that as the mother of young children, the Officer can un- derstand it having been more difficult for her to enter the job market. However, the Officer notes that while her IMM 5001 form states that she does not plan to work in the future, the Mother apparently accepted a full-time job to teach Arabic in London in 2010. 16 The Officer notes the Applicants’ file contains a bank statement esti- mating the Father’s assets at over $40,000 and numerous letters and at- testations from his business contacts touting his professionalism. The Of- ficer notes that this is a positive aspect in the application. 17 The Officer then lists some of the documentation submitted to demonstrate the Applicants’ establishment in Canada including: • A petition signed by 121 persons showing that the applicants have contributed substantially to the Muslim community and London, Ontario, as a whole; • Attestations from three Muslim organizations, including two schools, confirming this favourable opinion; • Approximately 30 letters of support from relatives and friends il- lustrating the merits and virtues of each of the Applicants. The Officer notes that these are all positive aspects. 18 Before making her conclusions on establishment, the Officer quotes from a decision of Justice Blanchard in Uddin v. Canada (Minister of Citizenship & Immigration), 2002 FCT 937, 116 A.C.W.S. (3d) 930 (Fed. T.D.) [Uddin], which defines the contours of analysis for the test for determining whether an applicant has demonstrated a sufficient de- gree of establishment in Canada such that their removal would constitute an unusual, undeserved or disproportionate hardship. 19 The Officer then concludes by stating that the Applicants will inevita- bly experience hardship if they return to Lebanon after such a long ab- sence if required to apply from Lebanon for a permanent resident visa. However, the Officer concludes that this hardship would not be unusual, undeserved or disproportionate. Hamam v. Canada (MCI) Leonard S. Mandamin J. 295

The best interests of the children 20 The Officer considered the best interests of the four children, two born in the United states and two in Canada, who were respectively nine, six, five and three years of age. The Officer acknowledged it was her duty to be alert, alive and sensitive to the interest of these children. 21 The Officer notes there is no doubt that these children were very at- tached to their three Canadian uncles and other relatives, including, in particular, their cousins. The Officer states that the Father and Mother have an even greater number of siblings in Lebanon and that no reason had been advanced that the children would not benefit from contact with their Lebanese close relatives whom they had not yet met. 22 The Officer comments on the Father and Mother’s assertions that the children speak little or no Arabic and cannot read or write in that lan- guage at all. The Officer finds these assertions confusing as the evidence appears to show that the Mother had given Arabic lessons to children on a volunteer basis. The Officer also notes the oldest child’s school report card emphasized his excellent academic performance in Arabic. The Of- ficer therefore did not rely on the Father and Mother’s statements with regard to the impact that the children’s leaving would have on their education. 23 The Officer notes nothing is specifically mentioned about the chil- dren’s health, particular needs or vital interests or the challenges they would face. 24 The Officer concluded the best interests of the children analysis by stating that the onus was on the Applicants to meet the burden of proving that there would be unusual, undeserved or disproportionate hardship in the event they returned to Lebanon and that the Applicants had failed to do so.

Relevant Legislation 25 The Immigration and Refugee Protection Act, SC 2001, c 27 pro- vides: 11. (1) A foreign national must, before entering Canada, apply to an officer for a visa or for any other document required by the regula- tions. The visa or document may be issued if, following an examina- tion, the officer is satisfied that the foreign national is not inadmissi- ble and meets the requirements of this Act. ... 296 IMMIGRATION LAW REPORTER 3 Imm. L.R. (4th)

25. (1) The Minister must, on request of a foreign national in Canada who is inadmissible or who does not meet the requirements of this Act, and may, on request of a foreign national outside Canada, ex- amine the circumstances concerning the foreign national and may grant the foreign national permanent resident status or an exemption from any applicable criteria or obligations of this Act if the Minister is of the opinion that it is justified by humanitarian and compassion- ate considerations relating to the foreign national, taking into account the best interests of a child directly affected. 11. (1) L’´etranger doit, pr´ealablement a` son entr´ee au Canada, de- mander a` l’agent les visa et autres documents requis par r`eglement. L’agent peut les d´elivrer sur preuve, a` la suite d’un contrˆole, que l’´etranger n’est pas interdit de territoire et se conforme a` la pr´esente loi. ... 25. (1) Le ministre doit, sur demande d’un etranger´ se trouvant au Canada qui est interdit de territoire ou qui ne se conforme pas a` la pr´esente loi, et peut, sur demande d’un etranger´ se trouvant hors du Canada, etudier´ le cas de cet etranger;´ il peut lui octroyer le statut de r´esident permanent ou lever tout ou partie des crit`eres et obligations applicables, s’il estime que des consid´erations d’ordre humanitaire relatives a` l’´etranger le justifient, compte tenu de l’int´erˆet sup´erieur de l’enfant directement touch´e.

Issues 26 I consider the issues in this case to be: 1. Did the Officer err in her assessment of risk or hardship? 2. Were the Officer’s reasons in her assessment of the Applicants’ establishment in Canada inadequate? 3. Did the Officer err in her assessment of the best interests of the children?

Standard of Review 27 The appropriate standard of review for a decision on H&C grounds is reasonableness: Ramirez v. Canada (Minister of Citizenship & Immigra- tion), 2006 FC 1404, 60 Imm. L.R. (3d) 27 (F.C.) [Ramirez] at para 30. Given the discretion an Immigration Officer has in a H&C application, a heavy burden rests on the Applicants to satisfy the Court that the deci- sion under section 25 requires the intervention of the Court: Mikhno v. Hamam v. Canada (MCI) Leonard S. Mandamin J. 297

Canada (Minister of Citizenship & Immigration), 2010 FC 386, [2010] F.C.J. No. 583 (F.C.) at para 25. 28 Errors of law and breaches of procedural fairness should be reviewed on a standard of correctness: New Brunswick (Board of Management) v. Dunsmuir, 2008 SCC 9, [2008] 1 S.C.R. 190 (S.C.C.) at paras 47-50.

Analysis 29 The Applicants allege errors by the Officer in her assessment under all three H&C grounds. The Applicants submit the Officer erred by fail- ing to assess their new allegation of risk from Hezbollah and also by requiring personalized risk to the Applicants rather than assessing risk informed by hardship on removal to Lebanon. The Applicants also sub- mit the Officer failed to provide adequate reasons as to why the hardship they would experience if required to return to Lebanon, despite their ac- knowledged establishment in Canada, did not give rise to unusual or un- deserving and disproportionate hardship. Finally, the Applicants submit the Officer erred in applying the test of undeserved hardship in consider- ing the best interests of the children.

Risk 30 The Applicants submitted the Officer made two errors in her assess- ment of risk: the first is that the Officer failed to consider new evidence submitted by the Applicants that demonstrated fear of returning because of the presence of Hezbollah in the portion of Lebanon from which the Applicants are originally from; and the second is that the Officer applied the incorrect legal test for assessing risk in the context of an H&C application. 31 The Respondent argues the Officer was responsive to the submissions and evidence before her and that the Applicants essentially attempted to reargue their entire refugee claim in their H&C application. The Respon- dent submits the Applicants have an obligation to identify specific inci- dents or threats that they say give rise to unusual undeserving hardship and support them with evidence. This, the Respondent submits, the Ap- plicants did not do. 32 The Officer stated that her role was not to overturn the RPD’s deter- mination. She noted that the Applicants filed the evidence submitted to the RPD, the RPD reasons, and other material. The Applications submit- ted they feared return to Lebanon for the same reasons as they advanced in their refugee claim, but also because of risks posed by Hezbollah who 298 IMMIGRATION LAW REPORTER 3 Imm. L.R. (4th)

were hostile to western culture. The Applicants submit that the Officer analyzed their first allegation in her reasons but ultimately rejected the Applicants’ submissions by noting that her role was not to overturn the RPD’s determination. 33 The Applicants said their H&C application also contained allegations of risk that were not before the RPD which the Officer failed to expressly acknowledge. The Applicants’ submissions and documentary evidence made in support of the H&C application indicated that the Applicants, in addition to the risk set out in their failed refugee claim, feared to return to Lebanon due to the presence of Hezbollah in the area the Applicants are originally from. The Applicants stated in reply they provided a num- ber of articles highlighting the presence of Hezbollah in Lebanon to sup- port their claim. 34 The Applicants claim that the Officer completely failed to address the risk posed by Hezbollah. The Applicants argue that the Officer “specifi- cally indicated that the ‘only’ risks brought forward were those advanced at the RPD,” and since the second ground of risk was not before the RPD, the Officer must have failed to consider it. The Applicants rely on the following paragraphs from Tariq v. Canada (Minister of Citizenship & Immigration), 2005 FC 404, 44 Imm. L.R. (3d) 256 (F.C.) [Tariq], to show that this was a reviewable error: [20] The respondent submits that the Board is presumed to have con- sidered all of the evidence before it, and that I should therefore inter- pret the Board’s statement that the applicant had provided “no evi- dence” that they would be targeted by Sunni extremists if they were to return to Pakistan to mean that there was no such evidence that the Board found to be persuasive. [21] I do not accept this submission. It is true that a tribunal will ordinarily be presumed to have considered all of the evidence before it, even if no reference is specifically made to any given piece of evidence in the tribunal’s decision. That is not, however, the situation that we have here. This is not a case where the Board omitted to mention evidence. What we have here is a clear assertion by the Board that there was no evidence before it on a particular point when there was indeed just such evidence. [22] In my view, this leads to the inescapable inference that the evi- dence was overlooked by the Board. 35 After a careful review of the Officer’s decision, I am unable to con- clude that the Officer specifically indicated that she “only” took into ac- count the evidence and claims that were before the RPD. Hamam v. Canada (MCI) Leonard S. Mandamin J. 299

36 The Officer begins her assessment of risk by setting out the original allegations of fear put before the RPD and the history of the Father’s refugee claim. The Officer states that it was not her role to overturn the RPD’s determinations. The Officer then states: In this application for a visa exemption, the applicants repeated the allegations they made before the RPD. They filed the evidence that was submitted to the RPD, the RPD’s reasons, the principal appli- cant’s PIF, fifteen articles and general documents on the human rights situation in Lebanon and an attestation from a Muslim aid or- ganization, which echoes Ms. Issa’s statements regarding the risks she ascribes to her husband’s problems with the Syrian services and the security situation arising from the conflict with Israel. (Emphasis added). 37 While it is true that the Officer’s reasons do not mention the Appli- cants’ specific allegation of risk from Hezbollah, the Officer did take into account the evidence that was not before the RPD. The Officer acknowl- edged the Applicants’ submission of “fifteen articles and general docu- ments on the human rights situation in Lebanon and an attestation from a Muslim aid organization.” The inference can be made that these docu- ments were not part of the evidence before the RPD. Had they been it would have been unnecessary for the Officer to mention them specifically. 38 The situation in this case is not the same as in Tariq. The Officer did not claim that the only evidence she considered was that which had al- ready been before the RPD. While it would have been preferable the Of- ficer make specific mention of the second allegation made by the Appli- cants, it appears the Officer did at least acknowledge the evidence submitted on the risk from Hezbollah. 39 I conclude that the presumption that the Officer considered all the evidence remains in this case and that the Officer did consider the evi- dence of the Applicants’ fear of returning to Lebanon because of the presence of Hezbollah. No reviewable error was made by the Officer on this point. 40 The Applicants also submit the Officer misapplied the legal test from a PRRA analysis into this H&C application. The Applicants argue there is a clear difference between how an applicant’s claims of ‘risk’ in a PRRA should be treated, as opposed to ‘risk’ alleged in an H&C applica- tion. The Applicants submit a PRRA requires a decision maker to assess whether an applicant would be personally subjected to torture or to a risk 300 IMMIGRATION LAW REPORTER 3 Imm. L.R. (4th)

to life or cruel and unusual punishment. On the other hand, the Appli- cants submit an H&C application requires whether the applicant would face “unusual, undeserved or disproportionate hardship”. 41 The jurisprudence sets out that the risk in an H&C application is that of hardship which is different from the risk to be considered in a PRRA application. As Justice Montigny stated in Ramirez, “[i]t is beyond dis- pute that the concept of ‘hardship’ in an H&C application and the ‘risk’ contemplated in a PRRA are not equivalent and must be assessed accord- ing to a different standard.” 42 In Dharamraj v. Canada (Minister of Citizenship & Immigration), 2006 FC 674, 294 F.T.R. 156 (F.C.), Justice O’Keefe stated: [24] There is no dispute that there is a higher burden on the appli- cants to establish risk for the purposes of a PRRA than there is for H & C purposes. Consequently, there may be circumstances where risk would be relevant for an H & C application but not for a PRRA application. [25] In the present case, the officer merely adopted the assessment of risk made by the IRB and the PRRA officer without further analysis for the purposes of the H & C application. In my opinion, the officer made an unreasonable decision because she did not consider the risk factors in the context of the H & C application. 43 This appears to be the same situation as the case at bar. In this case, the Officer appears to have confused the duty given to her in this case which is to assess the Applicants’ allegation of risk under the context of an H&C application, not a PRRA. The Officer’s error is clear from the following passage from her decision: This set of documents does not confirm that the applicants will be exposed to personal risks, but it is consistent with the documentary sources I consulted. These sources indicate that although the security situation in Lebanon remains a cause for concern, the government and the armed forces continue to fulfill their riles. In particular, the attacks and assassinations more specifically targeted anti-Syrian figures or persons having a political profile. The onus was on the applicants to demonstrate a connection between the latent crisis situ- ation in Lebanon and their personal risk. They failed to do so. With regard to the first ground, I am therefore of the opinion that they would not face unusual and undeserved or disproportionate hardship in the event that they return to Lebanon to apply for permanent residence. [Emphasis added]. Hamam v. Canada (MCI) Leonard S. Mandamin J. 301

44 It is clear the Officer required the Applicants to demonstrate personal risk and found that the Applicants were unable to do so. However, this was the wrong legal test for an H&C application. The Officer does not express any appreciation that the test for hardship in an H&C context is different from the test for personalized risk in a PRRA assessment. 45 The Officer’s statement that the Applicants would not face unusual and undeserved or disproportionate hardship follows the Officer’s analy- sis with respect to personalized risk. The analysis is unreasonable as the wrong test was employed and it is not saved by reverting to a recitation of the proper H&C test in conclusion. 46 I conclude the Officer applied the incorrect legal test for assessing risk in an H&C application. This is a reviewable error.

Establishment in Canada 47 The Applicants submit that the Officer identifies numerous positive factors about the Applicants’ establishment in Canada but then con- cludes, without reasons, that the hardship they would experience if they return to Lebanon would not constitute unusual, undeserved or dispro- portionate hardship were they to return to Lebanon to apply for perma- nent residence. 48 The Applicants rely on Justice Mactavish’s analysis in Adu v. Canada (Minister of Citizenship & Immigration), 2005 FC 565 (F.C.) [Adu], where she held: [14] In my view, these ‘reasons’ are not really reasons at all, essen- tially consisting of a review of the facts and the statement of a con- clusion, without any analysis to back it up. That is, the officer simply reviewed the positive factors militating in favour of granting the ap- plication, concluding that, in her view, theses factors were not suffi- cient to justify the granting of an exemption, without any explanation as to why that is. This is not sufficient, as it leaves the applicants in the unenviable position of not knowing why their application was rejected. ... [20] In contrast, in this case, the officer reviewed the evidence of establishment in Canada offered by the applicants in support of their applications, and then simply stated her conclusion that this was not enough. We know from the officer’s reasons that she did not think that the applicants would suffer unusual, undeserved or dispropor- tionate harm if they were required to apply for permanent residence 302 IMMIGRATION LAW REPORTER 3 Imm. L.R. (4th)

from abroad. What we do not know from her reasons is why she came to that conclusion. [21] As a consequence, it is impossible to subject the officer’s rea- soning to a ‘somewhat probing’ analysis. 49 In this case, the Officer found that the Applicants were established in Canada. This is evident from the reasons where the Officer stated that the Applicants “have contributed substantially to the Muslim community and to the community in London, Ontario as a whole”, that “the applicants have close ties with numerous people in London”, and that “their efforts convey a willingness to integrate that is directed primarily towards their own community in this city”. 50 The Respondent submits that the Officer’s decision regarding the Ap- plicants’ establishment in Canada was reasonable. The Respondent ar- gues the Applicants have failed to show how their evidence of establish- ment in the community creates hardship on return to Lebanon that rises to the level of unusual, undeserved, or disproportionate. 51 The Respondent points out that the Officer’s reasons refer to the Ud- din case as having informed her analysis, in particular the included quote from Irimie v. Canada (Minister of Citizenship & Immigration) (2000), 10 Imm. L.R. (3d) 206, [2000] F.C.J. No. 1906 (Fed. T.D.), which stated: I return to my observation that the evidence suggests that the appli- cants would be a welcome addition to the Canadian community. Un- fortunately, that is not the test. To make it the test is to make the H & C process an ex post facto screening device which supplants the screening process contained in the Immigration Act and Regulations. This would encourage gambling on refugee claims in the belief that if someone can stay in Canada long enough to demonstrate that they are the kind of persons Canada wants, they would be allowed to stay. The H & C process is not designed to eliminate hardship; it is de- signed to provide relief from unusual, undeserved or disproportionate hardship. There is no doubt that the refusal of an applicants’ H & C application will cause hardship but, given the circumstances of the applicants’ presence in Canada and the state of the record, it is not unusual, undeserved or disproportionate hardship. ... 52 In Raudales v. Canada (Minister of Citizenship & Immigration), 2003 FCT 385, 121 A.C.W.S. (3d) 932 (Fed. T.D.) at paragraph 19, Justice Dawson, now of the Federal Court of Appeal, held that since establish- ment is a relevant factor to consider when assessing an application on H&C grounds, in the absence of a proper assessment of establishment in Canada, a proper determination cannot be made as to whether an appli- Hamam v. Canada (MCI) Leonard S. Mandamin J. 303

cant would suffer hardship if required to apply for permanent residence abroad. 53 The Applicants, as a family, were well established in Canada. This was evident where the Officer stated that the Applicants “have contrib- uted substantially to the Muslim community and to the community in London, Ontario as a whole”, that “the applicants have close ties with numerous people in London”, and that “their efforts convey a willingness to integrate that is directed primarily towards their own community in this city”. 54 The situation in the case at bar is strikingly similar to the case before Justice Mactavish in Adu. The only significant difference is the Officer’s reliance on Uddin to the effect that the H&C process is not designed to eliminate hardship, but rather is designed to provide relief from unusual, undeserved or disproportionate hardship. However, Uddin does not stand for the proposition that positive evidence of establishment is not a signif- icant factor to be considered and weighed in analysis. 55 The Officer was correct in relying on Uddin as the appropriate legal framework in which to ground her analysis. The problem is that the Of- ficer listed the Applicants’ positive establishment evidence, failed to con- duct any analysis, and simply concluded that the hardship the Applicants would face would not be unusual, undeserved or disproportionate. 56 I agree with the Applicants that the Officer simply provided her con- clusion without providing reasons as to why she made the conclusion she did. This is also a reviewable error.

Best Interests of the Children 57 The Applicants submit that the Officer only dealt with two concerns regarding the best interests of the children: 1. the children would be losing their close relationship with their ex- tended family in Canada, and 2. the children’s education would suffer due to their alleged diffi- culty with the Arabic language. 58 The Applicant submits that the Officer erred in not considering the best interests of a disabled nephew who had formed a close relationship with the Applicants. The Applicants further submit that the Officer again erred by applying the wrong approach to the best interests of the child analysis by finding the Applicants have not discharged the onus to prove there would be unusual, undeserved or disproportionate hardship in the 304 IMMIGRATION LAW REPORTER 3 Imm. L.R. (4th)

event they returned to Lebanon to apply for permanent residence in Can- ada. They say the best interest of a child analysis does not engage a ques- tion of finding underserved hardship. 59 The Respondent submits that the Officer noted the Applicants’ chil- dren would suffer dislocation if they had to travel to Lebanon with their parents, but also found that they would likely have significant family support in Lebanon. The Respondent interprets the Officer’s conclusion that the interest of the children did not establish that the family would face unusual, undeserved or disproportionate hardship if they have to live in Lebanon. This, the Respondent submits, was a correct application of the H&C test. 60 This Court’s decision in Kolosovs v. Canada (Minister of Citizenship & Immigration), 2008 FC 165, 323 F.T.R. 181 (Eng.) (F.C.), sets out some of the factors to be taken into account when assessing the chil- dren’s best interests. 61 Upon a review of the Officer’s decision, it does appear that the Of- ficer was alert, alive and sensitive to the interests of the children. The Officer took into account the factors put forward by the Applicants such as the children’s age, their level of dependency on the Father and Mother, and their establishment in Canada. 62 While the Applicants submit that the Officer did not take into account the best interests of the nephew who has special needs and is close to the Applicants, the evidence does not, in my view, establish a level of bond- ing or dependency that requires the Officer to include that relationship in the H&C consideration of the best interests of the child. 63 The only issue I believe that would require a closer look is whether the Officer erred by incorporating the “unusual and undeserved hard- ship” threshold standards into the assessment of the best interest of the child. As Justice Barnes stated in Arulraj v. Canada (Minister of Citizenship & Immigration), 2006 FC 529, 148 A.C.W.S. (3d) 305 (F.C.), at paragraph 14: ... The similar terms found in the IP5 Guidelines of “unusual”, “un- deserved” or “disproportionate” are used in the context of consider- ing an applicant’s H & C interests in staying in Canada and not hav- ing to apply for landing from abroad. It is an error to incorporate such threshold standards into the exercise that aspect of the H & C discretion which requires that the interests of the children be weighed. Hamam v. Canada (MCI) Leonard S. Mandamin J. 305

64 The Officer’s wording is unclear. The Officer said the onus was on the Applicants to meet the burden of proving there would be unusual, underserved or disproportionate hardship in the event of return to Leba- non. The Officer both concludes the paragraph on the best interests of the children and ends the overall analysis at this point. It is unclear whether the Officer makes the statement about hardship as her conclusion in the analysis of the best interests of the children or as her overall conclusion. 65 Given that the Officer’s decision is an English translation from French, I do not think it wise to decide this issue on the material and submissions before me. As I have already found that the Officer’s deci- sion should be set aside, I need not decide this question.

Conclusion 66 The application for judicial review is granted. The matter is referred back for redetermination by a different decision maker. 67 No question of general importance is certified.

Judgment THIS COURT’S JUDGMENT is that: 1. The application for judicial review is granted. The matter is re- ferred back for redetermination by a different decision maker. 2. No serious question of general importance is certified. Application granted. 306 IMMIGRATION LAW REPORTER 3 Imm. L.R. (4th)

[Indexed as: Khan v. Canada (Minister of Citizenship & Immigration)] MD Golam Azam Khan, Applicant and The Minister of Citizenship and Immigration, Respondent Federal Court Docket: IMM-994-11 2011 FC 1314 James Russell J. Heard: October 5, 2011 Judgment: November 16, 2011 Immigration and citizenship –––– Admission — Appeals and judicial re- view — Judicial review — Standard of review. Immigration and citizenship –––– Admission — Appeals and judicial re- view — Judicial review — Duty of fairness –––– Applicant, citizen of Ban- gladesh, received job offer in Canada and applied for permanent resident visa under Federal Skilled Worker Program (FSWP) — Immigration Officer found applicant’s MBA in marketing from Green University in Bangladesh and total of 16 years of full-time education qualified him for 22 points for education — Of- ficer awarded total of 65 points and refused applicant’s application — Applicant applied for judicial review — Application granted — Officer found that appli- cant had Master’s degree, but only 16 years of full-time education, which only entitled applicant to 22 points — Applicant submitted that he completed 17 years of full-time education in addition to Master’s degree, entitling him to full 25 points — No one knew how officer calculated number of years of full-time education, so it was not possible to understand this aspect of decision or to de- termine whether it was reasonable — This was issue of procedural fairness — Officer did not need to provide elaborate reasons, but it was necessary to know basis of calculation — Without any explanation for calculation, reasons were to- tally inadequate on that issue — Matter to be returned for reconsideration. Immigration and citizenship –––– Admission — Immigrants — Skilled workers — Categories (point system) — Education — Equivalency –––– Calculation of years of education — Applicant, citizen of Bangladesh, received job offer in Canada and applied for permanent resident visa under Federal Skilled Worker Program (FSWP) — Immigration Officer found that applicant’s MBA in marketing from Green University in Bangladesh and total of 16 years of full-time education qualified him for 22 points for education — Officer awarded total of 65 points and refused applicant’s application — Applicant ap- Khan v. Canada (MCI) 307

plied for judicial review — Application granted — Officer found that applicant had Master’s degree, but only 16 years of full-time education, which only enti- tled applicant to 22 points — Applicant submitted that he completed 17 years of full-time education in addition to Master’s degree, entitling him to full 25 points — No one knew how officer calculated number of years of full-time edu- cation, so it was not possible to understand this aspect of decision or to deter- mine whether it was reasonable — This was issue of procedural fairness — Of- ficer did not need to provide elaborate reasons, but it was necessary to know basis of calculation — Without any explanation for calculation, reasons were to- tally inadequate on that issue — Matter to be returned for reconsideration. Cases considered by James Russell J.: Abdullah v. Canada (Minister of Citizenship & Immigration) (2006), 2006 Car- swellNat 3096, 2006 FC 1185, 2006 CF 1185, 2006 CarswellNat 4768, (sub nom. bin Abdullah v. Canada (Minister of Citizenship and Immigration)) [2006] F.C.J. No. 1482 (F.C.) — referred to Bellido v. Canada (Minister of Citizenship & Immigration) (2005), 2005 Car- swellNat 889, 2005 FC 452, [2005] F.C.J. No. 572 (F.C.) — considered C.U.P.E. v. Ontario (Minister of Labour) (2003), 2003 CarswellOnt 1803, 2003 SCC 29, 2003 CarswellOnt 1770, 2003 C.L.L.C. 220-040, [2003] 1 S.C.R. 539, (sub nom. Canadian Union of Public Employees v. Ontario (Minister of Labour)) 173 O.A.C. 38, (sub nom. Canadian Union of Public Employees v. Ontario (Minister of Labour)) 66 O.R. (3d) 735 (note), 226 D.L.R. (4th) 193, (sub nom. Canadian Union of Public Employees v. Ontario (Minister of Labour)) 304 N.R. 76, 50 Admin. L.R. (3d) 1, REJB 2003-41592, [2003] S.C.J. No. 28 (S.C.C.) — considered Choi v. Canada (Minister of Citizenship & Immigration) (2008), 2008 Car- swellNat 1372, 2008 FC 577, [2008] F.C.J. No. 734 (F.C.) — considered Eslamieh v. Canada (Minister of Citizenship & Immigration) (2008), 2008 Car- swellNat 1701, 2008 FC 722, 2008 CF 722, 2008 CarswellNat 2799, [2008] F.C.J. No. 909 (F.C.) — referred to Fernandes v. Canada (Minister of Citizenship & Immigration) (2008), 2008 CarswellNat 465, 71 Imm. L.R. (3d) 134, 2008 FC 243, [2008] F.C.J. No. 302 (F.C.) — considered Healey v. Canada (Minister of Citizenship & Immigration) (2009), 2009 Car- swellNat 891, 2009 FC 355, 80 Imm. L.R. (3d) 138, 342 F.T.R. 61 (Eng.), [2009] F.C.J. No. 439 (F.C.) — referred to Jogiat v. Canada (Minister of Citizenship & Immigration) (2009), 85 Imm. L.R. (3d) 297, 2009 CarswellNat 3844, 2009 FC 815 (F.C.) — referred to Khosa v. Canada (Minister of Citizenship & Immigration) (2009), 82 Admin. L.R. (4th) 1, 2009 SCC 12, 2009 CarswellNat 434, 2009 CarswellNat 435, 304 D.L.R. (4th) 1, 77 Imm. L.R. (3d) 1, 385 N.R. 206, (sub nom. Canada (Citizenship & Immigration) v. Khosa) [2009] 1 S.C.R. 339, [2009] S.C.J. No. 12 (S.C.C.) — referred to 308 IMMIGRATION LAW REPORTER 3 Imm. L.R. (4th)

Kniazeva v. Canada (Minister of Citizenship & Immigration) (2006), 2006 Car- swellNat 472, 2006 CF 268, 2006 CarswellNat 2836, 52 Imm. L.R. (3d) 298, 2006 FC 268, 288 F.T.R. 282 (Eng.), [2006] A.C.F. No. 336, [2006] F.C.J. No. 336 (F.C.) — considered Lackhee v. Canada (Minister of Citizenship & Immigration) (2008), 76 Imm. L.R. (3d) 283, 2008 CF 1270, 2008 CarswellNat 5316, 337 F.T.R. 299 (Eng.), 2008 FC 1270, 2008 CarswellNat 4270, [2008] F.C.J. No. 1615 (F.C.) — considered Miranda v. Canada (Minister of Citizenship & Immigration) (2010), 2010 Car- swellNat 1041, 2010 FC 424, 2010 CarswellNat 2073, 2010 CF 424 (F.C.) — considered New Brunswick (Board of Management) v. Dunsmuir (2008), 372 N.R. 1, 69 Admin. L.R. (4th) 1, 69 Imm. L.R. (3d) 1, (sub nom. Dunsmuir v. New Brunswick) [2008] 1 S.C.R. 190, 844 A.P.R. 1, (sub nom. Dunsmuir v. New Brunswick) 2008 C.L.L.C. 220-020, D.T.E. 2008T-223, 329 N.B.R. (2d) 1, (sub nom. Dunsmuir v. New Brunswick) 170 L.A.C. (4th) 1, (sub nom. Dunsmuir v. New Brunswick) 291 D.L.R. (4th) 577, 2008 CarswellNB 124, 2008 CarswellNB 125, 2008 SCC 9, 64 C.C.E.L. (3d) 1, (sub nom. Dunsmuir v. New Brunswick) 95 L.C.R. 65, [2008] S.C.J. No. 9, [2008] A.C.S. No. 9 (S.C.C.) — followed Persaud v. Canada (Minister of Citizenship & Immigration) (2009), 2009 FC 206, 2009 CarswellNat 437, [2009] F.C.J. No. 229 (F.C.) — considered Silva v. Canada (Minister of Citizenship & Immigration) (2007), 2007 Car- swellNat 2989, 2007 CF 733, 2007 FC 733, 2007 CarswellNat 1988, 63 Imm. L.R. (3d) 176, [2007] F.C.J. No. 981 (F.C.) — considered Sketchley v. Canada (Attorney General) (2005), 2006 C.L.L.C. 230-002, 2005 CAF 404, 2005 CarswellNat 5119, [2006] 3 F.C.R. 392, 2005 FCA 404, 2005 CarswellNat 4194, 344 N.R. 257, 44 Admin. L.R. (4th) 4, 56 C.H.R.R. D/490, 263 D.L.R. (4th) 113, [2005] F.C.J. No. 2056 (F.C.A.) — considered Tokuda v. Canada (Minister of Citizenship & Immigration) (2011), 2011 FC 483, 2011 CarswellNat 1224, 2011 CarswellNat 2219, 2011 CF 483 (F.C.) — considered Tong v. Canada (Minister of Citizenship & Immigration) (2007), 2007 FC 165, 2007 CarswellNat 315, 2007 CarswellNat 4933, 2007 CF 165, 309 F.T.R. 259 (Eng.), [2007] F.C.J. No. 216 (F.C.) — referred to Xu v. Canada (Minister of Citizenship & Immigration) (2010), 366 F.T.R. 230 (Eng.), 2010 FC 418, 2010 CarswellNat 1051, 2010 CF 418, 2010 Car- swellNat 2139 (F.C.) — considered Statutes considered: Immigration and Refugee Protection Act, S.C. 2001, c. 27 Generally — referred to s. 12(2) — referred to Khan v. Canada (MCI) James Russell J. 309

s. 72(1) — pursuant to Regulations considered: Immigration and Refugee Protection Act, S.C. 2001, c. 27 Immigration and Refugee Protection Regulations, SOR/2002-227 Generally — referred to s. 75(1) — referred to s. 76(1) — considered s. 76(1)(a) — considered s. 76(1)(a)(i) — considered s. 76(1)(a)(vi) — considered s. 76(3) — considered s. 76(4) — considered s. 78(2)(e)(i) — considered s. 78(2)(e)(ii) — considered s. 78(2)(f) — considered s. 78(3) — considered s. 78(4) — considered s. 82(2) — considered

APPLICATION for judicial review of decision of Immigration Officer refusing applicant’s application for permanent resident visa.

Chayanika Dutta, for Applicant Nimanthika Kaneira, for Respondent

James Russell J.:

1 This is an application pursuant to subsection 72(1) of the Immigration and Refugee Protection Act, S.C. 2001, c. 27 (Act) for judicial review of the decision of the Immigration Officer (Officer) stationed at the High Commission of Canada in Singapore (High Commission). The decision, dated 14 January 2011 (Decision), refused the Applicant’s application for a permanent resident visa as a member of the Federal Skilled Worker class under subsection 75(1) of the Immigration and Refugee Protection Regulations SOR/2002-227 (Regulations).

Background 2 The Applicant is a citizen of Bangladesh. On 20 December 2009, he received on offer of employment (Offer) as vice-principal of A+ Acad- emy of Advancement in Toronto (Employer). On the strength of the Of- fer, the Applicant applied for permanent residence in Canada under the 310 IMMIGRATION LAW REPORTER 3 Imm. L.R. (4th)

Federal Skilled Worker program on 20 December 2009 (Application). The High Commission received the full Application on 12 February 2010. The Applicant’s wife and two sons were included in the Applica- tion, but they are not parties to this proceeding. 3 On 26 July 2010, Service Canada issued a positive Arranged Employ- ment Opinion (AEO) on the Offer. The AEO verified that the Offer was genuine and indicated that the position required written and spoken En- glish and Bengali. The High Commission received the AEO on 11 Au- gust 2011. 4 The Applicant submitted documentation to prove his educational qualifications. He also submitted form IMM-0008 — Application for Permanent Residence, including the Schedule 1 — Back- ground/Declaration form (Schedule 1). In Schedule 1, he indicated that he had completed: an MBA at Green University, Bangladesh; a Bachelor of Business Administration degree (BBA) also at Green University; a Civil Engineering Diploma from the Bogra Technical Institute; and pri- mary and secondary school. In total, he had completed ten years of pri- mary school, three years of secondary school, and five years of univer- sity, for a total of eighteen years. He also provided certified copies of his transcripts and certificates for his MBA, BBA, and Diploma and a certi- fied copy of his secondary school transcript. 5 The Applicant also indicated on form IMM5406 that he has a brother in Canada, MD Badsha Alam. 6 To prove his financial means, the Applicant completed and submitted form IMM0008 Schedule 3 — Economic Classes — Federal Skilled Workers. On this form he indicated that he had $33,300 in unencum- bered, transferable funds. He also submitted a personal balance sheet which indicated that he had a personal net worth of $193,967, including real property in Singapore worth approximately $160,000. On the bal- ance sheet, he indicated that $33,000 would accompany him to Canada immediately and $160,000 in assets was to be transferred later. 7 The Applicant also submitted an IELTS certificate. This certificate shows his English abilities rated at 4.5 in speaking, 4.5 in writing, 4.0 in reading, and 3.5 in listening. He had an overall score of 4.0. 8 The Officer was concerned that the Applicant’s Offer was not genu- ine and sent the Applicant letters requesting additional documentation from the Employer to address these concerns on 11August, 17 September and 26 November 2010. In response, the Employer sent tax documents, the faculty list for the A+ Academy of Advancement, and documents Khan v. Canada (MCI) James Russell J. 311

showing the addresses of its branches, the number of students enrolled, and the tuition fees charged. 9 After receiving documentation from the Employer, the Officer was still concerned about the genuineness of the Offer. On 14 January 2011, the Officer telephoned the Employer and asked about the Offer. The Of- ficer also asked the Employer about its financial situation. The Employer said that its financial situation had recently improved, so it would be able to pay the Applicant’s salary. The Officer further asked the Employer about the Applicant’s ability to do the job, given his low level of English proficiency. The Employer said most of the parents and faculty at the school were Bangladeshi “or at least Asian” so the Applicant would be able to communicate with them. 10 The Officer made his final assessment of the Application on 14 Janu- ary 2011. He awarded 65 points, which is less than the minimum 67 points required for a permanent resident visa under the Federal Skilled Worker Program. The Officer sent a letter to the Applicant dated 14 Jan- uary 2011 informing him that his Application had been refused.

Decision under Review 11 The Decision consists of the Officer’s letter to the Applicant, dated 14 January 2011 and his CAIPS notes on the file. In the letter, the Officer informed the Applicant that he had been awarded 64 points. However, the points awarded in each category as shown in the Officer’s letter do not add up to 64 points. The Officer actually awarded the Applicant a total of 65 points as follows: Category Points Assessed Maximum Age 10 10 Education 22 25 Official language profi- 03 24 ciency Experience 21 21 Arranged employment 0 10 Adaptability 9 10 TOTAL 65 100 12 At issue in this case are the Officer’s awards of points in the educa- tion and arranged employment categories. 312 IMMIGRATION LAW REPORTER 3 Imm. L.R. (4th)

Education 13 The Officer found that the Applicant’s MBA in marketing from Green University in Bangladesh and a total of 16 years of full-time edu- cation qualified him for 22 points for education.

Arranged Employment 14 The Officer concluded that the Applicant would not be able to per- form the tasks required in the Offer. He said that the job description in- cluded proficiency in written and spoken English and required the Appli- cant to liaise with the Provincial Ministry of Education to ensure the school met the Ministry’s standards. The Officer noted that the Employer had assured him that the Applicant would be able to communicate with parents and faculty members, even though he lacked strong English skills. In spite of the Employer’s assurances, the Officer found that the Applicant’s lack of English meant that he did not meet the requirements of the Offer and would not be able to perform the tasks required. The Officer therefore awarded no points for arranged employment.

Relevant Legislation 15 The following provisions of the Act are applicable in this proceeding: 12. (2) A foreign national may be selected as a member of the eco- nomic class on the basis of their ability to become economically es- tablished in Canada. 12. (2) La s´election des etrangers´ de la cat´egorie « immigration economique´ » se fait en fonction de leur capacit´e a` r´eussir leur etab-´ lissement economique´ au Canada. 16 The following provisions of the Regulations are also applicable in this proceeding: 76. (1) For the purpose of determining whether a skilled worker, as a member of the federal skilled worker class, will be able to become economically established in Canada, they must be assessed on the basis of the following criteria: (a) the skilled worker must be awarded not less than the mini- mum number of required points referred to in subsection (2) on the basis of the following factors, namely, (i) education, in accordance with section 78, ... (v) arranged employment, in accordance with section 82, and Khan v. Canada (MCI) James Russell J. 313

(vi) adaptability, in accordance with section 83; and (b) the skilled worker must (i) have in the form of transferable and available funds, unencumbered by debts or other obligations, an amount equal to half the minimum necessary income applicable in respect of the group of persons consist- ing of the skilled worker and their family members, or (ii) be awarded the number of points referred to in subsec- tion 82(2) for arranged employment in Canada within the meaning of subsection 82(1). (3) Whether or not the skilled worker has been awarded the mini- mum number of required points referred to in subsection (2), an of- ficer may substitute for the criteria set out in paragraph (1)(a) their evaluation of the likelihood of the ability of the skilled worker to become economically established in Canada if the number of points awarded is not a sufficient indicator of whether the skilled worker may become economically established in Canada. ... 78. (2) A maximum of 25 points shall be awarded for a skilled worker’s education as follows: (e) 22 points for (i) a three-year post-secondary educational credential, other than a university educational credential, and a total of at least 15 years of completed fulltime or full- time equivalent studies, or (ii) two or more university educational credentials at the bachelor’s level and a total of at least 15 years of completed fulltime or full-time equivalent studies; and (f) 25 points for a university educational credential at the master’s or doctoral level and a total of at least 17 years of completed full-time or full-time equivalent studies. (3) For the purposes of subsection (2), points (a) shall not be awarded cumulatively on the basis of more than one single educational credential; and (b) shall be awarded (i) for the purposes of paragraphs (2)(a) to (d), subpara- graph (2)(e)(i) and paragraph (2)(f), on the basis of the single educational credential that results in the highest number of points, and 314 IMMIGRATION LAW REPORTER 3 Imm. L.R. (4th)

(ii) for the purposes of subparagraph (2)(e)(ii), on the ba- sis of the combined educational credentials referred to in that paragraph. ... 78 (4) For the purposes of subsection (2), if a skilled worker has an educational credential referred to in paragraph (2)(b), subparagraph (2)(c)(i) or (ii), (d)(i) or (ii) or (e)(i) or (ii) or paragraph (2)(f), but not the total number of years of full-time or fulltime equivalent studies required by that paragraph or subparagraph, the skilled worker shall be awarded the same number of points as the number of years of completed full-time or full-time equivalent studies set out in the para- graph or subparagraph. ... 82. (2) Ten points shall be awarded to a skilled worker for arranged employment in Canada in an occupation that is listed in Skill Type 0 Management Occupations or Skill Level A or B of the National Oc- cupational Classification matrix if they are able to perform and are likely to accept and carry out the employment [...] 76. (1) Les crit`eres ci-apr`es indiquent que le travailleur qualifi´e peut r´eussir son etablissement´ economique´ au Canada a` titre de membre de la cat´egorie des travailleurs qualifi´es (f´ed´eral): a) le travailleur qualifi´e accumule le nombre minimum de points vis´e au paragraphe (2), au titre des facteurs suivants: (i) les etudes,´ aux termes de l’article 78, ... (v) l’exercice d’un emploi r´eserv´e, aux termes de l’article 82, (vi) la capacit´e d’adaptation, aux termes de l’article 83; b) le travailleur qualifi´e: (i) soit dispose de fonds transf´erables — non grev´es de dettes ou d’autres obligations financi`eres — d’un montant l´egal a` la moiti´e du revenu vital minimum qui lui permettrait de subvenir a` ses propres besoins et a` ceux des membres de sa famille, (ii) soit s’est vu attribuer le nombre de points pr´evu au paragraphe 82(2) pour un emploi r´eserv´e au Canada au sens du paragraphe 82(1). (3) Si le nombre de points obtenu par un travailleur qualifi´e — que celui-ci obtienne ou non le nombre minimum de points vis´e au Khan v. Canada (MCI) James Russell J. 315

paragraphe (2) — n’est pas un indicateur suffisant de l’aptitude de ce travailleur qualifi´e a` r´eussir son etablissement´ economique´ au Can- ada, l’agent peut substituerson appr´eciation aux crit`eres pr´evus a` l’alin´ea (1)a). ... 78. (2) Un maximum de 25 points d’appr´eciation sont attribu´es pour les etudes´ du travailleur qualifi´e selon la grille suivante: e) 22 points, si, selon le cas: (i) il a obtenu un diplˆome postsecondaire — autre qu’un diplˆome universitaire — n´ecessitant trois ann´ees d’´etudes et a accumul´e un total de quinze ann´ees d’´etudes a` temps plein compl`etes ou l’´equivalent temps plein, (ii) il a obtenu au moins deux diplˆomes universitaires de premier cycle et a accumul´e un total d’au moins quinze ann´ees d’´etudes a` temps plein compl`etes ou l’´equivalent temps plein; f) 25 points, s’il a obtenu un diplˆome universitaire de deuxi`eme ou de troisi`eme cycle et a accumul´e un total d’au moins dix- sept ann´ees d’´etudes a` temps plein compl`etes ou l’´equivalent temps plein. (3) Pour l’application du paragraphe (2), les points sont accumul´es de la fa¸con suivante: a) ils ne peuvent etreˆ additionn´es les uns aux autres du fait que le travailleur qualifi´e poss`ede plus d’un diplˆome; b) ils sont attribu´es: (i) pour l’application des alin´eas (2)a) a` d), du sousalin´ea (2)e)(i) et de l’alin´ea (2)f), en fonction du diplˆome qui procure le plus de points selon la grille, (ii) pour l’application du sousalin´ea (2)e)(ii), en fonction de l’ensemble des diplˆomes vis´es a` ce sous-alin´ea. ... 78 (4) Pour l’application du paragraphe (2), si le travailleur qualifi´e est titulaire d’un diplˆome vis´e a` l’un des alin´eas (2)b), des sous- alin´eas (2)c)(i) et (ii), (2)d)(i) et (ii) et (2)e)(i) et (ii) ou a` l’alin´ea (2)f) mais n’a pas accumul´e le nombre d’ann´ees d’´etudes a` temps plein ou l’´equivalent temps plein pr´evu a` l’un de ces alin´eas ou sous- alin´eas, il obtient le nombre de points correspondant au nombre d’ann´ees d’´etudes a` temps plein compl`etes — ou leur equivalent´ temps plein — mentionn´e dans ces dispositions. 316 IMMIGRATION LAW REPORTER 3 Imm. L.R. (4th)

... 82. (2) Dix points sont attribu´es au travailleur qualifi´e pour un em- ploi r´eserv´e appartenant aux genre de comp´etence 0 Gestion ou niveaux de comp´etences A ou B de la matrice de la Classification nationale des professions, s’il est en mesure d’exercer les fonctions de l’emploi et s’il est vraisemblable qu’il acceptera de les exercer [...]

Issues 17 The Applicant formally raise the following issues: a. Whether the Officer erred in awarding no points for arranged employment; b. Whether the Officer erred in only awarding 22 points for educa- tion; and c. Whether the Officer erred when he did not consider a substitute evaluation under subsection 76(3) of the Regulations. 18 The Applicant also raises the following issue in his written argument: d. Whether the Officer breached the Applicant’s right to procedural fairness by denying him the opportunity to respond to concerns about his ability to perform the job requirements.

Standard of Review 19 The Supreme Court of Canada in New Brunswick (Board of Management) v. Dunsmuir, 2008 SCC 9 (S.C.C.), held that a standard of review analysis need not be conducted in every instance. Instead, where the standard of review applicable to a particular question before the court is well-settled by past jurisprudence, the reviewing court may adopt that standard of review. Only where this search proves fruitless must the re- viewing court undertake a consideration of the four factors comprising the standard of review analysis 20 In Kniazeva v. Canada (Minister of Citizenship & Immigration), 2006 FC 268 (F.C.), Justice held that the assessment of an application for permanent residence under the Federal Skilled Worker Class is an exercise of discretion that should be given a high degree of deference. Further, in Persaud v. Canada (Minister of Citizenship & Im- migration), 2009 FC 206 (F.C.), Justice John O’Keefe held that the ap- propriate standard of review for a determination under the Federal Skilled worker class is reasonableness. See alsoTong v. Canada (Minister of Citizenship & Immigration), 2007 FC 165 (F.C.). The stan- dard of review on the first two issues is reasonableness. Khan v. Canada (MCI) James Russell J. 317

21 When reviewing a decision on the standard of reasonableness, the analysis will be concerned with “the existence of justification, trans- parency and intelligibility within the decisionmaking process [and also with] whether the decision falls within a range of possible, acceptable outcomes which are defensible in respect of the facts and law.” See Dun- smuir, above, at paragraph 47, and Khosa v. Canada (Minister of Citizenship & Immigration), 2009 SCC 12 (S.C.C.) at paragraph 59. Put another way, the Court should intervene only if the Decision was unrea- sonable in the sense that it falls outside the “range of possible, acceptable outcomes which are defensible in respect of the facts and law.” 22 The Applicant challenges the Officer’s failure to consider a substitute evaluation under subsection 76(3) of the Regulations. In Fernandes v. Canada (Minister of Citizenship & Immigration), 2008 FC 243 (F.C.), Justice had this to say on the issue, at paragraph 8: What is being alleged here is the failure of the Visa Officer to con- sider the question of whether the discretion should be exercised, not that it was exercised wrongly. While a failure to exercise the discre- tion has often been treated as a breach of procedural fairness (see e.g. Nayyar, [[2007] F.C.J. No. 342], at para. 8) it appears to me to in- volve a question of law: namely has the Visa Officer taken every step that the law requires? In either case the standard of review is correct- ness and that is a standard I will apply to this issue. 23 Justice David Near, in Miranda v. Canada (Minister of Citizenship & Immigration), 2010 FC 424 (F.C.) relied on Fernandes and held at para- graph 9 that the standard of review with respect to an officer’s considera- tion of a request for a substituted evaluation under subsection 76(3) of the regulations is correctness. Where an applicant requests a substituted evaluation under subsection 76(3) of the Regulations, the officer process- ing the application must consider that request. I am satisfied that the standard of review on the third issue in this case is correctness. 24 The opportunity to respond is also an issue of procedural fairness. In C.U.P.E. v. Ontario (Minister of Labour), 2003 SCC 29 (S.C.C.), the Supreme Court of Canada held that the standard of review with respect to questions of procedural fairness is correctness. Further, the Federal Court of Appeal in Sketchley v. Canada (Attorney General), 2005 FCA 404 (F.C.A.) at paragraph 53 held that the “procedural fairness element is reviewed as a question of law. No deference is due. The decision-maker has either complied with the content of the duty of fairness appropriate for the particular circumstances, or has breached this duty.” The standard of review with respect to the fourth issue is correctness. 318 IMMIGRATION LAW REPORTER 3 Imm. L.R. (4th)

25 Also in Dunsmuir, above, the Supreme Court of Canada held at para- graph 50 that When applying the correctness standard, a reviewing court will not show deference to the decision maker’s reasoning process; it will rather undertake its own analysis of the question. The analysis will bring the court to decide whether it agrees with the determination of the decision maker; if not, the court will substitute its own view and provide the correct answer. From the outset, the court must ask whether the tribunal’s decision was correct.

Arguments 26 The Applicant and the Respondent agree that the Officer made an er- ror in adding the points he awarded and that the Officer awarded 65 points.

The Applicant The Officer’s Award of no Points for Arranged Employment was Unreasonable 27 The Officer’s award of no points for arranged employment was un- reasonable because it was made in ignorance of the evidence before him and based on a denial of procedural fairness.

The Officer Ignored Evidence 28 The Officer ignored the Employer’s confirmation over the telephone that he would be able to do the job even though his IELTS scores were low. The Applicant says that his case is analogous to Choi v. Canada (Minister of Citizenship & Immigration), 2008 FC 577 (F.C.) [Choi] where Justice Michael Kelen held that it was an error when the assessing officer failed to take into account a letter from the employer which said he had no concerns about the applicant’s ability to fulfil the requirements of the position offered to her. The Applicant says that his case is identical to Choi and so should be decided in the same way.

The Applicant was Denied Procedural Fairness 29 The Applicant also says that, following Choi, above, the Officer in this case denied the Applicant an opportunity to respond to his concerns when he did not send the Applicant a letter detailing his concerns about the Applicant’s language ability. The Applicant notes that the Citizenship and Immigration Canada’s (CIC) manual OP-6: Federal Skilled Workers Khan v. Canada (MCI) James Russell J. 319

at section 12.15 says that, “If [Officers] have any concerns about the ap- plicant’s ability or likelihood to accept and carry out the employment, they will communicate these to the applicant and provide the opportunity to respond.” Though the manual is not binding, it strongly suggests that the Officer should have written a letter and allowed the Applicant an op- portunity to address his concerns.

The Officer’s Award of 22 Points for Education was Unreasonable 30 The Applicant also argues that the Officer’s award of 22 points for education was unreasonable because it was not based on the evidence before him. The Applicant says that his form IMM0008 clearly discloses that he has a Master’s degree and at least 17 years of full-time studies. He says that his Primary and Secondary education were ten years, his Civil Engineering Diploma was three years, his BBA was three years and his MBA was one year for a total of 17 years. The Officer accepted that the Applicant had a Master’s degree, so he should have awarded the Ap- plicant 25 points under paragraph 78(2)(f) of the Regulations. With the additional three points above the Officer’s original award of 22 points for education, this would bring the Applicant’s total to 68 points, which is above the threshold for a granting a permanent resident visa.

The Officer Unreasonably Failed to Consider a Substituted Assessment Under Subsections 76(3) and (4) of the Regulations 31 Finally, the Applicant argues that, given the evidence that was before him, the Officer erred by not conducting a substitute assessment under subsections of the Regulations. The Applicant says that Choi (above) al- lows for settlement funds to be considered in a substitute assessment in addition to the criteria listed in paragraph 76(1)(a) of the Regulations. He says he had settlement funds of $200,000. The Applicant’s offer of em- ployment, settlement funds of $200,000, his brother in Canada, and his award of 68 points all indicate that his was the kind of case that war- ranted a substitute assessment. It was impossible to conclude other than that a person having all these characteristics would be able to success- fully establish himself economically in Canada. The Applicant says that his is the kind of case that falls within the Silva v. Canada (Minister of Citizenship & Immigration), 2007 FC 733 (F.C.) standard of “cases that present unusual facts or where the applicant has come close to obtaining [the required] units of assessment. 320 IMMIGRATION LAW REPORTER 3 Imm. L.R. (4th)

32 The Applicant also notes that in Lackhee v. Canada (Minister of Citizenship & Immigration), 2008 FC 1270 (F.C.), at paragraph 20, Jus- tice held that The officer’s failure to make any reference to the considerable assets available to the applicant in either her decision or her notes consti- tutes a reviewable error warranting this Court’s intervention The Applicant says he submitted evidence showing that he had CDN $200,000 in settlement funds. The Officer’s failure to consider this was an error.

The Respondent The Award of 22 Points for Education was Reasonable 33 The Respondent argues that the Officer was reasonable in awarding 22 points under the education category and that this conclusion was based on evidence before him. The Applicant clearly indicates in IMM0008 Schedule 1 — Background/Declaration that he has only 16 years of full-time instruction. This does not meet the threshold under par- agraph 78(2)(f) for an award of 25 points.

The Award of no Points for Arranged Employment was Reasonable 34 The Respondent argues that the Officer’s award of no points for ar- ranged employment was reasonable because he was required to investi- gate whether the Applicant could perform the tasks required of him in the Offer and reasonably concluded he could not. Relying on Bellido v. Canada (Minister of Citizenship & Immigration), 2005 FC 452 (F.C.) at paragraph 21, the Respondent says that an AEO does not remove the obligation for an officer to assess whether an applicant is able to perform the job described in the AEO. It is clear from the CAIPS notes that the Officer considered the information provided by the Employer, though he ultimately decided that it was not sufficient to overcome his concerns. The Officer was entitled to put little weight on the evidence of the Em- ployer. It is not for the Court to question the weight put on each piece of evidence, so long as the Officer’s findings were rationally based on the material before him. 35 The Respondent also says that the Officer found that the job descrip- tion included liaising with the Province to ensure that standards were be- ing met by the school. The Applicant would not be able to perform this aspect of the job because he lacked proficiency in English as shown by Khan v. Canada (MCI) James Russell J. 321

his low IELTS scores. This was a reasonable conclusion based on the facts before the Officer.

The Officer’s Decision Not to Conduct a Substitute Assessment was Reasonable 36 Finally, the Respondent argues that the Officer’s decision not to con- duct a substitute assessment under subsections 76(3) and (4) was reason- able because the Officer was under no duty to do so. The Applicant did not specifically request a substitute assessment so the Officer had no ob- ligation to make one. The Respondent relies on Miranda, above, and Eslamieh v. Canada (Minister of Citizenship & Immigration), 2008 FC 722 (F.C.). The Respondent distinguishes Choi (above), saying that in Choi the applicant requested that the officer engage in a substitute as- sessment where here the Applicant did not make such a request. 37 The Respondent also says that Xu v. Canada (Minister of Citizenship & Immigration), 2010 FC 418 (F.C.) and Tokuda v. Canada (Minister of Citizenship & Immigration), 2011 FC 483 (F.C.), hold that a substituted assessment under subsection 76(3) may only be used to consider the fac- tors in paragraph 76(1)(a) of the Regulation. These cases specifically ex- clude from the 76(3) substituted assessment the amount of settlement funds held by an applicant. It was not open to the Officer to consider the Applicant’s $200,000 in settlement funds in a substituted evaluation.

Analysis Did the Officer Err in Awarding 22 Points Instead of 25 Points Under the Education Category? 38 In my view, the first issue regarding the appropriate points awarded under the education category is determinative in this case. The Officer found that the Applicant had a Master’s degree, but only 16 years of full- time education which only entitled the Applicant to 22 points. 39 The Applicant submits that he completed 17 years of full-time educa- tion in addition to a Master’s degree, thus entitling him to the full 25 points. 40 What is important for this case is the Applicant’s Schedule 1 (Back- ground/declaration) form where the Applicant sets out his education. This can be found at page 188 of the Certified Tribunal Record. 41 Unfortunately, this page creates some confusion. The Applicant claims that he completed 10 years of elementary/primary school, three 322 IMMIGRATION LAW REPORTER 3 Imm. L.R. (4th)

years of secondary/high school and five years of university/college. If correct, this would equal 18 years of full-time education. 42 The first problem is that it is not possible to tell from the Decision (including the CAIPS notes) how the Officer calculated the number of full-time years of education completed by the Applicant. This was con- ceded by Respondent’s counsel at the judicial review. She led the Court through her own calculation, but she arrived at a lesser number than the 16 years used by the Officer. Applicant’s counsel also demonstrated how she had calculated the number of years of fulltime education and her fig- ure was 17. 43 But the important point is that no one knows how the Officer calcu- lated the number of years of full-time education (conceded by the Re- spondent) so it really is not possible to understand this aspect of the De- cision or to determine whether it is reasonable. Hence, there is a procedural fairness issue. In this kind of situation the Officer does not need to provide elaborate reasons, but it is still necessary to know the basis of the calculation. When Respondent’s counsel cannot explain the basis of the Officer’s calculation I think it would be unfair to expect the Applicant to be able to do so. In other words, without some explanation for the calculation, the reasons are totally inadequate on this issue. See Jogiat v. Canada (Minister of Citizenship & Immigration), 2009 FC 815 (F.C.) at paragraphs 36 to 44, Healey v. Canada (Minister of Citizenship & Immigration), 2009 FC 355 (F.C.), at paragraphs 58-60 and Abdullah v. Canada (Minister of Citizenship & Immigration), 2006 FC 1185 (F.C.). 44 The concern over this point is compounded by the fact that the Of- ficer told the Applicant he had awarded a total of 64 points. In fact, when his points for each category are added up they come to 65 points. This could be a simple error in addition, but it suggests to the Court that the Officer has been a little slap-dash and this matter needs to be returned for reconsideration. 45 The Applicant has raised a number of further issues, but unless the parties and the Court can understand how the Officer went about calcu- lating the points award, I see little point in reviewing issues. 46 For reasons set out above, I conclude that this matter must be returned for reconsideration. 47 Counsel agree there is no question for certification and the Court concurs. Khan v. Canada (MCI) James Russell J. 323

Judgment THIS COURT’S JUDGMENT is that 1. The application is allowed. The Decision is quashed and the mat- ter is returned for reconsideration. 2. There is no question for certification. Application granted. 324 IMMIGRATION LAW REPORTER 3 Imm. L.R. (4th)

[Indexed as: Victoria v. Canada (Minister of Public Safety & Emergency Preparedness)] Amparo Torres Victoria, Applicant and The Minister of Public Safety and Emergency Preparedness, Respondent Federal Court Docket: IMM-1862-11 2011 FC 1392 Yves de Montigny J. Heard: September 20, 2011 Judgment: November 30, 2011 Immigration and citizenship –––– Constitutional issues — Charter of Rights and Freedoms — Visitors and immigrants — Exclusion and removal –––– Applicant fled Colombia following persecution due to her political activities — Applicant was recognized as Convention refugee in Mexico and then arrived in Canada as permanent resident — Minister learned of applicant’s involvement with guerrilla force in Colombia and alleged that applicant was inadmissible due to membership in terrorist organization — Minister relied on confidential evi- dence, disclosure of which would be injurious to national security — Applicant asked immigration board to make immediate determination as to whether her rights under s. 7 of Canadian Charter of Rights and Freedoms were engaged in proceedings — In declining request, board member explained that she would re- serve on Charter issues until hearing all evidence and submissions on admissi- bility issues — Applicant brought application for judicial review — Application dismissed — Mootness was not valid basis upon which to decline to rule on ap- plication — Latest decision of board addressed applicant’s claim that her admis- sibility hearing engaged her s. 7 rights, but not at fundamental level at which it was argued: that entire proceeding was in violation of Charter — As to merits of application, there was no legal justification for court to intervene at this stage of proceedings before board — Board’s decision was interlocutory and was not re- viewable on judicial review — It would be contrary to rationale underlying judi- cial review for court to pronounce on issued before administrative decision- maker had opportunity to consider it — There were no special circumstances warranting immediate judicial review of this interlocutory decision — There was no prejudice to applicant that was not capable of being remedied at some later juncture — Courts should refrain from deciding constitutional issues when it is not strictly required in order to determine case — Board was justified in reserving decision. Victoria v. Canada 325

Administrative law –––– Prerequisites to judicial review — Miscellane- ous –––– Applicant fled Colombia following persecution due to her political ac- tivities — Applicant was recognized as Convention refugee in Mexico and then arrived in Canada as permanent resident — Minister learned of applicant’s in- volvement with guerrilla force in Colombia and alleged that applicant was inad- missible due to membership in terrorist organization — Minister relied on confi- dential evidence, disclosure of which would be injurious to national security — Applicant asked immigration board to make immediate determination as to whether her rights under s. 7 of Canadian Charter of Rights and Freedoms were engaged in proceedings — In declining request, board member explained that she would reserve on Charter issues until hearing all evidence and submissions on admissibility issues — Applicant brought application for judicial review — Application dismissed — Mootness was not valid basis upon which to decline to rule on application — Latest decision of board addressed applicant’s claim that her admissibility hearing engaged her s. 7 rights, but not at fundamental level at which it was argued: that entire proceeding was in violation of Charter — As to merits of application, there was no legal justification for court to intervene at this stage of proceedings before board — Board’s decision was interlocutory and was not reviewable on judicial review — It would be contrary to rationale underlying judicial review for court to pronounce on issued before administra- tive decision-maker had opportunity to consider it — There were no special cir- cumstances warranting immediate judicial review of this interlocutory deci- sion — There was no prejudice to applicant that was not capable of being remedied at some later juncture — Courts should refrain from deciding constitu- tional issues when it is not strictly required in order to determine case — Board was justified in reserving decision. Cases considered by Yves de Montigny J.: Almrei, Re (2008), 76 Imm. L.R. (3d) 126, 180 C.R.R. 338, 2008 FC 1216, 2008 CarswellNat 4406, [2009] 3 F.C.R. 497, [2008] F.C.J. No. 1488 (F.C.) — considered Almrei, Re (2009), 2009 CF 1263, 2009 CarswellNat 4286, 2009 FC 1263, [2011] 1 F.C.R. 163, 355 F.T.R. 222 (Eng.), 2009 CarswellNat 5657, 86 Imm. L.R. (3d) 212, [2009] F.C.J. No. 1579, [2009] A.C.F. No. 1579 (F.C.) — referred to Borowski v. Canada (Attorney General) (1989), [1989] 3 W.W.R. 97, [1989] 1 S.C.R. 342, 57 D.L.R. (4th) 231, 92 N.R. 110, 75 Sask. R. 82, 47 C.C.C. (3d) 1, 33 C.P.C. (2d) 105, 38 C.R.R. 232, 1989 CarswellSask 241, 1989 CarswellSask 465, [1989] S.C.J. No. 14 (S.C.C.) — considered C.B. Powell Ltd. c. Canada (Agence des services frontaliers) (2010), 2010 Car- swellNat 1197, 2010 CAF 61, [2011] 2 F.C.R. 332, (sub nom. Powell (C.B.) Ltd. v. Canada Border Services Agency (President)) 400 N.R. 367, 2010 FCA 61, 2010 CarswellNat 391, [2010] F.C.J. No. 274 (F.C.A.) — referred to 326 IMMIGRATION LAW REPORTER 3 Imm. L.R. (4th)

Charkaoui, Re (2007), 54 Admin. L.R. (4th) 1, (sub nom. Charkaoui v. Canada (Minister of Citizenship & Immigration)) 152 C.R.R. (2d) 17, 2007 SCC 9, 44 C.R. (6th) 1, 59 Imm. L.R. (3d) 1, 2007 CarswellNat 325, 2007 Car- swellNat 326, 358 N.R. 1, 276 D.L.R. (4th) 594, (sub nom. Charkaoui v. Canada) [2007] 1 S.C.R. 350, [2007] S.C.J. No. 9 (S.C.C.) — followed Charkaoui, Re (2008), 2008 CarswellNat 1898, 2008 CarswellNat 1899, 2008 SCC 38, (sub nom. Charkaoui v. Canada (Minister of Citizenship & Immi- gration)) 175 C.R.R. (2d) 120, 294 D.L.R. (4th) 478, 58 C.R. (6th) 45, 376 N.R. 154, (sub nom. Charkaoui v. Canada (Minister of Citizenship and Im- migration)) [2008] 2 S.C.R. 326, 70 Imm. L.R. (3d) 1, [2008] S.C.J. No. 39 (S.C.C.) — followed Cuddy Chicks Ltd. v. Ontario (Labour Relations Board) (1991), 91 C.L.L.C. 14,024, 3 O.R. (3d) 128 (note), 50 Admin. L.R. 44, 122 N.R. 361, 81 D.L.R. (4th) 121, [1991] O.L.R.B. Rep. 790, 47 O.A.C. 271, 4 C.R.R. (2d) 1, [1991] 2 S.C.R. 5, 1991 CarswellOnt 976, 1991 CarswellOnt 3004, [1991] S.C.J. No. 42, EYB 1991-67701 (S.C.C.) — referred to Danson v. Ontario (Attorney General) (1990), 43 C.P.C. (2d) 165, 73 D.L.R. (4th) 686, [1990] 2 S.C.R. 1086, 50 C.R.R. 59, 41 O.A.C. 250, 112 N.R. 362, 1990 CarswellOnt 366, 74 O.R. (2d) 763 (note), 1990 CarswellOnt 1004, [1990] S.C.J. No. 92, EYB 1990-67568 (S.C.C.) — referred to Douglas/Kwantlen Faculty Assn. v. Douglas College (1990), (sub nom. Douglas College v. Douglas/Kwantlen Faculty Assn.) 52 B.C.L.R. (2d) 68, (sub nom. Douglas College v. Douglas/Kwantlen Faculty Assn.) 77 D.L.R. (4th) 94, (sub nom. Douglas College v. Douglas/Kwantlen Faculty Assn.) 13 C.H.R.R. D/403, (sub nom. Douglas College v. Douglas/Kwantlen Faculty Assn.) 50 Admin. L.R. 69, (sub nom. Douglas College v. Douglas/Kwantlen Faculty Assn.) 2 C.R.R. (2d) 157, (sub nom. Douglas College v. Douglas/Kwantlen Faculty Assn.) 91 C.L.L.C. 17,002, (sub nom. Douglas College v. Douglas/Kwantlen Faculty Assn.) [1990] 3 S.C.R. 570, (sub nom. Douglas College v. Douglas/Kwantlen Faculty Assn.) [1991] 1 W.W.R. 643, (sub nom. Douglas College v. Douglas/Kwantlen Faculty Assn.) 118 N.R. 340, 1990 CarswellBC 278, 1990 CarswellBC 766, [1990] S.C.J. No. 124, EYB 1990-67026 (S.C.C.) — referred to Harkat, Re (2010), 380 F.T.R. 61 (Eng.), 2010 CF 1241, 2010 FC 1241, 2010 CarswellNat 4699, 2010 CarswellNat 4700 (F.C.) — referred to Harkat, Re (2010), 94 Imm. L.R. (3d) 179, 380 F.T.R. 163 (Eng.), 2010 CF 1242, 2010 FC 1242, 2010 CarswellNat 4714, 2010 CarswellNat 4715 (F.C.) — referred to Jaballah, Re (2010), 204 C.R.R. (2d) 262, [2011] 2 F.C.R. 145, 2010 Car- swellNat 130, 2010 FC 79, 364 F.T.R. 72 (Eng.), 2010 CarswellNat 1657, 2010 CF 79 (F.C.) — referred to Moysa v. Alberta (Labour Relations Board) (1989), 34 C.P.C. (2d) 97, [1989] 1 S.C.R. 1572, [1989] 4 W.W.R. 596, 60 D.L.R. (4th) 1, 96 N.R. 70, 67 Alta. Victoria v. Canada 327

L.R. (2d) 193, 97 A.R. 368, 89 C.L.L.C. 14,028, 40 C.R.R. 197, 1989 CarswellAlta 86, 1989 CarswellAlta 616, EYB 1989-66920 (S.C.C.) — re- ferred to Stables v. Canada (Minister of Citizenship & Immigration) (2011), 2011 FC 1319, 2011 CarswellNat 4770 (F.C.) — followed T´etreault-Gadoury v. Canada (Employment & Immigration Commission) (1991), 4 C.R.R. (2d) 12, 36 C.C.E.L. 117, 50 Admin. L.R. 1, 126 N.R. 1, 81 D.L.R. (4th) 358, (sub nom. Canada (Employment & Immigration Commission) v. T´etreault-Gadoury) [1991] 2 S.C.R. 22, 91 C.L.L.C. 14,023, 1991 CarswellNat 829, 1991 CarswellNat 346, [1991] S.C.J. No. 41, EYB 1991-67264 (S.C.C.) — referred to Statutes considered: Canadian Charter of Rights and Freedoms, Part I of the Constitution Act, 1982, being Schedule B to the Canada Act 1982 (U.K.), 1982, c. 11 Generally — referred to s. 1 — referred to s. 7 — referred to s. 24(1) — referred to Immigration and Refugee Protection Act, S.C. 2001, c. 27 Generally — referred to s. 33 — referred to s. 34 — referred to s. 34(1)(f) — considered s. 44(2) — referred to ss. 77-85 — referred to s. 83 — referred to s. 83(1)(b) — referred to s. 85.1(1) [en. 2008, c. 3, s. 4] — referred to s. 85.4 [en. 2008, c. 3, s. 4] — referred to s. 85.4(1) [en. 2008, c. 3, s. 4] — referred to s. 86 — referred to s. 162(1) — referred to Immigration and Refugee Protection Act (certificate and special advocate) and to make a consequential amendment to another Act, Act to amend the, S.C. 2008, c. 3 Generally — referred to Rules considered: Immigration Division Rules, SOR/2002-229 Generally — referred to R. 3 — considered 328 IMMIGRATION LAW REPORTER 3 Imm. L.R. (4th)

R. 47 — considered

APPLICATION for judicial review of decision declining request for immediate ruling on applicability of Canadian Charter of Rights and Freedoms to immi- gration admissibility proceedings.

Raoul Boulakia, for Applicant Martin Anderson, for Respondent

Yves de Montigny J.:

1 This is an application for judicial review of a decision of the Immi- gration Division of the Immigration and Refugee Board (the “Board” or the “Tribunal”) dated March 4, 2011, whereby the Board declined the Applicant’s request for an immediate ruling on the applicability of the Canadian Charter of Rights and Freedoms, Part I of the Constitution Act, 1982, being Schedule B to the Canada Act 1982 (UK), 1982, c 11 [Charter] in the proceeding. The Board Member explained that she would reserve on the Charter issues until she had heard all of the evi- dence and submissions pertaining to the Applicant’s admissibility inquiry pursuant to subsection 34(1)(f) of the Immigration and Refugee Protec- tion Act, SC 2001, c 27 [IRPA]. 2 For the reasons that follow, I am of the view that this application ought to be dismissed, essentially because it is for the Immigration Divi- sion to decide the Charter issues advanced by the Applicant before this Court can be called upon to review such a decision.

1. Facts 3 The Applicant, Amparo Torres Victoria, is a citizen of Colombia. She was born on June 21, 1955, in the city of Cali. She claims to be a trade union and human rights activist, as well as a founding member of the Union Patriotica, a political movement which was apparently an um- brella organization for leftist political parties in Colombia. The main guerrilla force in Colombia, the Revolutionary Armed Forces of Colom- bia (“FARC”), was involved in peace negotiations with the government of Colombia in 1985. As part of the peace talks, the FARC agreed to become a legalized political movement and joined the Union Patriotica until they decided to abandon the peace process, in 1987. 4 As a result of her activities, the Applicant was the subject of numer- ous threats to her life. She was also kidnapped, beaten, abused and de- Victoria v. Canada Yves de Montigny J. 329

tained for several months, allegedly by the paramilitaries, for her activi- ties as a member of the Union Patriotica and for being associated with the FARC. She fled Colombia upon her release in February 1993, along with her common-law spouse and their three children. She was recog- nized as a refugee by the United Nations High Commission for Refugees in Mexico, and she chose to immigrate to Canada with two of her sons. Her husband decided to remain in Mexico, to join the FARC, and to be- come the international media spokesman for the FARC. 5 The Applicant’s older brother is a member of the FARC, and one of her sisters was first kidnapped and then killed by the paramilitaries be- cause of his involvement in the organization. Her two other sisters and her mother fled to Mexico and claimed refugee status when her other sister was first abducted. 6 Having been recognized as a Convention refugee, the Applicant ar- rived in Canada on December 10, 1996 as a permanent resident. She then applied for citizenship on June 13, 2000. 7 The Respondent later learned of her involvement in the FARC, and referred a report to the Immigration Division pursuant to ss. 44(2) of IRPA alleging that she was inadmissible under ss. 34(1)(f) due to her membership in a terrorist organization. In advancing that allegation, the Minister has relied on confidential evidence, the disclosure of which would be injurious to Canada’s national security. 8 This is the second proceeding involving the Applicant before the Im- migration Division. At the first proceeding, the member decided to deter- mine the subsection 34(1)(f) allegation on its merits, prior to assessing the Applicant’s Charter challenge to IRPA’s secret evidence provisions in the context of an admissibility hearing. The member heard all the evi- dence and adjourned the hearing in order to prepare his decision. Unfor- tunately, he later advised that he would be unable to render his decision before his authority under IRPA had lapsed. 9 In February 2007, the Supreme Court of Canada released its decision in Charkaoui, Re, 2007 SCC 9, [2007] 1 S.C.R. 350 (S.C.C.) [Charkaoui #1]. In that decision, the Court found, inter alia, that IRPA did not ade- quately protect the rights of the named person to a fair hearing, and therefore struck down s. 33 and 77-85 of IRPA as infringing s. 7 of the Charter. It is on the basis of that decision that counsel for the Applicant made a motion before the first member of the Immigration Division deal- ing with her case, arguing that the same reasoning applies to the non- 330 IMMIGRATION LAW REPORTER 3 Imm. L.R. (4th)

disclosure of information in the context of an admissibility hearing pur- suant to s. 86 of IRPA. 10 When the second proceeding before the Immigration Division started, Parliament had adopted Bill C-3. This Bill, which came into force on February 22, 2008, was in response to the declaration of invalidity pro- nounced in Charkaoui #1. In a nutshell, these amendments to IRPA intro- duced the special advocate regime; pursuant to paragraph 83(1)(b), the designated judge shall appoint a special advocate whose name must be on a list established by the Minister of Justice. The role of the special advocate is “to protect the interests” of the named person in closed hear- ings (subsection 85.1(1) of IRPA). Of relevance for the case at bar, these amendments to IRPA have extended the role of special advocates to all the proceedings before the Immigration Division involving confidential evidence, including an admissibility hearing (see s. 86 of IRPA). 11 In June 2008, upon the Applicant’s request, the Immigration Division appointed Mr. Waldman as the Applicant’s special advocate. In July 2008, the Minister provided the Applicant with the open source evidence that it intended to rely on to establish the ss. 34(1)(f) allegation. In Sep- tember 2008, Member Funston was assigned to determine the ss. 34(1)(f) allegation on its merits. In October 2008, the Applicant advanced several motions, including a request for a ruling that s. 7 of the Charter be en- gaged in the proceeding. In October 2008, Member Funston declined that request. 12 Further open source information was adduced in August 2009. In No- vember 2009, following the Applicant’s and Mr. Waldman’s request, Mr. Dadour was appointed as a second special advocate. In December 2009, Mr. Dadour was provided with copies of prior correspondence, decisions and a transcript of a pre-hearing conference. Mr. Dadour was given ac- cess to the closed material in February 2010. 13 The special advocates filed two motions in late March 2010. Closed hearings were held in June and July 2010 on the first motion concerning the order of proceedings. In September 2010, the Immigration Division held that counsel for the Applicant should participate in the special advo- cates’ first motion and make submissions thereon on the Applicant’s be- half. Counsel was provided with a copy of the special advocates’ and the Minister’s submissions on the order of proceedings issue. 14 On March 4, 2011, the Board Member ruled that the closed proceed- ing should proceed before the public proceeding, in order for the Appli- cant to be as informed as possible regarding the issues and evidence that Victoria v. Canada Yves de Montigny J. 331

confronted her. With the Minister presenting his secret evidence first, and the special advocates being given the opportunity to challenge that evidence and cross-examine any witnesses, this would result in as much evidence as possible being potentially disclosed to the Applicant at the proceeding, as well as any further summaries of the evidence. That being said, the Board Member did not preclude the possibility to return to the closed proceeding, after the public proceeding, in order to enquire into the Minister’s secret evidence should any new evidence arise in the pub- lic proceeding that could have led the special advocates to challenge the relevancy, reliability and sufficiency of any aspect of the Minister’s se- cret evidence. This aspect of the Board’s decision is not challenged in this application for judicial review. 15 In early 2011, the Applicant asked the Immigration Division to make an immediate determination as to whether her s. 7 Charter rights were engaged in the Immigration Division proceedings concerning her. In her March 4, 2011 decision, the Board Member recognized that the issues of fairness, the principles of fundamental justice and one’s rights guaran- teed under the Charter have arisen throughout these proceedings. She also acknowledged that both public counsel and the special advocates have argued that the Applicant’s s. 7 Charter rights are engaged, as the Applicant is subject to a proceeding that could ultimately lead to her re- moval from Canada and subsequent persecution. She nevertheless de- clined to rule immediately on this issue, explaining that she would re- serve on the Charter issues until she had heard all the evidence and submissions on the admissibility issues: In my view, it is premature to make findings with respect to an indi- vidual’s Charter rights with respect to the potential consequences of an admissibility proceeding while the issue turns upon something that, in fact, may never happen: i.e., the issuance of a removal order. 16 This is the decision that is being challenged in the present application for judicial review, which was filed on March 22, 2011. 17 Subsequent to that decision, the Immigration Division released an- other decision that is material to the case at bar. The special advocates have put forward a number of preliminary motions, starting on Septem- ber 11, 2009, for full disclosure of the material relating to the Applicant, including the entire Canadian Security Intelligence Service (“CSIS”) file. These motions have led to additional disclosure by the Minister, who agreed voluntarily to provide further material to the special advocates, first on December 11, 2009 and then again, on December 23, 2010. Upon 332 IMMIGRATION LAW REPORTER 3 Imm. L.R. (4th)

review of the new material, however, the special advocates renewed their application for full disclosure on February 21, 2011. 18 The motion of the special advocates is predicated on the applicability of the decision of the Supreme Court of Canada in Charkaoui, Re, 2008 SCC 38, [2008] 2 S.C.R. 326 (S.C.C.) (Charkaoui #2) to the circum- stances of the present case. In that decision, it will be remembered, it was held that to uphold the right to procedural fairness of people subject to a security certificate, CSIS is required to retain all its operational notes and to disclose them to the ministers for the issuance of a security certificate. Subsequently they would be required to disclose them to the designated judge for the review of the reasonableness of the certificate and of the need to detain the named person. Pursuant to ss. 85.4(1) of IRPA, added by Bill C-3, the special advocates shall be provided with a copy of all information and evidence that is provided to the judge by the Minister. 19 The special advocates asserted that their request for full disclosure was in keeping with disclosure requirements set out in Charkaoui #2; basic administrative law principles of procedural fairness and natural jus- tice, and the rights enjoyed by the Applicant under section 7 of the Char- ter. They were of the view that an admissibility hearing before the Immi- gration Division involving an application for non-disclosure is akin to a security certificate proceeding. They both involve the non-disclosure of evidence to the subject of the proceedings, they both require that there be a substantial substitute in place of the person concerned, and the conse- quences to the named person in security certificate cases is the same as those encountered by the person concerned in an admissibility hearing, namely, as in this case, possible removal to Colombia. 20 The Immigration Division granted the motion for disclosure. In doing so, Member Funston highlighted her understanding of Charkaoui #2, to the effect that for a security certificate proceeding to comply with funda- mental justice, there must be disclosure of the materials in CSIS’ posses- sion that relates to the named person. She determined that the procedural protections mandated by Charkaoui #2 would also apply to the Immigra- tion Division concerning the Applicant. Similar to a security certificate proceeding, the hearing on the ss. 34(1)(f) allegation could result in a deportation order. The Immigration Division proceeding was subject to the same protection of information scheme (under IRPA) that applies to security certificates. Therefore, she agreed with the special advocates that there was no real difference between the consequences of a security Victoria v. Canada Yves de Montigny J. 333

certificate and those of the ss. 34(1)(f) proceeding before the Immigra- tion Division. 21 For good measure, the Immigration Division did point out one real difference between security certificate cases and section 86 proceedings in the context of an admissibility hearing. As the Board Member noted, in the case of a security certificate, the deportation order is issued first by the Minister. The relevance, reliability and sufficiency of the Minister’s information is then challenged by the special advocates in the context of the closed proceedings. The order is reversed in the case of an admissi- bility hearing, where the relevance, reliability and sufficiency of the Min- ister’s information is challenged by special advocates during the closed proceedings involving the section 86 portion of the admissibility hearing. This would occur prior to any final determination as to whether or not the person concerned is inadmissible and, therefore, should be issued a deportation order. That being said, the Immigration Division found that distinction of no significance, as the ultimate determination made by the Federal Court in a security certificate proceeding and by the Immigration Division in the context of an admissibility hearing, can produce the same result (i.e. the person is or is not inadmissible pursuant to s. 34 of IRPA). Moreover, what happens at the closed and open proceedings before the Federal Court and the Immigration Division is virtually the same. 22 Despite these similarities, the Minister had argued that as this is not a security certificate case, neither the Charter nor the disclosure require- ments set out in Charkaoui #2 apply in an admissibility hearing before the Immigration Division. Relying on Rule 3 of the Immigration Division Rules, SOR/2002-229 [Rules], which provides that “the Minister must provide ...any relevant information or document that the Minister may have...”, the Minister argued that there is no duty to disclose irrelevant material, nor to disclose more evidence or information to the special ad- vocates than what would be disclosed to the person concerned. 23 The Immigration Division rejected the Minister’s arguments, and found that the circumstances of the Applicant are not those of a typical admissibility hearing for two reasons. First, she is a declared Convention refugee who was found to have a well-founded fear of persecution in Colombia. Second, she is the subject of an admissibility hearing where the Minister has applied for non-disclosure of information pursuant to section 86 of IRPA. Accordingly, the Board Member determined that those characteristics made the Applicant’s case comparable to that of a security certificate, and that the same procedural protections that funda- 334 IMMIGRATION LAW REPORTER 3 Imm. L.R. (4th)

mental justice would require there, would also apply to the ss. 34(1)(f) proceeding: [33] Both procedures involve inadmissibility on security grounds, both procedures involve protected information that is not disclosed to the subject of the proceedings, both procedures are governed by the same statutory provisions regarding the protection of information, both procedures involve Special Advocates whose role and responsi- bilities are identical in both proceedings. There are, in my view, many more similarities between the two proceedings than there are differences. [34] In my view, the aforementioned distinguishing characteristics lead to a more apt comparison with the Security Certificate cases. Through the section 86 proceedings, Ms. Torres, potentially, is being denied the right to know the entire case to meet. I am more persuaded by the arguments of the Special Advocates in their submissions that Ms. Torres’ case is more akin to the Security Certificate cases in that there is essentially no difference between the two proceedings. As such, since the procedural protections of section 7 of the Charter ap- ply in the Security Certificate cases, so too should those same protec- tions apply in this particular case.

2. Issues 24 This application for judicial review raises two issues. The first one is whether this application is moot, in light of the decision made by the Immigration Division on May 12, 2011. The second is whether the Court should decline to rule on this application because it would be premature to do so.

3. Analysis a) Mootness 25 Counsel for the Minister argued that the application for judicial re- view is now moot as a result of the decision reached by the Immigration Division on May 12, 2011. It is suggested that the Applicant’s original complaint was with respect to the delay in having the Immigration Divi- sion determine whether her s. 7 Charter rights were engaged in the pro- ceedings. Member Funston having since found that the Applicant’s s. 7 Charter rights are engaged in the ss. 34(1)(f) proceeding and that the Charkaoui #2 decision should apply in the circumstances of this case, it is argued that the Immigration Division has made a decision on the Char- Victoria v. Canada Yves de Montigny J. 335

ter engagement issue, and that it is therefore moot for all intents and purposes. 26 Of course, a Court is always left with the discretion to hear a case even if the required tangible and concrete dispute has disappeared and the issues have become academic (see Borowski v. Canada (Attorney General), [1989] 1 S.C.R. 342 (S.C.C.) at pp 358-ff [Borowski]). Ac- cording to the Respondent, however, the Court should not exercise its discretion to hear this moot judicial review, because the question in issue is specific to the Applicant in this context and is not one of public impor- tance. The Respondent argues that the issue at bar arises sparingly, and is not one that is of a short duration and escapes review. 27 I cannot agree with the Respondent. It is true that, at some level, the latest decision of the Immigration Division does answer the Applicant’s claim that her admissibility hearing engages her section 7 rights. How- ever, the argument she is making is at a more fundamental level. Her position is not only that the Charter is applicable to her case and that she is entitled to some procedural guarantees, which is the position that the special advocates seem to have taken in requesting full disclosure in ac- cordance with Charkaoui #2; what she claims, in essence, is that the en- tire proceeding is in violation of the Charter because there is no valid basis for relying on secret evidence in her admissibility hearing. 28 The main thesis of the Applicant is that reliance on secret evidence in the context of an admissibility hearing before the Immigration Division breaches her right to full answer and defence and infringes her s. 7 Char- ter rights. She further contends that the introduction of a special advocate regime was suggested as a possible cure to this Charter breach in Charkaoui #1, because the Supreme Court accepted that the protection of Canada’s national security and related intelligence sources undoubtedly constitutes a pressing and substantial objective. It is in that context that the Supreme Court was prepared to accept that special advocates ap- pointed to represent the interests of a named person would strike a better balance between the protection of sensitive information and the procedu- ral rights of an individual; in other words, a revamped security certificate regime with the introduction of special advocates could be found to mini- mally impair a named person’s right. In the absence of a security threat, argues the Applicant, a breach of her right to full answer and defence cannot be saved under section 1, even if she is represented in the closed proceedings by special advocates. 336 IMMIGRATION LAW REPORTER 3 Imm. L.R. (4th)

29 One need not assess the strength of this argument, let alone rule on it, to determine whether the May 12, 2011 decision of the Immigration Board completely settles the argument put forward by the Applicant. It clearly does not. The Board Member accepted that the Applicant’s s. 7 rights were engaged by virtue of the fact that she could be removed to a country where it has been established, she has a well-founded fear of persecution. The Board Member was also prepared to accept that the de- nial of her right to know the entire case to meet infringes the principles of fundamental justice, just as in the context of a security certificate. She clearly did not go as far as saying that the use of secret evidence in ad- missibility hearings irremediably vitiates her Charter rights, in a manner that cannot be justified under s. 1, irrespective of the procedural safe- guards that are found in IRPA and the Rules and that can be ordered by the Immigration Division. 30 It cannot be said that the issue raised initially by the Applicant has become academic, or that the concrete and tangible dispute between the parties has disappeared. There is still an existing controversy between the Applicant and the Respondent with respect to the fundamental question that lies at the core of the Applicant’s thesis. The latest decision of the Immigration Division has partially addressed the Applicant’s argument, but it has not drawn the full consequences from the application of s. 7 that the Applicant would like it to draw — i.e., that the whole inadmissi- bility proceeding violates her constitutional rights given her particular circumstances, irrespective of any procedural safeguards she may benefit from. 31 Before bringing this discussion to a close, it is worth quoting the fol- lowing excerpt of Borowski, above, at p 358, where the Supreme Court articulates the rationale underlying the concept of mootness: The doctrine of mootness is an aspect of a general policy or practice that a court may decline to decide a case which raises merely a hypo- thetical or abstract question. The general principle applies when the decision of the court will not have the effect of resolving some con- troversy which affects or may affect the rights of the parties. If the decision of the court will have no practical effect on such rights, the court will decline to decide the case. This essential ingredient must be present not only when the action or proceeding is commenced but at the time when the court is called upon to reach a decision. Accord- ingly if, subsequent to the initiation of the action or proceeding, events occur which affect the relationship of the parties so that no Victoria v. Canada Yves de Montigny J. 337

present live controversy exists which affects the rights of the parties, the case is said to be moot. 32 As I have demonstrated, despite the decision reached by the Immigra- tion Division on May 12, 2011, there remains a real controversy between the parties. The overall conformity of this admissibility hearing with the values enshrined in the Charter is still very much at issue. Accordingly, mootness is not a valid basis upon which this Court ought to or may decline to rule on this application for judicial review.

b) Prematurity 33 Counsel for the Applicant has argued since the inception of the pro- ceedings before the Immigration Division that these proceedings are in violation of s. 7 of the Charter and must therefore be stopped immedi- ately. The gist of this argument is best captured by the following grounds raised in this application for leave and judicial review: 2. The Supreme Court of Canada has held, in Charkaoui, that reli- ance on undisclosed evidence in certificate proceedings in the Fed- eral Court contravened section 7 of the Charter, and that the specific procedure was not justified under s. 1 of the Charter. Section 86 of the current IRPA permits the Immigration Division to rely on the same powers the Federal Court holds under s. 83 of the IRPA in the Applicant’s admissibility hearing. The Court is asked to declare that section 86 contravenes the Charter as the proceeding before the Im- migration Division violates her rights under s. 7 of the Charter and is not justified under s. 1 of the Charter. 3. The Applicant’s section 7 Charter right to security of the person is at stake in the admissibility hearing. The Supreme Court has deter- mined in Singh that the right to security of the person is at stake in a refugee determination hearing. As the Board Member could ulti- mately rule that the Applicant, who is a Convention refugee and per- manent resident, be ordered removed to her country of origin, her security is likewise at stake in the hearing. Furthermore, for a victim of torture who has been determined a Convention refugee, the threat of deportation is serious state-imposed psychological stress. 4. The Applicant is not alleged to be a security threat. There is no justification under s. 1 of the Charter, for refusal to apply s. 7 of the Charter in the context of her admissibility hearing. 5. Sections 86 and 83 of the IRPA do not permit her to engage in full answer and defence. The provision of a Special Advocate is not a substitute for permitting full answer and defence. As the Applicant’s right to security of the person is at stake, and limitation of her rights 338 IMMIGRATION LAW REPORTER 3 Imm. L.R. (4th)

is not made out under s. 1 of the Charter, there should be no deroga- tion of her right to full answer and defence. 34 Counsel for the Applicant also initially argued that another important distinction between an inadmissibility proceeding and a security certifi- cate lies in the fact that the adjudicator of an inadmissibility proceeding is not necessarily a lawyer or law school graduate. The implication be- ing, of course, that a ruling by an Immigration Division member with no legal training would violate the Applicant’s right to natural justice. Sub- sequently, counsel abandoned this argument. 35 As previously mentioned, the Board Member refused to rule on that broad submission in her March 4, 2011 decision, preferring to leave it until she had heard all the evidence and submissions with respect to the Applicant’s admissibility. 36 Counsel for the Applicant forcefully submitted before this Court that there is no reason to wait any longer before ruling on this issue. It was argued that the Applicant has already been trapped in endless litigation for the past six years, enduring the severe stress of such a situation, and that her psychological well-being will be profoundly affected by a deter- mination stripping her of Canada’s protection, branding her as a “terror- ist” and threatening her with potential removal. It is also contended that the Applicant has been financially drained of any means she had to pay for her legal fees, and that it would be a waste of energy and resources to go through the admissibility proceedings if ever the Board Member or this Court eventually agrees with the Applicant. 37 As much as the Court sympathizes with the Applicant’s plight, and despite the skilful arguments put forward by her counsel, there is no legal justification for this Court to intervene at this stage of the proceedings before the Immigration Division. I feel bound to agree with the Respon- dent that the Board’s decision of March 4, 2011 is an interlocutory deci- sion that it is not, as such, reviewable on judicial review (see, for example: C.B. Powell Ltd. c. Canada (Agence des services frontaliers), 2010 FCA 61 (F.C.A.) at para 31, (2010), [2011] 2 F.C.R. 332 (F.C.A.)). 38 The Immigration Division undoubtedly possesses the jurisdiction both to determine the Charter issues raised by the Applicant and to grant relief if it determines that there has been an infringement to the Appli- cant’s rights. Not only is it a court of competent jurisdiction pursuant to ss. 24(1) of the Charter, but ss. 162(1) of IRPA grants each Division of the Board sole and exclusive jurisdiction to hear and determine questions of law and fact, including questions of jurisdiction. Moreover, Rule 47 of Victoria v. Canada Yves de Montigny J. 339

the Rules specifically addresses the procedure for challenging the consti- tutional validity, applicability or operability of any legislative provision under IRPA. The Immigration Division is clearly empowered to deal with the Charter arguments raised by the Applicant, in light of the semi- nal decisions of the Supreme Court (see, Cuddy Chicks Ltd. v. Ontario (Labour Relations Board), [1991] 2 S.C.R. 5 (S.C.C.); Douglas/Kwantlen Faculty Assn. v. Douglas College, [1990] 3 S.C.R. 570 (S.C.C.) and T´etreault-Gadoury v. Canada (Employment & Immigration Commission), [1991] 2 S.C.R. 22 (S.C.C.)). According to these decisions, administrative tribunals endowed with the power to de- cide questions of law, have the authority to resolve constitutional ques- tions that are inextricably linked to matters properly before them, unless such questions have been explicitly withdrawn from their jurisdiction. 39 Recently confronted with the same issue, I held that it is preferable for this Court, as a matter of policy, to rule on Charter issues on the basis of a full evidentiary record and of an informed decision by the adminis- trative tribunal tasked with the responsibility to make findings of fact and law (see, Stables v. Canada (Minister of Citizenship & Immigration), 2011 FC 1319 (F.C.)). I reiterate what I then said in this respect: [27] The Supreme Court has held that tribunals with expertise and authority to decide questions of law are in the best position to hear and decide the constitutionality of their statutory provisions, and should play a primary role in determining Charter issues within their jurisdiction. Writing for the majority in Cuddy Chicks Ltd. v Ontario (Labour Relations Board), [1991] 2 SCR 5 at para 16, Justice LaFor- est captured the usefulness and the value of a tribunal’s factual find- ings when considering a constitutional question in the following terms: It must be emphasized that the process of Charter decision making is not confined to abstract ruminations on consti- tutional theory. In the case of Charter matters which arise in a particular regulatory context, the ability of the deci- sion maker to analyze competing policy concerns is criti- cal...The informed view of the Board, as manifested in a sensitivity to relevant facts and an ability to compile a co- gent record, is also of invaluable assistance. (Quoted with approval by Mr. Justice Gonthier, for a unanimous Court, in Nova Scotia (Workers’ Compensa- tion Board) v Martin, 2003 SCC 54 at para 30, [2003] 2 SCR 504). 340 IMMIGRATION LAW REPORTER 3 Imm. L.R. (4th)

40 This approach is all the more appropriate in the context of an applica- tion for judicial review, where the Court’s mandate is to assess the pro- priety of the Immigration Division’s decision on the issues that it has decided. It would be contrary to the rationale underlying judicial review for a court to pronounce on an issue before the administrative decision- maker had the opportunity to consider it. 41 The March 4, 2011 decision of the Immigration Division is an inter- locutory decision, which does not purport to rule definitively either on the merits of the ss. 34(1)(f) allegation nor on the issue of the Charter applicability to those proceedings. Moreover, there are no special cir- cumstances warranting the immediate judicial review of this interlocu- tory decision. It does not cause the Applicant immediate prejudice that is not capable of being remedied by the administrative tribunal at some later juncture or by this Court, on judicial review of the final decision. 42 Moreover, it is a well established principle that Courts should refrain from deciding constitutional issues when it is not strictly required in or- der to determine a case (see, for example: Borowski, above, at pp 363- 365; Moysa v. Alberta (Labour Relations Board), [1989] 1 S.C.R. 1572 (S.C.C.) at pp 1579-1580; Danson v. Ontario (Attorney General), [1990] 2 S.C.R. 1086 (S.C.C.) at pp 1099-1102). Not only should courts avoid ruling on allegations of Charter infringements in a factual vacuum, but the ultimate decision on the merit may well render an assessment of the Charter issues unnecessary. 43 This is precisely the approach that has been followed by this Court in the comparable security certificate context. As pointed out by counsel for the Respondent, this Court has refused to assess s. 7 Charter claims when insufficient facts to properly assess them are not present. In Almrei, Re, 2008 FC 1216, [2009] 3 F.C.R. 497 (F.C.), Chief Justice Lutfy found that it would be premature to rule on a motion challenging the require- ment that communications among special advocates and other persons must be judicially authorized for lack of conformity with the Charter. Having quashed the security certificate, Justice Mosley eventually found it unnecessary to consider the issue (Almrei, Re, 2009 FC 1263 (F.C.), [2011] FC 1241). In Harkat, Re, 2010 FC 1242, 380 F.T.R. 163 (Eng.) (F.C.), Justice No¨el followed the same course of action ruling on the Charter issues only after having decided on the merits of the security certificate (Harkat, Re, 2010 FC 1241, 380 F.T.R. 61 (Eng.) (F.C.) [Harkat, Re]). Victoria v. Canada Yves de Montigny J. 341

44 On the basis of the foregoing, the Immigration Division Member was similarly justified to reserve her decision on the broad Charter issue until she had the necessary factual foundation to rule on it. She did not close the door on that argument, but merely postponed its assessment until the entirety of the process pursuant to which the Applicant’s admissibility had run its course. Such an approach was entirely legitimate and sensi- ble. Section 85.4 of IRPA grants the Member some flexibility in the ad- ministration of the non-disclosure regime, and it is only upon completion of the process mandated by s. 83, that it will be possible to assess whether the Applicant’s right to a fair hearing is compromised. To rule on the constitutionality of the scheme in the abstract would allow an in- terlocutory motion to take on a life of its own. This in turn may be totally unnecessary and unwarranted if the Immigration Division dismisses the allegation advanced by the Minister on the merit. 45 Finally, it is worth mentioning that the Applicant devotes most of her submissions to the issue of section 7 applicability. The Applicant says very little as to why the use of the nondisclosure regime pursuant to s. 86 of IRPA, in the context of an admissibility hearing, would infringe the principles of fundamental justice. Beyond stating boldly that the Su- preme Court in Charkaoui #2, above, accepted a limitation on the right to make full answer and defence in the context of security certificates on the basis of a security threat, counsel for the Applicant offers very little explanation as to why the non-disclosure regime revamped by Parliament in the wake of that decision, which has been found to be in compliance with the Charter in Harkat, Re, above, and in , Jaballah, Re, 2010 FC 79, [2011] 2 F.C.R. 145 (F.C.), would run afoul of the principles of fun- damental justice in the context of admissibility proceedings. Without prejudging the issue, the alternative rationale suggested by the Respon- dent — that the need to protect sensitive information is the pressing ob- jective of s. 86 — cannot be ruled out as a possible justification of any impairment to the right to full answer and defence. The Respondent has not put forward any evidence pertaining to section 1 of the Charter, as no infringement of s. 7 has yet been found. This Court therefore should re- frain from ruling on this issue, even if the declaration sought by the Ap- plicant is limited to her specific fact situation. 46 This application for judicial review is therefore dismissed. 47 As agreed at the hearing, the parties are invited to submit serious questions of general importance. They shall have fifteen (15) days to do 342 IMMIGRATION LAW REPORTER 3 Imm. L.R. (4th)

so and an additional five (5) days to comment on the questions submit- ted, if any. 48 As for the Applicant’s request that the non-publication Order of my colleague Justice MacTavish be maintained, it has not been opposed by the Respondent and shall be granted. As a result, the following pages of the Certified Tribunal Record shall not be published: 1547-1551 2272-2275 2766-2776 1561-1584 2317-2323 2805-2811 1599-1600 2325 2830-2832 1609-1620 2396-2399 3197-3241 1648 2666-2687 3366-3381 1654 2688-2689 3788-4004 1656 2698-2699 4340-4356 1699-1717 2703-2714 4369-4373 2061-2063 2724-2725 4751-4769 2078-2084 2728-2758

Judgment THIS COURT’S JUDGMENT is that this application for judicial re- view is dismissed. The parties shall have fifteen (15) days to submit questions of general importance for certification purposes, and an addi- tional five (5) days to comment on the questions submitted, if any. THIS COURT ALSO ORDERS THAT the following pages of the Cer- tified Tribunal Record shall not be published: 1547-1551 2272-2275 2766-2776 1561-1584 2317-2323 2805-2811 1599-1600 2325 2830-2832 1609-1620 2396-2399 3197-3241 1648 2666-2687 3366-3381 1654 2688-2689 3788-4004 1656 2698-2699 4340-4356 1699-1717 2703-2714 4369-4373 2061-2063 2724-2725 4751-4769 2078-2084 2728-2758 Application dismissed. Satnarine v. Canada (MCI) 343

[Indexed as: Satnarine v. Canada (Minister of Citizenship & Immigration)] Derek Satnarine, Stephanie Angel Persaud, Applicants and The Minister of Citizenship and Immigration, Respondent Federal Court Docket: T-1179-11 2012 FC 91 D.G. Near J. Heard: January 9, 2012 Judgment: January 24, 2012 Immigration and citizenship –––– Citizenship — Application for grant of or retention of citizenship — Appeals — Jurisdiction –––– DS first met SP and decided to adopt her while she was visiting Toronto in 2005 — She was biologi- cal daughter of his sister currently living in Guyana — SP’s mother did not con- sent to adoption until 2008 — Since March 2008, SP had been living in Canada with DS and had not returned to Guyana — She continued to contact biological parents on Christmas, New Year’s and birthdays — Citizenship Officer refused citizenship application of DS for his adopted daughter, SP — Applicants brought application of judicial review of decision — Application dismissed — Officer was reasonable in assessment of evidence and concern that adoption had been entered into for purpose of acquiring status or privilege in relation to immi- gration or citizenship contrary to subsection 5.1(1)(d) of the Citizenship Act — There was no breach of procedural fairness or fettering of jurisdiction in having third party assistance for editing where officer remained sole decision-maker. Cases considered by D.G. Near J.: Jardine v. Canada (Minister of Citizenship & Immigration) (2011), 2011 Car- swellNat 1981, 2011 FC 565, 2011 CarswellNat 2986, 2011 CF 565, [2011] F.C.J. No. 782 (F.C.) — considered Khosa v. Canada (Minister of Citizenship & Immigration) (2009), 82 Admin. L.R. (4th) 1, 2009 SCC 12, 2009 CarswellNat 434, 2009 CarswellNat 435, 304 D.L.R. (4th) 1, 77 Imm. L.R. (3d) 1, 385 N.R. 206, (sub nom. Canada (Citizenship & Immigration) v. Khosa) [2009] 1 S.C.R. 339, [2009] S.C.J. No. 12 (S.C.C.) — referred to New Brunswick (Board of Management) v. Dunsmuir (2008), 372 N.R. 1, 69 Admin. L.R. (4th) 1, 69 Imm. L.R. (3d) 1, (sub nom. Dunsmuir v. New Brunswick) [2008] 1 S.C.R. 190, 844 A.P.R. 1, (sub nom. Dunsmuir v. New Brunswick) 2008 C.L.L.C. 220-020, D.T.E. 2008T-223, 329 N.B.R. (2d) 1, (sub nom. Dunsmuir v. New Brunswick) 170 L.A.C. (4th) 1, (sub nom. 344 IMMIGRATION LAW REPORTER 3 Imm. L.R. (4th)

Dunsmuir v. New Brunswick) 291 D.L.R. (4th) 577, 2008 CarswellNB 124, 2008 CarswellNB 125, 2008 SCC 9, 64 C.C.E.L. (3d) 1, (sub nom. Dunsmuir v. New Brunswick) 95 L.C.R. 65, [2008] S.C.J. No. 9, [2008] A.C.S. No. 9 (S.C.C.) — referred to Statutes considered: Citizenship Act, R.S.C. 1985, c. C-29 s. 5.1(1)(d) [en. 2007, c. 24, s. 2] — considered

APPLICATION by applicants for judicial review of decision of Citizenship Of- ficer who refused citizenship application of applicant for his adopted daughter.

Micheal Crane, for Applicants Leanne Briscoe, for Respondent

D.G. Near J.:

1 This is an application for judicial review of the decision of a Citizen- ship Officer, dated June 28, 2011, refusing the citizenship application of Derek Satnarine for his adopted daughter, Stephanie Angel Persaud (re- ferred to collectively as the Applicants). The Officer was not satisfied that Stephanie’s adoption was not entered into primarily for the purpose of acquiring a status or privilege in relation to immigration or citizenship under subsection 5.1(1)(d) of the Citizenship Act, RSC, 1985, c C-29 (the Act). 2 For the following reasons, the application is dismissed.

I. Background 3 Derek Satnarine first met Stephanie Angel Persaud and decided to adopt her while she was visiting Toronto in 2005. She is the biological daughter of his sister currently living in Guyana. Stephanie’s mother did not consent to this adoption until 2008. In the intervening period, Derek did not visit Stephanie. 4 Since March 2008, Stephanie has been living in Canada with Derek and has not returned to Guyana. She continues to contact her biological parents on Christmas, New Year’s and birthdays.

II. Decision Under Review 5 In her letter, the Officer reviewed information gleaned during sepa- rate interviews with the Applicants regarding the adoption including that: Satnarine v. Canada (MCI) D.G. Near J. 345

• Derek had adopted Stephanie to help out his sister and ensure a better life in Canada where Stephanie, who excelled academically, could pursue an academic career in accounting at Ryerson University • Stephanie made regular and continuous phone calls to her biologi- cal parents on special occasions and holidays • Stephanie had not gone back to Guyana to visit her biological par- ents or siblings because there is a matter of her getting a visa to come back to Canada • Derek had not opened a Registered Education Savings Plan (RESP) for Stephanie’s future education, stating that he could not because she does not have a Social Insurance Number • When asked whether having an additional family member posed a financial strain, Derek stated that this was only the case when Ste- phanie had to go for medical treatment, since she does not yet have an Ontario Health Card • Stephanie is given an allowance of $500 per month • Stephanie stated she goes to the mall with Derek as part of ‘fun things’ to do as a family • Stephanie has a pending application for permanent residence, filed under humanitarian and compassionate (H&C) considerations • Having spent most of her childhood with her biological family, Stephanie was 14 years old at the time of adoption and had met Derek only once prior 6 Based on this information, the Officer was not satisfied that the adop- tion was not entered into primarily for the purpose of acquiring a status or privilege in relation to immigration or citizenship. The evidence indi- cated the relationship between the Applicants began only after the adop- tion took place in 2008. There was a continuing relationship between Stephanie and her birth mother through regular contact. 7 Moreover, the reasons given as to why the adoption took place were for the purpose of helping out her birth mother and giving Stephanie a better quality of life and education in Canada. As a consequence, the Cit- izenship Officer determined that Derek had failed to establish Stephanie met the requirements for citizenship.

III. Issues 8 This application raises the following issues: 346 IMMIGRATION LAW REPORTER 3 Imm. L.R. (4th)

a) Did the Officer err in her assessment of the evidence? b) Did the Officer breach procedural fairness or fetter jurisdiction by seeking third party assistance in preparing her reasons?

IV. Standard of Review 9 The Officer’s fact-driven inquiry and assessment of evidence necessi- tated by section 5.1 of the Act attracts the reasonableness standard of review (see Jardine v. Canada (Minister of Citizenship & Immigration), 2011 FC 565, [2011] F.C.J. No. 782 (F.C.) at paras 16-17). Based on this standard, the Court should not intervene unless the decision falls outside the range of possible, acceptable outcomes or does not accord with the principles of justification, transparency and intelligibility (see New Brunswick (Board of Management) v. Dunsmuir, 2008 SCC 9, [2008] 1 S.C.R. 190 (S.C.C.) at para 47). 10 By contrast, questions of procedural fairness and jurisdiction, as raised by the Applicants under Issue B, demand the correctness standard (see Khosa v. Canada (Minister of Citizenship & Immigration), 2009 SCC 12, 2009 CarswellNat 434 (S.C.C.) at para 42-43).

V. Analysis A. Did the Officer Err in her Assessment of the Evidence? 11 The Applicants dispute the Officer’s factual findings as being made without regard for the evidence or in a perverse and capricious manner. They insist that their affidavits contradict those findings. 12 Based on the Respondent’s submissions, however, I am not con- vinced that the Officer’s assessment of the evidence would be considered unreasonable. 13 For example, Stephanie’s affidavit contests the Officer’s conclusion that there are “regular and continuous telephone calls” to her biological mother or regular contact more generally. However, she confirms that calls are made on Christmas, New Year’s and birthdays. This is not at odds with the Officer’s summation that there is regular contact on “spe- cial occasions and holidays.” 14 Stephanie’s affidavit acknowledges that she was refused visitor’s visas on 2 occasions, but insists that she was never asked whether the “entire family was refused visitor visas numerous times” as implied by the Officer. I note that the Officer merely stated “it appears” as though Satnarine v. Canada (MCI) D.G. Near J. 347

the information was found in the file rather than attributing this assertion to any specific statement from Stephanie. 15 Derek’s affidavit claims that he “did not state that [he had] not set aside funds for her education” and was only asked about the RESP. The Officer’s statement in this regard is nonetheless consistent with his re- sponse to the question “Have you started an RESP for Stephanie’s future education — college, university?” Notes taken during the interview indi- cate that Derek stated: “No. Cannot open account for her — does not have SIN. I have not set aside funds for university.” 16 Similarly, Derek takes issue with the Officer referring to the question of “whether having an additional family member living with you posed a financial strain” in his affidavit. The Officer asked him more specifically in the interview whether “[a]dopting Stephanie would have incurred ad- ded expense. How has this affected you financially?” Although the word- ing is slightly different, it elicits a similar response. The Officer made reference to Stephanie not having a Health Card as well as Derek paying her bills and a $500 allowance. This evidence is reflected in the Officer’s decision and the Applicants’ affidavits. 17 Derek’s affidavit also elaborates on his reasons for adopting Stepha- nie. He refers to the financial support he could provide, giving the diffi- culties facing his sister in caring for her on her own. Derek expresses his desire to have a child of his own and give Stephanie a better life and education in Canada. The Officer’s assessment that the reasons were to help out her birth mother and give Stephanie a better quality of life and education in Canada reflects these statements and do not amount to mis- construing the evidence. 18 The Applicants have also expressed disbelief in the Officer’s conclu- sion that their relationship began after the adoption took place in 2008. They note that Stephanie started living with Derek in March 2008 but the adoption order was not rendered until December 10, 2008. Prior to living with Derek, however, the Applicants had only met on one occasion in 2005. Irrespective of the exact date of the adoption order, they began living together and only developed any sort of relationship starting in 2008 for the purposes of the adoption contemplated by Derek as early as the first meeting in 2005. The Officer’s conclusion was justified. 19 Although Justice Richard Mosley faulted a citizenship officer for at- tributing no weight to certain key contradictory pieces of evidence relat- ing to an adoption in Jardine, above at para 29, he acknowledged that “deference may still have been owed to the officer and the decision 348 IMMIGRATION LAW REPORTER 3 Imm. L.R. (4th)

found to fall within the range of acceptable outcomes defensible in re- spect of the facts and law had it been clear that the officer properly con- sidered the totality of the evidence.” 20 The Officer considered the totality of the evidence in this case and is entitled to considerable deference in the assessment of facts. There was no key piece of contradictory evidence ignored by the Officer amounting to an error as in Jardine, above. 21 The Applicants’ affidavits confirm much of the information presented in the Officer’s decision and the concerns raised amount to a disagree- ment with the exact wording used and weighing of the evidence. As the Officer’s assessments are within the range of possible, acceptable out- comes, the Court’s intervention is unwarranted.

B. Did the Officer Breach Procedural Fairness or Fetter Jurisdiction by Seeking Third Party Assistance in Preparing her Reasons? 22 The Applicants also take issue with the Officer seeking assistance from a Program Coordinator in the preparation of her reasons. They char- acterize this as an error going to jurisdiction, since the Officer has a stat- utory mandate to consider and render the decision that cannot be shared with others. 23 However, I am prepared to agree with the Respondent that there was no fettering of jurisdiction or breach of procedural fairness in this in- stance. The Program Coordinator provided editing suggestions related to style and formatting or adding certain details. Despite some minor ad- justments to the wording, the drafts and subsequent refusal letter clarify that the Officer reached her own conclusion and wrote the reasons inde- pendently. The suggestions provided did not relate to or affect the final outcome.

VI. Conclusion 24 The Officer was reasonable in her assessment of the evidence and concern that the adoption had been entered into for the purpose of acquir- ing a status or privilege in relation to immigration or citizenship contrary to subsection 5.1(1)(d). There was no breach of procedural fairness or fettering of jurisdiction in having third party assistance for editing where the Officer remains the sole decision-maker. 25 Accordingly, this application is dismissed. Satnarine v. Canada (MCI) D.G. Near J. 349

Judgment THIS COURT’S JUDGMENT is that this application is dismissed. Application dismissed.